Vous êtes sur la page 1sur 232

15

Verbal Section
*******************
Q1:
So dogged were Frances Perkins’ investigations of the garment industry, and her
lobbying for wage and hour reform was persistent, Alfred E. Smith and Franklin D.
Roosevelt recruited Perkins to work within the government, rather than as a social
worker.
A. and her lobbying for wage and hour reform was persistent,
B. and lobbying for wage and hour reform was persistent, so that
C. her lobbying for wage and hour reform persistent, that
D. lobbying for wage and hour reform was so persistent,
E. so persistent her lobbying for wage and hour reform, that
Answer:
*******************
Q2:
Guidebook writer: I have visited hotels throughout the country and have noticed that in
those built before 1930 the quality of the original carpentry work is generally superior to
that in hotels built afterward. Clearly carpenters working on hotels before 1930 typically
worked with more skill, care, and effort than carpenters who have worked on hotels built
subsequently.
Which of the following, if true, most seriously weakens the guidebook writer’s
argument?
A. The quality of original carpentry in hotels is generally far superior to the quality
of original carpentry in other structures, such as houses and stores.
B. Hotels built since 1930 can generally accommodate more guests than those built
before 1930.
C. The materials available to carpenters working before 1930 were not significantly
different in quality from the materials available to carpenters working after 1930.
D. The better the quality of original carpentry in a building, the less likely that
building is to fall into disuse and be demolished.
E. The average length of apprenticeship for carpenters has declined significantly
since 1930.
Answer:
*******************
Q3:
The average hourly wage of television assemblers in Vernland has long been significantly
lower than that in neighboring Borodia. Since Borodia dropped all tariffs on Vernlandian
televisions three years ago, the number of televisions sold annually in Borodia has not
changed. However, recent statistics show a droip in the number of television assemblers
in Borodia. Therefore, updated trade statistics will probably indicate that the number of
televisions Borodia imports annually from Vernland has increased.
16
Which of the following is an assumption on which the argument depends?
A. The number of television assemblers in Vernland has increased by at least as
much as the number of television assemblers in Borodia has decreased.
B. Televisions assembled in Vernland have features that televisions assembled in
Borodia do not have.
C. The average number of hours it takes a Borodian television assembler to assemble
a television has not decreased significantly during the past three years.
D. The number of televisions assembled annually in Vernland has increased
significantly during the past three years.
E. The difference between the hourly wage of television assemblers in Vernland and
the hourly wage of television assemblers in Borodia is likely to decrease in the
next few years.
Answer:
*******************
Q4:
The Quechuans believed that all things participated in both the material level and the
mystical level of reality, and many individual Quechuans claimed to have contact with it
directly with an ichana (dream) experience.
A. contact with it directly with
B. direct contact with it by way of
C. contact with the last directly through
D. direct contact with the latter by means of
E. contact directly with the mystical level due to
Answer:
*******************
Q5 to Q7:
Historians who study European (20) Florentine widow Alessandra
women of the Renaissance try to Strozzi, a
measure historian who specializes in European
“independence,” “options,” and women of the Renaissance attributes
Line other indicators of the degree to individual intention and authorship of
which actions to her subject. This historian
(5) the expression of women’s (25) assumes that Alessandra had goals
individuality and interests different from those of her
was either permitted or suppressed. sons, yet much of the historian’s own
Influenced by Western individualism, research reveals that Alessandra
these historians define a peculiar form acted primarily as a champion of her
of personhood: an innately bounded (30) sons’ interests, taking their goals as
(10) unit, autonomous and standing apart her own. Thus Alessandra conforms
from both nature and society. An more closely to the anthropologist’s
anthropologist, however, would contend notion that personal motivation is
that a person can be conceived in ways embedded in a social context. Indeed,
other than as an “individual.” In many (35) one could argue that Alessandra did
(15) societies a person’s identity is not not distinguish her personhood from
intrinsically unique and self-contained that of her sons. In Renaissance
but instead is defined within a complex Europe the boundaries of the conceptual
17 self were not always firm
web of social relationships. (40) and closed and did not necessarily
In her study of the fifteenth-century coincide with the boundaries of
the bodily self.
--------------------------------------------------------------------------------
Q5:
In the first paragraph, the author of the passage mentions a contention that would be
made by an anthropologist most likely in order to
A. present a theory that will be undermined in the discussion of a historian’s study
later in the passage
B. offer a perspective on the concept of personhood that can usefully be applied to
the study of women in Renaissance Europe
C. undermine the view that the individuality of European women of the Renaissance
was largely suppressed
D. argue that anthropologists have applied the Western concept of individualism in
their research
E. lay the groundwork for the conclusion that Alessandra’s is a unique case among
European women of the Renaissance whose lives have been studied by historians
Answer:
--------------------------------------------------------------------------------
Q6:
According to the passage, much of the research on Alessandra Strozzi done by the
historian mentioned in the second paragraph (lines 19-42) supports which of the
following conclusions?
18
A. Alessandra used her position as her sons’ sole guardian to further interests
different from those of her sons.
B. Alessandra unwillingly sacrificed her own interests in favor of those of her sons.
C. Alessandra’s actions indicate that her motivations and intentions were those of an
independent individual.
D. Alessandra’s social context encouraged her to take independent action.
E. Alessandra regarded her sons’ goals and interests as her own.
Answer:
--------------------------------------------------------------------------------
QX:
It can be inferred that the author of the passage believes which of the following about the
study of Alessandra Strozzi done by the historian mentioned in the second paragraph
(lines 19-42)?
A. Alessandra was atypical of her time and was therefore an inappropriate choice
for the subject of the historian’s research.
B. In order to bolster her thesis, the historian adopted the anthropological
perspective on personhood.
C. The historian argues that the boundaries of the conceptual self were not always
firm and closed in Renaissance Europe.
D. In her study, the historian reverts to a traditional approach that is out of step with
the work of other historians of Renaissance Europe.
E. The interpretation of Alessandra’s actions that the historian puts forward is not
supported by much of the historian’s research.
Answer:
--------------------------------------------------------------------------------
Q7:
The passage suggests that the historian mentioned in the second paragraph (lines 19-42)
would be most likely to agree with which of the following assertions regarding
Alessandra Strozzi?
A. Alessandra was able to act more independently than most women of her time
because she was a widow.
B. Alessandra was aware that her personal motivation was embedded in a social
context.
C. Alessandra had goals and interests similar to those of many other widows in her
society.
D. Alessandra is an example of a Renaissance woman who expressed her
individuality through independent action.
E. Alessandra was exceptional because she was able to effect changes in the social
constraints placed upon women in her society.
Answer:
*******************
Q8:
19
When a new restaurant, Martin’s Cafe, opened in Riverville last year, many people
predicted that business at the Wildflower Inn, Riverville’s only other restaurant, would
suffer from the competition. Surprisingly, however, in the year since Martin’s Cafe
opened, the average number of meals per night served at the Wildflower Inn has
increased significantly.
Which of the following, if true, most helps to explain the increase?
A. Unlike the Wildflower Inn, Martin’s Cafe serves considerably more meals on
weekends than it does on weekdays.
B. Most of the customers of Martin’s Cafe had never dined in Riverville before this
restaurant opened, and on most days Martin’s Cafe attracts more customers than it
can seat.
C. The profit per meal is higher, on average, for meals served at Martin’s Cafe than
for those served at the Wildflower Inn.
D. The Wildflower Inn is not open on Sundays, and therefore Riverville residents
who choose to dine out on that day must either eat at Martin’s Cafe or go to
neighboring towns to eat.
E. A significant proportion of the staff at Martin’s Cafe are people who formerly
worked at the Wildflower Inn and were hired away by the owner of Martin’s
Cafe.
Answer:
*******************
Q9 to Q12: (5) ioral factors such as organizational
Behavior science courses should culture and employee relations are
be gaining prominence in business among the few remaining sources of
school curricula. Recent theoretical sustainable competitive advantage in
Line work convincingly shows why modern organizations. Furthermore,
behav- (10) empirical evidence demonstrates
clear linkages between human sensitive to business norms and
resource (HR) practices based in (30) practices, and current business
the behavioral sciences and various practices have generally been
aspects of a firm’s financial success. moving away from an emphasis on
(15) Additionally, some of the world’s understanding human behavior and
most toward more mechanistic organiza-
successful organizations have made (35) tional models. Furthermore, the
unique HR practices a core element status of HR professionals within
of their overall business strategies. organizations tends to be lower
Yet the behavior sciences than that of other executives.
(20) are struggling for credibility in Students’ perceptions would
many (40) matter less if business schools
business schools. Surveys show were not increasingly dependent on
that business students often regard external funding—form legislatures,
20 businesses, and private foundations—
behavioral studies as peripheral to for survival. Concerned with their
the mainstream business curriculum. (45) institutions’ ability to attract
(25) This perception can be explained by funding,
the fact that business students, hoping administrators are increasingly targeting
to increase their attractiveness to low-enrollment courses and
prospective employers, are highly degree programs for elimination.
--------------------------------------------------------------------------------
Q9:
The primary purpose of the passage is to
A. propose a particular change to business school curricula
B. characterize students’ perceptions of business school curricula
C. predict the consequences of a particular change in business school curricula
D. challenge one explanation for the failure to adopt a particular change in business
school curricula
E. identify factors that have affected the prestige of a particular field in business
school curricula
Answer:
--------------------------------------------------------------------------------
Q10:
The author of the passage mentions “empirical evidence” (line 10) primarily in order to
A. question the value of certain commonly used HR practices
B. illustrate a point about the methodology behind recent theoretical work in the
behavioral sciences
21
C. support a claim about the importance that business schools should place on
courses in the behavioral sciences
D. draw a distinction between two different factors that affect the financial success of
a business
E. explain how the behavioral sciences have shaped HR practices in some business
organizations
Answer:
--------------------------------------------------------------------------------
Q11:
The author of the passage suggests which of the following about HR professionals in
business organizations?
A. They are generally skeptical about the value of mechanistic organizational
models.
B. Their work increasingly relies on an understanding of human behavior.
C. Their work generally has little effect on the financial performance of those
organizations.
D. Their status relative to other business executives affects the attitude of business
school students toward the behavioral sciences.
E. Their practices are unaffected by the relative prominence of the behavioral
sciences within business schools.
Answer:
--------------------------------------------------------------------------------
Q12:
The author of the passage considers each of the following to be a factor that has
contributed to the prevailing attitude in business schools toward the behavioral sciences
EXCEPT
A. business students’ sensitivity to current business norms and practices
B. the relative status of HR professionals among business executives
C. business schools’ reliance on legislatures, businesses, and private foundations for
funding
D. businesses’ tendency to value mechanistic organizational models over an
understanding of human behavior
E. theoretical work on the relationship between behavioral factors and a firm’s
financial performance
Answer:
*******************
Q13:
It is true of both men and women that those who marry as young adults live longer than
those who never marry. This dose not show that marriage causes people to live longer,
since, as compared with other people of the same age, young adults who are about to get
married have fewer of the unhealthy habits that can cause a person to have a shorter life,
most notably smoking and immoderate drinking of alcohol.
Which of the following, if true, most strengthens the argument above?
22
A. Marriage tends to cause people to engage less regularly in sports that involve risk
of bodily harm.
B. A married person who has an unhealthy habit is more likely to give up that habit
than a person with the same habit who is unmarried.
C. A person who smokes is much more likely than a nonsmoker to marry a person
who smokes at the time of marriage, and the same is true for people who drink
alcohol immoderately.
D. Among people who marry as young adults, most of those who give up an
unhealthy habit after marriage do not resume the habit later in life.
E. Among people who as young adults neither drink alcohol immoderately nor
smoke, those who never marry live as long as those who marry.
Answer:
*******************
Q14:
Even though more money was removed out of stock funds in July as in any month since
October 1987, sales of fund shares in July were not as low as an industry trade group had
previously estimated.
A. as in any month since October 1987, sales of fund shares in July were not as low
as
B. as had been in any other month since October 1987, sales of fund shares in July
were not as low as what
C. than there was in any other month since October 1987, sales of fund shares in July
were not as low as that which
D. than in any month since October 1987, sales of fund shares in July were not as
low as
E. than in any other month since October 1987, sales of fund shares in July were not
as low as what
Answer:
*******************
Q15:
Lightbox, Inc., owns almost all of the movie theaters in Washington County and has
announced plans to double the number of movie screens it has in the county within five
years. Yet attendance at Lightbox’s theaters is only just large enough for profitability
now and the county’s population is not expected to increase over the next ten years.
Clearly, therefore, if there is indeed no increase in population, Lightbox’s new screens
are unlikely to prove profitable.
Which of the following, if true about Washington County, most seriously weakens the
argument?
A. Though little change in the size of the population is expected, a pronounced shift
toward a younger, more affluent, and more entertainment-oriented population is
expected to occur.
23
B. The sales of snacks and drinks in its movie theaters account for more of
Lightbox’s profits than ticket sales do.
C. In selecting the mix of movies shown at its theaters, Lightbox’s policy is to avoid
those that appeal to only a small segment of the moviegoing population.
D. Spending on video purchases, as well as spending on video rentals, is currently no
longer increasing.
E. There are no population centers in the county that are not already served by at
least one of the movie theaters that Lightbox owns and operates.
Answer:
*******************
Q16:
Maize contains the vitamin niacin, but not in a form the body can absorb. Pellagra is a
disease that results from niacin deficiency. When maize was introduced into southern
Europe from the Americas in the eighteenth century, it quickly became a dietary staple,
and many Europeans who came to subsist primarily on maize developed pellagra.
Pellagra was virtually unknown at that time in the Americas, however, even among
people who subsisted primarily on maize.
Which of the following, if true, most helps to explain the contrasting incidence of
pellagra described above?
A. Once introduced into southern Europe, maize became popular with landowners
because of its high yields relative to other cereal crops.
B. Maize grown in the Americas contained more niacin than maize grown in Europe
did.
C. Traditional ways of preparing maize in the Americas convert maize’s niacin into a
nutritionally useful form.
D. In southern Europe many of the people who consumed maize also ate niacin-rich
foods.
E. Before the discovery of pellagra’s link with niacin, it was widely believed that the
disease was an infection that could be transmitted from person to person.
Answer:
*******************
Q17:
Today’s technology allows manufacturers to make small cars more fuel-efficient now
than at any time in their production history.
A. small cars more fuel-efficient now than at any time in their
B. small cars that are more fuel-efficient than they were at any time in their
C. small cars that are more fuel-efficient than those at any other time in
D. more fuel-efficient small cars than those at any other time in their
E. more fuel-efficient small cars now than at any time in
Answer:
*******************
Q18:
24
Antarctica receives more solar radiation than does any other place on Earth, yet the
temperatures are so cold and the ice cap is reflective, so that little polar ice melts during
the summer; otherwise, the water levels of the oceans would rise 250 feet and engulf
most of the world’s great cities.
A. is reflective, so that little polar ice melts during the summer; otherwise,
B. is so reflective that little of the polar ice melts during the summer; were it to do
so,
C. so reflective that little polar ice melts during the summer, or else
D. reflective, so that little of the polar ice melts during the summer, or
E. reflects so that little of the polar ice melts during the summer; if it did
Answer:
*******************
Q19:
Historian: In the Drindian Empire, censuses were conducted annually to determine the
population of each village. Village census records for the last half of the 1600’s are
remarkably complete. This very completeness makes one point stand out; in five
different years, villages overwhelmingly reported significant population declines.
Tellingly, each of those five years immediately followed an increase in a certain Drindian
tax. This tax, which was assessed on villages, was computed by the central government
using the annual census figures. Obviously, whenever the tax went up, villages had an
especially powerful economic incentive to minimize the number of people they recorded;
and concealing the size of a village’s population from government census takers would
have been easy. Therefore, it is reasonable to think that the reported declines did not
happen.
In the historian’s argument, the two portions in boldface play which of the following
roles?
A. The first supplies a context for the historian’s argument; the second acknowledges
a consideration that has been used to argue against the position the historian seeks
to establish.
B. The first presents evidence to support the position that the historian seeks to
establish; the second acknowledges a consideration that has been used to argue
against that position.
C. The first provides a context for certain evidence that supports the position that the
historian seeks to establish; the second is that position.
D. The first is a position for which the historian argues; the second is an assumption
that serves as the basis of that argument.
E. The first is an assumption that the historian explicitly makes in arguing for a
certain position; the second acknowledges a consideration that calls that
assumption into question.
Answer:
*******************
Q20:
25
Scientists typically do their most creative work before the age of forty. It is commonly
thought that this happens because aging by itself brings about a loss of creative
capacity. However, studies show that a disproportionately large number of the
scientists who produce highly creative work beyond the age of forty entered their
field at an older age than is usual. Since by the age of forty the large majority of
scientists have been working in their field for at least fifteen years, the studies’ finding
strongly suggests that the real reason why scientists over forty rarely produce highly
creative work is not that they have simply aged but rather that they generally have spent
too long in a given field.
In the argument given, the two portions in boldface play which of the following roles?
A. The first is the position that the argument as a whole opposes; the second is an
objection that has been raised against a position defended in the argument.
B. The first is a claim that has been advanced in support of a position that the
argument opposes; the second is a finding that has been used in support of that
position.
C. The first is an explanation that the argument challenges; the second is a finding
that has been used in support of that explanation.
D. The first is an explanation that the argument challenges; the second is a finding on
which that challenge is based.
E. The first is an explanation that the argument defends; the second is a finding that
has been used to challenge that explanation.
Answer:
*******************
Q21:
In Teruvia, the quantity of rice produced per year is currently just large enough to satisfy
domestic demand. Teruvia’s total rice acreage will not be expanded in the foreseeable
future, nor will rice yields per acre increase appreciably. Teruvia’s population, however,
will be increasing significantly for years to come. Clearly, therefore, Teruvia will soon
have to begin importing rice.
Which of the following is an assumption on which the argument depends?
A. No pronounced trend of decreasing per capita demand for rice is imminent in
Teruvia.
B. Not all of the acreage in Teruvia currently planted with rice is well suited to the
cultivation of rice.
C. None of the strains of rice grown in Teruvia are exceptionally high-yielding.
D. There are no populated regions in Teruvia in which the population will not
increase.
E. There are no major crops other than rice for which domestic production and
domestic demand are currently in balance in Teruvia.
Answer:
*******************
Q22 to Q25:
26
Most pre-1990 literature on businesses’ 1.1 percent between 1973 and 1989,
use of information technology compared
(IT)—defined as any form of computer- with 2.4 percent in the preceding
Line based information system—focused 25-year period. Proponents of IT argued
on (20) that it takes both time and a critical
(5) spectacular IT successes and mass
reflected of investment for IT to yield benefits,
a general optimism concerning IT’s and
potential some suggested that growth figures for
as a resource for creating competitive the 1990’s proved these benefits were
advantage. But toward the end of the finally being realized. They also argued
1980’s, some economists spoke of a (25) that measures of productivity ignore
(10) “productivity paradox”: despite what
huge IT would have happened without
investments, most notably in the service investments
sectors, productivity stagnated. In the in IT—productivity gains might have
retail industry, for example, in which IT been
had been widely adopted during the even lower. There were even claims that
(15) 1980’s, productivity (average output IT had improved the performance of the
per (30) service sector significantly,
hour) rose at an average annual rate of although macroeconomic
measures of productivity did
not reflect the improvement. of any direct effect was found, the
But some observers questioned why, frequent
if IT had conferred economic value, it negative correlations between IT
did and performance suggested that IT had
(35) not produce direct competitive (50) probably weakened some firms’
advantages competitive
for individual firms. Resource-based positions. However, firms’ human
theory offers an answer, asserting that, resources, in and of themselves, did
in general, firms gain competitive explain improved performance, and
advantages some firms gained IT-related advan-
by accumulating resources that are (55) tages by merging IT with
(40) economically valuable, relatively complementary
scarce, resources, particularly human resources.
and not easily replicated. According to The findings support the notion, founded
a recent study of retail firms, which in resource-based theory, that
confirmed competitive
that IT has become pervasive advantages do not arise from easily
and relatively easy to acquire, IT by (60) replicated resources, no matter how
(45) itself appeared to have conferred impressive or economically valuable
little they may be, but from complex,
advantage. In fact, though little evidence intangible
27 resources.
--------------------------------------------------------------------------------
Q22:
The passage is primarily concerned with
A. describing a resource and indicating various methods used to study it
B. presenting a theory and offering an opposing point of view
C. providing an explanation for unexpected findings
D. demonstrating why a particular theory is unfounded
E. resolving a disagreement regarding the uses of a technology
Answer:
--------------------------------------------------------------------------------
Q23:
The passage suggests that proponents of resource-based theory would be likely to explain
IT’s inability to produce direct competitive advantages for individual firms by pointing
out that
A. IT is not a resource that is difficult to obtain
B. IT is not an economically valuable resource
C. IT is a complex, intangible resource
D. economic progress has resulted from IT only in the service sector
E. changes brought about by IT cannot be detected by macroeconomic measures
Answer:
--------------------------------------------------------------------------------
Q24:
The author of the passage discusses productivity in the retail industry in the first
paragraph primarily in order to
A. suggest a way in which IT can be used to create a competitive advantage
B. provide an illustration of the “productivity paradox”
28
C. emphasize the practical value of the introduction of IT
D. cite an industry in which productivity did not stagnate during the 1980’s
E. counter the argument that IT could potentially create competitive advantage
Answer:
--------------------------------------------------------------------------------
Q25:
According to the passage, most pre-1990 literature on businesses’ use of IT included
which of the following?
A. Recommendations regarding effective ways to use IT to gain competitive
advantage
B. Explanations of the advantages and disadvantages of adopting IT
C. Information about ways in which IT combined with human resources could be
used to increase competitive advantage
D. A warning regarding the negative effect on competitive advantage that would
occur if IT were not adopted
E. A belief in the likelihood of increased competitive advantage for firms using IT
Answer:
*******************
Q26:
Healthy lungs produce a natural antibiotic that protects them from infection by routinely
killing harmful bacteria on airway surfaces. People with cystic fibrosis, however, are
unable to fight off such bacteria, even though their lungs produce normal amounts of the
antibiotic. The fluid on airway surfaces in the lungs of people with cystic fibrosis has an
abnormally high salt concentration; accordingly, scientists hypothesize that the high salt
concentration is what makes the antibiotic ineffective.
Which of the following, if true, most strongly supports the scientists’ hypothesis?
A. When the salt concentration of the fluid on the airway surfaces of healthy people
is raised artificially, the salt concentration soon returns to normal.
B. A sample of the antibiotic was capable of killing bacteria in an environment with
an unusually low concentration of salt.
C. When lung tissue from people with cystic fibrosis is maintained in a solution with
a normal salt concentration, the tissue can resist bacteria.
D. Many lung infections can be treated by applying synthetic antibiotics to the
airway surfaces.
E. High salt concentrations have an antibiotic effect in many circumstances.
Answer:
*******************
Q27:
In April 1997, Hillary Rodham Clinton hosted an all-day White House scientific
conference on new findings that indicates a child’s acquiring language, thinking, and
emotional skills as an active process that may be largely completed before age three.
A. that indicates a child’s acquiring language, thinking, and emotional skills as
29
B. that are indicative of a child acquiring language, thinking, and emotional skills as
C. to indicate that when a child acquires language, thinking, and emotional skills,
that it is
D. indicating that a child’s acquisition of language, thinking, and emotional skills is
E. indicative of a child’s acquisition of language, thinking, and emotional skills as
Answer:
*******************
Q28:
Wind farms, which generate electricity using arrays of thousands of wind-powered
turbines, require vast expanses of open land. County X and County Y have similar
terrain, but the population density of County X is significantly higher than that of County
Y. Therefore, a wind farm proposed for one of the two counties should be built in
County Y rather than in County X.
Which of the following, if true, most seriously weakens the planner’s argument?
A. County X and County Y are adjacent to each other, and both are located in the
windiest area of the state.
B. The total population of County Y is substantially greater than that of County X.
C. Some of the electricity generated by wind farms in County Y would be purchased
by users outside the county.
D. Wind farms require more land per unit of electricity generated than does any other
type of electrical-generation facility.
E. Nearly all of County X’s population is concentrated in a small part of the county,
while County Y’s population is spread evenly throughout the country.
Answer:
*******************
Q29:
Over the past five years, the price gap between name-brand cereals and less expensive
store-brand cereals has become so wide that consumers have been switching increasingly
to store brands despite the name brands’ reputation for better quality. To attract these
consumers back, several manufacturers of name-brand cereals plan to narrow the price
gap between their cereals and store brands to less than what it was five years ago.
Which of the following, if true, most seriously calls into question the likelihood that the
manufacturers’ plan will succeed in attracting back a large percentage of consumers who
have switched to store brands?
A. There is no significant difference among manufacturers of name-brand cereals in
the prices they charge for their products.
B. Consumers who have switched to store-brand cereals have generally been
satisfied with the quality of those cereals.
C. Many consumers would never think of switching to store-brand cereals because
they believe the name brand cereals to be of better quality.
D. Because of lower advertising costs, stores are able to offer their own brands of
cereals at significantly lower prices than those charged for name-brand cereals.
30
E. Total annual sales of cereals—including both name-brand and store-brand
cereals—have not increased significantly over the past five years
Answer:
*******************
Q30:
Which of the following most logically completes the argument?
The irradiation of food kills bacteria and thus retards spoilage. However, it also lowers
the nutritional value of many foods. For example, irradiation destroys a significant
percentage of whatever vitamin B1 a food may contain. Proponents of irradiation point
out that irradiation is no worse in this respect than cooking. However, this fact is either
beside the point, since much irradiated food is eaten raw, or else misleading, since
_______.
A. many of the proponents of irradiation are food distributors who gain from food’s
having a longer shelf life
B. it is clear that killing bacteria that may be present on food is not the only effect
that irradiation has
C. cooking is usually the final step in preparing food for consumption, whereas
irradiation serves to ensure a longer shelf life for perishable foods
D. certain kinds of cooking are, in fact, even more destructive of vitamin B1 than
carefully controlled irradiation is
E. for food that is both irradiated and cooked, the reduction of vitamin B1 associated
with either process individually is compounded
Answer:
*******************
Q31:
Studies in restaurants show that the tips left by customers who pay their bill in cash tend
to be larger when the bill is presented on a tray that bears a credit-card logo. Consumer
psychologists hypothesize that simply seeing a credit-card logo makes many credit-card
holders willing to spend more because it reminds them that their spending power exceeds
the cash they have immediately available.
Which of the following, if true, most strongly supports the psychologists’ interpretation
of the studies?
A. The effect noted in the studies is not limited to patrons who have credit cards.
B. Patrons who are under financial pressure from their credit-card obligations tend to
tip less when presented with a restaurant bill on a tray with credit-card logo than
when the tray has no logo.
C. In virtually all of the cases in the studies, the patrons who paid bills in cash did
not possess credit cards.
D. In general, restaurant patrons who pay their bills in cash leave larger tips than do
those who pay by credit card.
31
E. The percentage of restaurant bills paid with given brand of credit card increases
when that credit card’s logo is displayed on the tray with which the bill is
prepared.
Answer:
*******************
Q32:
Which of the following most logically completes the argument?
Although the pesticide TDX has been widely used by fruit growers since the early
1960’s, a regulation in force since 1960 has prohibited sale of fruit on which any TDX
residue can be detected. That regulation is about to be replaced by one that allows sale of
fruit on which trace amounts of TDX residue are detected. In fact, however, the change
will not allow more TDX on fruit than was allowed in the 1960’s, because ______.
A. pre-1970 techniques for detecting TDX residue could detect it only when it was
present on fruit in more than the trace amounts allowed by the new regulations
B. many more people today than in the 1960’s habitually purchase and eat fruit
without making an effort to clean residues off the fruit
C. people today do not individually consume any more pieces of fruit, on average,
than did the people in the 1960’s
D. at least a small fraction of the fruit sold each year since the early 1960’s has had
on it greater levels of TDX than the regulation allows
E. the presence of TDX on fruit in greater than trace amounts has not been shown to
cause any harm even to children who eat large amounts of fruit
Answer:
*******************
Q33:
Wolves generally avoid human settlements. For this reason, domestic sheep, though
essentially easy prey for wolves, are not usually attacked by them. In Hylantia prior to
1910, farmers nevertheless lost considerable numbers of sheep to wolves each year.
Attributing this to the large number for wolves, in 1910 the government began offering
rewards to hunters for killing wolves. From 1910 to 1915, large numbers of wolves were
killed. Yet wolf attacks on sheep increased significantly.
Which of the following, if true, most helps to explain the increase in wolf attacks on
sheep?
A. Populations of deer and other wild animals that wolves typically prey on
increased significantly in numbers from 1910 to 1915.
B. Prior to 1910, there were no legal restrictions in Hylantia on the hunting of
wolves.
C. After 1910 hunters shot and wounded a substantial number of wolves, thereby
greatly diminishing these wolves’ ability to prey on wild animals.
D. Domestic sheep are significantly less able than most wild animals to defend
themselves against wolf attacks.
32
E. The systematic hunting of wolves encouraged by the program drove many wolves
in Hylantia to migrate to remote mountain areas uninhabited by humans.
Answer:
*******************
Q34:
Many entomologists say that campaigns to eradicate the fire ant in the United States have
failed because the chemicals that were used were effective only in wiping out the ant’s
natural enemies, which made it easier for them to spread.
A. which made it easier for them
B. which makes it easier for it
C. thus making it easier for them
D. thus making it easier for the ant
E. thereby, it was made easier for the ant
Answer:
*******************
Q35 to Q37:
Even more than mountainside slides when one aspen—a single stem—dies,
of mud or snow, naturally occurring (20) the entire clone is affected. While
forest alive,
fires promote the survival of aspen trees. a stem sends hormones into the root
Line Aspens’ need for fire may seem system to suppress formation of further
illogical stems. But when the stem dies, its
(5) since aspens are particularly hormone signal also ceases. If a clone
vulnerable (25) loses many stems simultaneously,
to fires; whereas the bark of most trees the
consists of dead cells, the aspen’s bark resulting hormonal imbalance triggers a
is a living, functioning tissue that— huge increase in new, rapidly growing
along shoots that can outnumber the ones
with the rest of the tree—succumbs destroyed. An aspen grove needs to
quickly 33
(10) to fire. (30) experience fire or some other
The explanation is that each aspen, disturbance
while appearing to exist separately as regularly, or it will fail to regenerate and
a single tree, is in fact only the stem or spread. Instead, coniferous trees will
shoot of a far larger organism. A group invade the aspen grove’s borders and
(15) of thousands of aspens can actually increasingly block out sunlight needed
constitute a single organism, called a by
clone, that shares an interconnected root the aspens.
system and a unique set of genes. Thus,
--------------------------------------------------------------------------------
Q35:
The primary purpose of the passage is to explain the
A. qualities that make a particular organism unique
B. evolutionary change undergone by a particular organism
C. reasons that a phenomenon benefits a particular organism
D. way in which two particular organisms compete for a resource
E. means by which a particular organism has been able to survive in a barren region
Answer:
--------------------------------------------------------------------------------
Q36:
It can be inferred from the passage that when aspen groves experience a “disturbance”
(line 30), such a disturbance
A. leads to a hormonal imbalance within an aspen clone
B. provides soil conditions that are favorable for new shoots
C. thins out aspen groves that have become overly dense
D. suppresses the formation of too many new aspen stems
E. protects aspen groves by primarily destroying coniferous trees rather than aspens
Answer:
--------------------------------------------------------------------------------
Q37:
The author of the passage refers to “the bark of most trees” (line 6) most likely in order to
emphasize the
A. vulnerability of aspens to damage from fire when compared to other trees
B. rapidity with which trees other than aspens succumb to destruction by fire
C. relatively great degree of difficulty with which aspens catch on fire when
compared to other trees
D. difference in appearance between the bark of aspens and that of other trees
E. benefits of fire to the survival of various types of trees
Answer:
*******************
Q38:
Nitrogen dioxide is a pollutant emitted by automobiles. Catalytic converters, devices
designed to reduce nitrogen dioxide emissions, have been required in all new cars in
Donia since 1993, and as a result, nitrogen dioxide emissions have been significantly
reduced throughout most of the country. Yet although the proportion of new cars in
34
Donia’s capital city has always been comparatively high, nitrogen dioxide emissions
there have showed only an insignificant decline since 1993.
Which of the following, if true, most helps to explain the insignificant decline in nitrogen
dioxide emissions in Donia’s capital city?
A. More of the cars in Donia’s capital city were made before 1993 than after 1993.
B. The number of new cars sold per year in Donia has declined slightly since 1993.
C. Pollutants other than nitrogen dioxide that are emitted by automobiles have also
been significantly reduced in Donia since 1993.
D. Many Donians who own cars made before 1993 have had catalytic converters
installed in their cars.
E. Most car trips in Donia’s capital city are too short for the catalytic converter to
reach its effective working temperature.
Answer:
*******************
Q39:
In 2000, a mere two dozen products accounted for half the increase in spending on
prescription drugs, a phenomenon that is explained not just because of more expensive
drugs but by the fact that doctors are writing many more prescriptions for higher-cost
drugs.
A. a phenomenon that is explained not just because of more expensive drugs but by
the fact that doctors are writing
B. a phenomenon that is explained not just by the fact that drugs are becoming more
expensive but also by the fact that doctors are writing
C. a phenomenon occurring not just because of drugs that are becoming more
expensive but because of doctors having also written
D. which occurred not just because drugs are becoming more expensive but doctors
are also writing
E. which occurred not just because of more expensive drugs but because doctors
have also written
Answer:
*******************
Q40:
Community activist: If Morganville wants to keep its central shopping district healthy, it
should prevent the opening of a huge SaveAll discount department store on the outskirts
of Morganville. Records from other small towns show that whenever SaveAll has
opened a store outside the central shopping district of a small town, within five years the
town has experienced the bankruptcies of more than a quarter of the stores in the
shopping district.
The answer to which of the following would be most useful for evaluating the community
activist’s reasoning?
35
A. Have community activists in other towns successfully campaigned against the
opening of a SaveAll store on the outskirts of their towns?
B. Do a large percentage of the residents of Morganville currently do almost all of
their shopping at stores in Morganville?
C. In towns with healthy central shopping districts, what proportion of the stores in
those districts suffer bankruptcy during a typical five-year period?
D. What proportion of the employees at the SaveAll store on the outskirts of
Morganville will be drawn form Morganville?
E. Do newly opened SaveAll stores ever lose money during their first five years of
operation?
Answer:
*******************
Q41:
In ancient Thailand, much of the local artisans’ creative energy was expended for the
creation of Buddha images and when they constructed and decorated the temples that
enshrined them.
A. much of the local artisans’ creative energy was expended for the creation of
Buddha images and when they constructed and decorated the temples that
enshrined them
B. much of the local artisans’ creative energy was expended on the creation of
Buddha images and on construction and decoration of the temples in which they
were enshrined
C. much of the local artisans’ creative energy was expended on the creation of
Buddha images as well as constructing and decoration of the temples in which
they were enshrined
D. creating images of Buddha accounted for much of the local artisans’ creative
energy, and also constructing and decorating the temples enshrining them
E. the creating of Buddha images accounted for much of the local artisans’ creative
energy as well as construction and decoration of the temples that enshrined them
Answer:
*******************
*******************
Answers:
EDCDB, E(E)DBEC, DEEEA, CCBCD, ACABE, CDEBE, BACDC, AAEBC, B
AWA: managers and policies, even when the
AA employees believe that the managers and
36 policies
Read the statement and the instructions are misguided."
that follow it, and then make any notes Discuss the extent to which you agree or
that will disagree with the opinion stated above.
help you plan your response. Begin Support
typing your response in the box at the your views with reasons and/or
bottom of the examples from your own experience,
screen. observations, or
The following appeared as part of a reading.
business plan created by the ********************************
management of the Take ********************************
Heart Fitness Center. ******
"After opening the new swimming pool Math:
early last summer, Take Heart saw a 12 *******************
percent Q1:
increase in the use of the center by its A student’s average (arithmetic
mean) test score on 4 tests is 78.
members.
What must be the student’s
Therefore, in order to increase score on a 5th test for the student’s
membership in Take Heart, we should average score on the 5 tests to be
continue to add new 80?
recreational facilities in subsequent A. 80
years: for example, a multipurpose game B. 82
room, a C. 84
tennis court, and a miniature golf course. D. 86
E. 88
Being the only center in the area offering Answer:
this *******************
range of activities would give us a Q2:
competitive ,' ,advantage in the health From a group of 3 boys and 3 girls, 4
and recreation children are to be randomly selected.
market." What is the
Discuss how well reasoned.., etc. probability that equal numbers of
AI boys and girls will be selected?
A.
Read the statement and the instructions
10
that follow it, and then make any notes
1
that will
B.
help you plan your response. Begin
9
typing your response in the box at the
4
bottom of the C.
screen. 2
"Employees should show loyalty to their 1
company by fully supporting the D.
company's 5
3
E. B. Statement (2) ALONE is sufficient,
3 but statement (1) alone is not
2 sufficient.
Answer: C. BOTH statements TOGETHER are
******************* sufficient, but NEITHER statement
Q3: ALONE is
If x and y are positive integers and 1 sufficient.
+ x + y + xy = 15, what is the value D. EACH statement ALONE is
of x + y? sufficient.
A. 3 E. Statements (1) and (2) TOGETHER
B. 5 are NOT sufficient.
C. 6 Answer:
D. 8 *******************
E. 9 Q6:
Answer: If r, s, and w are positive numbers
******************* such that w = 60r + 80s and r + s = 1,
Q4: is w < 70?
If Ann saves x dollars each week and (1) r > 0.5
Beth saves y dollars each week, what (2) r > s
is the total amount that A. Statement (1) ALONE is sufficient,
they save per week? but statement (2) alone is not
(1) Beth saves $5 more per week sufficient.
than Ann saves per week. B. Statement (2) ALONE is sufficient,
(2) It takes Ann 6 weeks to save the but statement (1) alone is not
same amount that Beth saves in 5 sufficient.
weeks. C. BOTH statements TOGETHER are
A. Statement (1) ALONE is sufficient, sufficient, but NEITHER statement
but statement (2) alone is not ALONE is
sufficient. sufficient.
B. Statement (2) ALONE is sufficient, D. EACH statement ALONE is
but statement (1) alone is not sufficient.
sufficient. E. Statements (1) and (2) TOGETHER
C. BOTH statements TOGETHER are are NOT sufficient.
sufficient, but NEITHER statement Answer:
ALONE is *******************
sufficient. Q7:
D. EACH statement ALONE is If a code word is defined to be a
sufficient. sequence of different letters chosen
E. Statements (1) and (2) TOGETHER from the 10 letters A, B, C, D,
are NOT sufficient. E, F, G, H, I, and J, what is the ratio
Answer: of the number of 5-letter code words
******************* to the number of 4-letter
Q5: code words?
If x is a positive integer, is the A. 5 to 4
remainder 0 when 3x + 1 is divided B. 3 to 2
by 10? C. 2 to 1
(1) x = 4n + 2, where n is a positive D. 5 to 1
integer. E. 6 to 1
(2) x > 4 Answer:
A. Statement (1) ALONE is sufficient, *******************
but statement (2) alone is not Q8:
sufficient.
Which of the following fractions has a 35
decimal equivalent that is a 17
terminating decimal? E.
A. 2
189 1
10 Answer:
B. *******************
196 Q10:
15 A certain machine produces 1,000
C. units of product P per hour. Working
225 continuously at this
16 constant rate, this machine will
D. produce how many units of product P
144 in 7 days?
A. 7,000
25 B. 24,000
E.
C. 40,000
128 D. 100,000
39 E. 168,000
Answer: Answer:
******************* ---------------------------------------------
Q9: ---------------------------------------------
Company S produces two kinds of
stereos: basic and deluxe. Of the
-------------
stereos produced by -----
Company S last month, Q11:
3 If 5x – 5x - 3 = (124)(5y), what is y in
terms of x?
2
were basic and the rest were deluxe.
A. x
B. x - 6
If it takes
C. x - 3
5 D. 2x + 3
7 E. 2x + 6
as many hours Answer:
to produce a deluxe stereo as it does *******************
to produce a basic stereo, then the Q12:
number of hours it took to Lines k and m are parallel to each
produce the deluxe stereos last other. Is the slope of line k positive?
month was what fraction of the total (1) Line k passes through the point
number of hours it took to (3, 2).
produce all the stereos? (2) Line m passes through the point
A. (-3, 2).
17 A. Statement (1) ALONE is sufficient,
7 but statement (2) alone is not
B. sufficient.
31 B. Statement (2) ALONE is sufficient,
14 but statement (1) alone is not
C. sufficient.
15 C. BOTH statements TOGETHER are
sufficient, but NEITHER statement
7
ALONE is
D.
sufficient.
D. EACH statement ALONE is C. III only
sufficient. D. I and III only
E. Statements (1) and (2) TOGETHER E. I, II, and III
are NOT sufficient. Answer:
Answer: *******************
******************* Q15:
Q13: +xyz
Theater M has 25 rows with 27 seats dm
in each row. How many of the seats en
were occupied during a f
certain show? The figure above shows two entries,
(1) During the show, there was indicated by m and n, in an addition
table. What is the value
an average (arithmetic mean) of of n + m?
10 unoccupied (1) d + y = -3
seats per row for the front 20 (2) e + z = 12
rows. A. Statement (1) ALONE is sufficient,
(2) During the show, there was an but statement (2) alone is not
average (arithmetic mean) of 20 sufficient.
unoccupied seats per B. Statement (2) ALONE is sufficient,
row for the back 15 rows. but statement (1) alone is not
A. Statement (1) ALONE is sufficient, sufficient.
but statement (2) alone is not C. BOTH statements TOGETHER are
sufficient. sufficient, but NEITHER statement
B. Statement (2) ALONE is sufficient, ALONE is
but statement (1) alone is not sufficient.
sufficient. D. EACH statement ALONE is
C. BOTH statements TOGETHER are sufficient.
sufficient, but NEITHER statement E. Statements (1) and (2) TOGETHER
ALONE is are NOT sufficient.
sufficient. Answer:
D. EACH statement ALONE is *******************
sufficient. Q16:
E. Statements (1) and (2) TOGETHER On a map Town G is 10 centimeters
are NOT sufficient. due east of Town H and 8
Answer: centimeters due south of Town J.
******************* Which of the following is closest to
Q14: the straight-line distance, in
The number of defects in the first centimeters, between Town H and
five cars to come through a new Town J on the map?
production line are 9, 7, 10, 4, A. 6
and 6, respectively. If the sixth car
B. 13
through the production line has
either 3, 7, or 12 defects, for C. 18
which of theses values does the D. 20
mean number of defects per car for E. 24
the first six cars equal the Answer:
median? *******************
I. 3 Q17:
II. 7 If x is the product of the positive
III. 12 integers from 1 to 8, inclusive, and if
A. I only i, k, m, and p are positive
B. II only
integers such that x = 2i3k5m7p, then i 1
+ k+m+ p= r
F. 4 B.
G. 7 22
H. 8 1
I. 11 r
J. 12 C.
Answer: 32
*******************
Q18:
1
In the xy-plane, the line with equation r
ax + by + c = 0, where abc ≠ 0, has D.
slope 3
3 1
2 r
. What is the E.
value of b? 23
(1) a = 4 1
(2) c = −6 r
A. Statement (1) ALONE is sufficient,
Answer:
but statement (2) alone is not
*******************
sufficient.
Q20:
B. Statement (2) ALONE is sufficient,
Leo can buy a certain computer for
but statement (1) alone is not
p1 dollars in State A, where the sales
sufficient.
tax is t1 percent, or he can
C. BOTH statements TOGETHER are
buy the same computer for p2 dollars
sufficient, but NEITHER statement
in State B, where the sales tax is t2
ALONE is
percent. Is the total cost
sufficient.
of the computer greater in State A
D. EACH statement ALONE is
than in State B?
sufficient.
E. Statements (1) and (2) TOGETHER
(1) t1 > t2
are NOT sufficient. (2) p1t1 > p2t2
Answer: A. Statement (1) ALONE is sufficient,
******************* but statement (2) alone is not
Q19: sufficient.
This year Henry will save a certain B. Statement (2) ALONE is sufficient,
amount of his income, and he will but statement (1) alone is not
spend the rest. Next year sufficient.
Henry will have no income, but for C. BOTH statements TOGETHER are
each dollar that he saves this year, sufficient, but NEITHER statement
he will have 1 + r dollars ALONE is
available to spend. In terms of r, sufficient.
what fraction of his income should D. EACH statement ALONE is
Henry save this year so that sufficient.
next year the amount he was E. Statements (1) and (2) TOGETHER
available to spend will be equal to are NOT sufficient.
half the amount that he spends this Answer:
year? *******************
Q21:
A. Is x an odd integer?
2
(1) x + 3 is an even integer. B. Statement (2) ALONE is sufficient,
(2) but statement (1) alone is not
3 sufficient.
x C. BOTH statements TOGETHER are
is an odd integer. sufficient, but NEITHER statement
A. Statement (1) ALONE is sufficient, ALONE is
but statement (2) alone is not sufficient.
sufficient. D. EACH statement ALONE is
B. Statement (2) ALONE is sufficient, sufficient.
but statement (1) alone is not E. Statements (1) and (2) TOGETHER
sufficient. are NOT sufficient.
C. BOTH statements TOGETHER are Answer:
sufficient, but NEITHER statement *******************
ALONE is Q24:
sufficient. If
D. EACH statement ALONE is 3
sufficient. 7x
E. Statements (1) and (2) TOGETHER + a = 8 and a > 8, then, in terms of a,
are NOT sufficient. 7
Answer: 3x
******************* =
Q22: A.
The surface of a certain planet a
reflects 80 percent of the light that
strikes it. The clouds around
1
the planet then absorb 40 percent of +
the reflected light. What percent of 8
the light that strikes the 1
planet is reflected from the surface B.
and passes through the clouds 8
without being absorbed? 1
A. 32% -
B. 40% a
C. 48% 1
D. 60% C.
E. 88% 8a
Answer:
*******************
1
Q23: D.
The selling price of an article is equal 7
to the cost of the article plus the 3
markup. The markup on a (8 – a)
certain television set is what percent E.
of the selling price? 49
(1) The markup on the television 9
set is 25 percent of the cost. (8 – a)
(2) The selling price of the television
Answer:
set is $250.
*******************
A. Statement (1) ALONE is sufficient,
Q25:
but statement (2) alone is not
What is the sum of a certain pair of
sufficient.
consecutive odd integers?
(1) At least one of the integers is A. Statement (1) ALONE is sufficient,
negative. but statement (2) alone is not
(2) At least one of the integers is sufficient.
positive. B. Statement (2) ALONE is sufficient,
A. Statement (1) ALONE is sufficient, but statement (1) alone is not
but statement (2) alone is not sufficient.
sufficient. D
B. Statement (2) ALONE is sufficient,
but statement (1) alone is not
sufficient. B
C. BOTH statements TOGETHER are C. BOTH statements TOGETHER are
sufficient, but NEITHER statement sufficient, but NEITHER statement
ALONE is ALONE is
sufficient. sufficient.
D. EACH statement ALONE is D. EACH statement ALONE is
sufficient. sufficient.
E. Statements (1) and (2) TOGETHER E. Statements (1) and (2) TOGETHER
are NOT sufficient. are NOT sufficient.
Answer: Answer:
******************* *******************
Q26: Q28:
If an automobile averaged 22.5 miles A certain company assigns
per gallon of gasoline, approximately employees to offices in such a way
how many kilometers that some of the offices can be
per liter of gasoline did the empty and more than one employee
automobile average? (1 mile = 1.6 can be assigned to an office. In how
kilometers and 1 gallon = 3.8 many ways can the
liters, both rounded to the nearest company assign 3 employees to 2
tenth.) different offices?
A. 5
A. 3.7
B. 6
B. 9.5 C. 7
C. 31.4 D. 8
D. 53.4 E. 9
E. 136.8 Answer:
Answer: *******************
******************* Q29:
Q27: If the sequence x1, x2, x3, …, xn, … is
AE such that x1 = 3 and xn+1 = 2xn – 1 for
In the figure, points A, B, C, D, and E n ≥ 1, then x20 – x19 =
lie on a line. A is on both circles, B is A. 219
the center of the B. 220
smaller circle, C is the center of the C. 221
larger circle, D is on the smaller D. 220 - 1
circle, and E is on the larger E. 221 - 1
circle. What is the area of the region Answer:
inside the larger circle and outside *******************
the smaller circle? Q30:
(1) AB = 3 and BC =2 If y is the smallest positive integer
(2) CD =1 and DE = 4 such that 3,150 multiplied by y is the
square of an integer, then
y must be
A. 2 Candidate M 40 20 40
B. 5 Candidate N 30 35 35
C. 6 The table above shows the
D. 7
E. 14
results of a survey of 100 voters
Answer: each responded
******************* “favorable” or “unfavorable” or
Q31: “not sure” when asked about
A certain quantity is measured on their impressions of
two different scales, the R-scale and candidate M and of candidate N.
the S-scale, that are related What was the number of voters
linearly. Measurements on the R-
scale of 6 and 24 correspond to
who responded
measurements on the S-scale of “favorable” for both candidates?
30 and 60, respectively. What (1) The number of voters who did not
measurement on the R-scale respond “favorable” for either
corresponds to a measurement of candidate was 40.
100 (2) The number of voters who
on the S-scale? responded “unfavorable” for both
A. 20 candidates was 10.
B. 36 A. Statement (1) ALONE is sufficient,
C. 48 but statement (2) alone is not
D. 60 sufficient.
E. 84 B. Statement (2) ALONE is sufficient,
Answer: but statement (1) alone is not
******************* sufficient.
Q32: C. BOTH statements TOGETHER are
If x3y4 = 5,000, is y = 5? sufficient, but NEITHER statement
(1) y is a positive integer. ALONE is
(2) x is an integer. sufficient.
A. Statement (1) ALONE is sufficient, D. EACH statement ALONE is
but statement (2) alone is not sufficient.
sufficient. E. Statements (1) and (2) TOGETHER
B. Statement (2) ALONE is sufficient, are NOT sufficient.
but statement (1) alone is not Answer:
sufficient. *******************
C. BOTH statements TOGETHER are Q34:
sufficient, but NEITHER statement If n is a positive integer, what is the
ALONE is remainder when 38n+3 + 2 is divided
sufficient. by 5?
D. EACH statement ALONE is A. 0
sufficient. B. 1
E. Statements (1) and (2) TOGETHER C. 2
are NOT sufficient. D. 3
Answer: E. 4
--------------------------------------------- Answer:
*******************
--------------------------------------------- Q35:
------------- The interior of a rectangular carton is
----- designed by a certain manufacturer
Q33: to have a volume of x
Favorable Unfavorable Not Sure
cubic feet and a ratio of length to B. 16
width to height of 3 : 2 : 2. In terms C. 17
of x, which of the D. 18
following equals the height of the E. 19
carton, in feet? Answer:
A. 3 x *******************
B. 3 Q37:
3 How many different factors does the
2x integer n have?
C. 3
(1) n = a4b3, where a and b are
different positive prime numbers.
2 (2) The only positive prime numbers
3x that are factors of n are 5 and 7.
D. 3
A. Statement (1) ALONE is sufficient,
3 but statement (2) alone is not
2 sufficient.
x B. Statement (2) ALONE is sufficient,
E. 3 but statement (1) alone is not
2 sufficient.
3 C. BOTH statements TOGETHER are
sufficient, but NEITHER statement
x
ALONE is
Answer:
sufficient.
*******************
D. EACH statement ALONE is
Q36:
sufficient.
How many integers from 0 to 50,
E. Statements (1) and (2) TOGETHER
inclusive, have a remainder of 1
are NOT sufficient.
when divided by 3 ?
Answer:
A. 15
*******************----
Answer:
EDCCA, DEEAE, CEEDC, BDAEE, DCAEC, BADAE,CCAAB,CA

Verbal Section
*******************
Q1:
Scientists claim that the discovery of the first authenticated mammal bones in amber
could provide important clues of determining, in addition to how, when mammals
colonized the islands of the West Indies.
A. of determining, in addition to how, when mammals colonized the islands of the
West Indies
B. in the determination of how and when the islands of the West Indies were
colonized by mammals
C. to determine how mammals colonized the islands of the West Indies and when
they did
D. for determining when the islands of the West Indies were colonized by mammals
and how they were
E. for determining how and when mammals colonized the islands of the West Indies
Answer:
---------------------------------------------------------------------------------------------------------
Q2:
Although women’s wages are improving, Department of Labor statistics show that the
ratio of their earnings with that of men have been roughly static since 1960.
A. with that of men have been
B. to that of men are
C. to those of men have been
D. with those of men is
E. to those of men has been
Answer:
*******************
Q3 to Q6:
Social learning in animals experiment showed that
is said to occur when direct house sparrows consumed
or indirect social interaction less red food after they
Line facilitates the acquisition observed others eating
(5) of a novel behavior. It (40) red food that was treated
usually takes the form of so as to be noxious. Studies
an experienced animal (the on nonavian species have
demonstrator) performing a not produced similar results,
behavior such that the naive leading researchers to
(10) animal (the observer) subsequently (45) speculate that avian social
expresses the learning may be fundamentally
same behavior sooner, or different from that of
more completely, than it mammals.
would have otherwise. One But Sherwin’s recent
(15) example of social learning (50) experiments with domestic
is the acquisition of preferences hens do not support the
for novel foods. notion that avian social
Some experiments learning necessarily facilitates
have suggested that among aversion to novel
(20) mammals, social learning (55) foods that are noxious or
facilitates the identification toxic. Even when demonstrator
of beneficial food items, hens reacted with
but that among birds, social obvious disgust to a specific
learning helps animals food, via vigorous head
(25) avoid toxic substances. (60) shaking and bill wiping,
For example, one study there was no evidence that
showed that when red-wing observers subsequently
blackbirds observed others avoided eating that food.
consuming a colored food Sherwin’s research team
(30) or a food in a distinctly (65) speculated that ecological
marked container and then or social constraints during
becoming ill, they subsequently the evolution of this species
avoided food might have resulted in there
associated with that color being little benefit from the
(35) or container. Another (70) social learning of unpalatability,
for instance, showed that if observer
selective pressures for this hens watched demonstrator
mode of learning would be hens react favorably to food
reduced if the birds rarely (85) of a particular color, then
(75) encountered noxious or observer hens ate more
toxic food or rarely interacted food of that color than they
after eating such food, ate of food of other colors.
or if the consequences of These results confirmed that
ingestion were minimal. (90) avian species can develop
(80) In a related experiment, preferences for palatable
the same researchers food through social learning.
--------------------------------------------------------------------------------
Q3:
The primary purpose of the passage is to discuss the
A. techniques used in certain experiments on social learning in birds
B. reasons for the differences between social learning in birds and in mammals
C. question of how social learning manifests itself in birds
D. basis for a widespread belief about a difference in behavior between birds and
mammals
E. possible reasons why birds may or may not learn from each other in a particular
way
Answer:
--------------------------------------------------------------------------------
Q4:
Missing!
--------------------------------------------------------------------------------
Q5:
According to the passage, which of the following is true of the experiments on domestic
hens conducted by Sherwin’s research team?
A. Only a small number of observer hens appeared to learn to avoid food that was
demonstrated by other hens to be noxious.
B. Observer hens ingested food preferentially only after numerous instances of
witnessing demonstrator hens preferentially ingest that type of food.
C. Observer hens appeared unable to recognize when demonstrator hens found a
particular food especially palatable.
D. Demonstrator hens reacted adversely to ingesting certain novel foods.
E. Demonstrator hens altered their behavior less obviously in response to noxious
foods than in response to highly palatable foods.
Answer:
--------------------------------------------------------------------------------
Q6:
It can be inferred that the author of the passage would be most likely to agree with which
of the following statements regarding the results of the recent experiments conducted by
Sherwin’s research team?
A. The experiments demonstrate that social learning in avian species facilitates the
identification of noxious or toxic foods.
B. The experiments suggest that social learning has made avian species less adept
than nonavian species at learning to prefer beneficial foods and avoid noxious and
toxic foods.
C. The experiments undermine the notion that most avian species have evolved in
environments where there is little benefit to the social learning of unpalatability.
D. The experiments suggest that the acquisition of food preferences in avian species
is largely unaffected by social learning.
E. The experiments show that social learning in avian species can promote the
preferential consumption of beneficial foods but do not support the claim that
social learning in avian species promotes the avoidance of noxious or toxic foods.
Answer:
*******************
Q7:
In 1960’s studies of rats, scientists found that crowding increases the number of attacks
among the animals significantly. But in recent experiments in which rhesus monkeys
were placed in crowded conditions, although there was an increase in instances of
“coping” behavior—such as submissive gestures and avoidance of dominant
individuals—attacks did not become any more frequent. Therefore it is not likely that,
for any species of monkey, crowding increases aggression as significantly as was seen in
rats.
Which of the following, if true, most strengthens the argument?
A. All the observed forms of coping behavior can be found among rhesus monkeys
living in uncrowded conditions.
B. In the studies of rats, nondominant individuals were found to increasingly avoid
dominant individuals when the animals were in crowded conditions.
C. Rhesus monkeys respond with aggression to a wider range of stimuli than any
other monkeys do.
D. Some individual monkeys in the experiment were involved in significantly more
attacks than the other monkeys were.
E. Some of the coping behavior displayed by rhesus monkeys is similar to behavior
rhesus monkeys use to bring to an end an attack that has begun.
Answer:
-----------------------------------------------------------------------------------------------------
Q8:
In the Louisiana Purchase of 1803, the United States acquired 828,000 square miles for
about four cents an acre, which more than doubled the country’s size and that brought its
western border within reach of the Pacific Ocean.
A. In the Louisiana Purchase of 1803, the United States acquired 828,000 square
miles for about four cents an acre, which more than doubled the country’s size
and that brought
B. For about four cents an acre the United States acquired, in the Louisiana Purchase
of 1803, 828,000 square miles, more than doubling the country’s size and it
brought
C. With the Louisiana Purchase in 1803, the United States acquired 828,000 square
miles for about four cents an acre, more than doubling its size and bringing
D. The United States, in the Louisiana Purchase of 1803, for about four cents an
acre, acquired 828,000 square miles, more than doubling the country’s size,
bringing
E. Acquiring 828,000 square miles in the Louisiana Purchase of 1803, the United
States bought it for about four cents an acre, more than doubling the country’s
size and bringing
Answer:
---------------------------------------------------------------------------------------------------------
Q9:
Early in the twentieth century, Lake Konfa became very polluted. Recently fish
populations have recovered as release of industrial pollutants has declined and the lake’s
waters have become cleaner. Fears are now being voiced that the planned construction of
an oil pipeline across the lake’s bottom might revive pollution and cause the fish
population to decline again. However, a technology for preventing leaks is being
installed. Therefore, provided this technology is effective, those fears are groundless.
The argument depends on assuming which of the following?
A. Apart from development related to the pipeline, there will be no new industrial
development around the lake that will create renewed pollution in its waters.
B. Other than the possibility of a leak, there is no realistic pollution threat posed to
the lake by the pipeline’s construction.
C. There is no reason to believe that the leak-preventing technology would be
ineffective when installed in the pipeline in Lake Konfa.
D. Damage to the lake’s fish populations would be the only harm that a leak of oil
from the pipeline would cause.
E. The species of fish that are present in Lake Konfa now are the same as those that
were in the lake before it was affected by pollution.
Answer:
*******************
Q10:
In an attempt to attract more tourists, the Swiss government commissioned several
lifesize
fiberglass cow statues, having them decorated by local artists, and set them up on the
streets of Zurich.
A. the Swiss government commissioned several life-size fiberglass cow statues,
having them decorated by local artists, and
B. the Swiss government commissioned several life-size fiberglass cow statues, they
were decorated by local artists, and then
C. the Swiss government commissioned several life-size fiberglass cow statues, had
them decorated by local artists, and
D. several life-size fiberglass cow statues were commissioned by the Swiss
government, decorated by local artists, and then
E. several life-size fiberglass cow statues, commissioned by the Swiss government,
were decorated by local artists, and they
Answer:
*******************
Q11:
Trancorp currently transports all its goods to Burland Island by truck. The only bridge
over the channel separating Burland from the mainland is congested, and trucks typically
spend hours in traffic. Trains can reach the channel more quickly than trucks, and freight
cars can be transported to Burland by barges that typically cross the channel in an hour.
Therefore, to reduce shipping time, Trancorp plans to switch to trains and barges to
transport goods to Burland.
Which of the following would be most important to know in determining whether
Trancorp’s plan, if implemented, is likely to achieve its goal?
A .Whether transportation by train and barge would be substantially less expensive
than transportation by truck.
B. Whether there are boats that can make the trip between the mainland and Burland
faster than barges can
C. Whether loading the freight cars onto barges is very time consuming
D. Whether the average number of vehicles traveling over the bridge into Burland
has been relatively constant in recent years
E. Whether most trucks transporting goods into Burland return to the mainland
empty
Answer:
*******************
Q12:
Over the next few years, increasing demands on the Chattahoochee River, which flows
into the Apalachicola River, could alter the saline content of Apalachicola Bay, which
would rob the oysters there of their flavor, and to make them decrease in size, less
distinctive, and less in demand.
A. which would rob the oysters there of their flavor, and to make them decrease in
size
B. and it would rob the oysters there of their flavor, make them smaller
C. and rob the oysters there of their flavor, making them decrease in size
D. robbing the oysters there of their flavor and making them smaller
E. robbing the oysters there of their flavor, and making them decrease in size
Answer:
*******************
Q13 to Q15:
Carotenoids, a family of natural prefer males with brighter carotenoid-
pigments, based
form an important art of the colorful (10) coloration. Owens and Olson
signals hypothesize that
used by many animals. Animals acquire the presence of carotenoids, as signaled
Line carotenoids either directly (from the by
plants coloration, would be meaningful in the
(5) and algae that produce them) or context
indirectly (by of mate selection if carotenoids were
eating insects) and store them in a either
variety of rare or required for health. The
tissues. Studies of several animal species conventional
have shown that when choosing mates, (15) view is that carotenoids are
females meaningful
because they are rare: healthier males can use scarce carotenoids either for
can immune
forage for more of the pigments than can (25) defense and detoxification or for
their attracting
inferior counterparts. Although this may females. Males that are more susceptible
be to
true, there is growing evidence that disease and parasites will have to use
carotenoids their
(20) are meaningful also because they carotenoids to boost their immune
are required: systems,
they are used by the immune system and whereas males that are genetically
for resistant
detoxification processes that are (30) will use fewer carotenoids for
important fighting disease
for maintaining health. It may be that and will advertise this by using the
males pigments
for flashy display instead.
--------------------------------------------------------------------------------
Q13:
Information in the passage suggests that which of the following is true of carotenoids that
a male animal uses for detoxification processes?
A. They were not acquired directly from plants and algae.
B. They cannot be replenished through foraging.
C. They cannot be used simultaneously to brighten coloration.
D. They do not affect the animal’s susceptibility to parasites.
E. They increase the chances that the animal will be selected as a mate.
Answer:
--------------------------------------------------------------------------------
Q14:
The passage suggests that relatively bright carotenoid-based coloration is a signal of
which of the following characteristics in males of certain animal species?
A. Readiness for mating behavior
B. Ability to fight
C. Particular feeding preferences
D. Recovery from parasite infestation
E. Fitness as a mate
Answer:
--------------------------------------------------------------------------------
Q15:
According to the “conventional view” referred to in lines 14-15 of the passage, brighter
carotenoid-based coloration in certain species suggests that an individual
A. lives in a habitat rich in carotenoid-bearing plants and insects
B. has efficient detoxification processes
C. has a superior immune system
D. possesses superior foraging capacity
E. is currently capable of reproducing
Answer:
*******************
Q16:
Paper&Print is a chain of British stores selling magazines, books, and stationery
products. In Britain, magazines’ retail prices are set by publishers, and the retailer’s
share of a magazine’s retail price is 25 percent. Since Paper&Print’s margin on books
and stationery products is much higher, the chain’s management plans to devote more of
its stores’ shelf space to books and stationery products and reduce the number of
magazine titles that its stores carry.
Which of the following, if true, most strongly argues that the plan, if put into effect, will
not increase Paper&Print’s profits?
A. Recently magazine publishers, seeking to increase share in competitive sectors of
the market, have been competitively cutting the retail prices of some of the largest
circulation magazines.
B. In market research surveys, few consumers identify Paper&Print as a book or
stationery store but many recognize and value the broad range of magazines it
carries.
C. The publisher’s share of a magazine’s retail price is 50 percent, and the publisher
also retains all of the magazine’s advertising revenue.
D. Consumers who subscribe to a magazine generally pay less per issue than they
would if they bought the magazine through a retail outlet such as Paper&Print.
E. Some of Paper& Print’s locations are in small towns and represent the only retail
outlet for books within the community.
Answer:
*******************
Q17:
In the nation of Partoria, large trucks currently account for 6 percent of miles driven on
Partoria’s roads but are involved in 12 percent of all highway fatalities. The very largest
trucks—those with three trailers—had less than a third of the accident rate of single-and
double-trailer trucks. Clearly, therefore, one way for Partoria to reduce highway deaths
would be to require shippers to increase their use of triple-trailer trucks.
Which of the following, if true, most seriously weakens the argument?
A. Partorian trucking companies have so far used triple-trailer trucks on lightly
traveled sections of major highways only.
B. No matter what changes Partoria makes in the regulation of trucking, it will have
to keep some smaller roads off-limits to all large trucks.
C. Very few fatal collisions involving trucks in Partoria are collisions between two
trucks.
D. In Partoria, the safety record of the trucking industry as a whole has improved
slightly over the past ten years.
E. In Partoria, the maximum legal payload of a triple-trailer truck is less than three
times the maximum legal payload of the largest of the single-trailer trucks.
Answer: A
*******************
Q 18:
Which of the following most logically completes the passage?
Concerned about financial well-being of its elderly citizens, the government of
Runagiadecided two years ago to increase by 20 percent the governmentprovided
pensionpaid to all Runagians over 65. Inflation in the intervening
period has been negligible,and the increase has been duly received by all eligible
Runagians. Nevertheless,many of them are no better off financially than they
were before the increase, in large part because ________.
A. They rely entirely on the government pension for their income
B. Runagian banks are so inefficient that it can take up to three weeks to cash a
pension check
C. They buy goods whose prices tend to rise especially fast in times of
inflation
D. The pension was increased when the number of elderly Runagians below
the poverty level reached an all-time high
E. In Runagia children typically supplement the income of elderly parents, but
only by enough to provide them with a comfortable living
Answer:
*******************
Q19:
Plankton generally thrive in areas of the ocean with sufficient concentrations of certain
nitrogen compounds near the surface, where plankton live. Nevertheless, some areas,
though rich in these nitrogen compounds, have few plankton. These areas have
particularly low concentrations of iron, and oceanographers hypothesize that this shortage
of iron prevents plankton from thriving. However, an experimental release of iron
compounds into one such area failed to produce a thriving plankton population, even
though local iron concentrations increased immediately.
Which of the following, if true, argues most strongly against concluding, on the basis of
the information above, that the oceanographers’ hypothesis is false?
A. Not all of the nitrogen compounds that are sometimes found in relatively high
concentrations in the oceans are nutrients for plankton.
B. Certain areas of the ocean support an abundance of plankton despite having
particularly low concentrations of iron.
C. The release of the iron compounds did not increase the supply of nitrogen
compounds in the area.
D. A few days after the iron compounds were released, ocean currents displaced the
iron-rich water from the surface.
E. The iron compounds released into the area occur naturally in areas of the ocean
where plankton thrive.
Answer:
------------------------------------------------------------------------------------------------
Q20:
In the 1820’s, anthracite coalfields opened in eastern Pennsylvania, and when canals and
eventually railroads reached into the region, there were substantial supplies of coal that
was exported to the nation’s growing mills, forges, factories, and railways.
A. when canals and eventually railroads reached into the region, there were
substantial supplies of coal that was
B. once canals and eventually railroads reaching into the region, there were
substantial supplies of coal having been
C. with canals and eventually railroads reaching into the region, there had been
substantial supplies of coal
D. canals and eventually railroads reached into the region, with substantial supplies
of coal being
E. as canals and eventually railroads reached into the region, substantial supplies of
coal were
Answer:
*******************
Q21:
According to entomologists, single locusts are quiet creatures, but when locusts are
placed with others of their species, they become excited, change color, vibrate, and even
hum.
A. when locusts are placed with others of their species,
B. when placing them with others of their species, then
C. locusts, when placed with others of their species,
D. placing them with others of their species,
E. placed with others of their species, then
Answer:
*******************
Q22:
A study by the Ocean Wildlife Campaign urged states to undertake a number of remedies
to reverse a decline in the shark population, which includes the establishment of size
limits for shark catches, closing state waters for shark fishing during pupping season, and
requiring commercial fishers to have federal shark permits.
A. which includes the establishment of size limits for shark catches, closing
B. which includes establishing limits to the size of sharks that can be caught, closing
C. which include the establishment of size limits for shark catches, the closing of
D. including establishing size limits for shark catches, closing
E. including the establishing of limits to the size of sharks that are caught, the
closing of
Answer:
*******************
Q23:
Goronian lawmaker: Goronia’s Cheese Importation Board, the agency responsible for
inspecting all wholesale shipments of cheese entering Goronia from abroad and rejecting
shipments that fail to meet specified standards, rejects about one percent of the cheese
that it inspects. Since the health consequences and associated costs of not rejecting that
one percent would be negligible, whereas the cost of maintaining the agency is not, the
agency’s cost clearly outweighs the benefits it provides.
Knowing the answer to which of the following would be most useful in evaluating the
lawmaker’s argument?
A. Are any of the types of cheeses that are imported into Goronia also produced in
Goronia?
B. Has the Cheese Importation Board, over the last several years, reduced its
operating costs by eliminating inefficiencies within the agency itself?
C. Does the possibility of having merchandise rejected by the Cheese Importation
Board deter many cheese exporters from shipping substandard cheese to Goronia?
D. Are there any exporters of cheese to Goronia whose merchandise is never rejected
by the Cheese Importation Board?
E. How is the cheese rejected by the Cheese Importation Board disposed of?
Answer:
*******************
Q24:
Until the passage of the Piracy and Counterfeiting Amendments Act in 1982, a first-time
charge of copyright infringement was merely a misdemeanor charge, federal prosecutors
being unlikely in pursuing criminal copyright infringers, while offenders were subject to
relatively small penalties.
A. charge, federal prosecutors being unlikely in pursuing criminal copyright
infringers, while offenders were
B. charge, with federal prosecutors who were unlikely to pursue criminal copyright
infringers, offenders being
C. charge, federal prosecutors unlikely to pursue criminal copyright infringers, while
offenders were
D. charge; therefore, federal prosecutors were unlikely in pursuing criminal
copyright infringers and offenders being
E. charge; therefore, federal prosecutors were unlikely to pursue criminal copyright
infringers, and offenders were
Answer:
*******************
Q25 to Q27:
Many politicians, business leaders, for disabling medical conditions must
and scholars discount the role of (20) also allow them for maternity and
public policy and emphasize the role that
Line of the labor market when failure to do so would constitute sex
explaining discrimination under the Civil Rights
(5) employers’ maternity-leave policies, Act of 1964. As early as 1973, a
arguing that prior to the passage of survey found that 58 percent of large
the Family and Medical Leave Act (25) employers had responded with new
(FMLA) of 1993, employers were maternity-leave policies. Because the
already providing maternity leave in 1972 EEOC ruling was contested in
(10) response to the increase in the court, the ruling won press attention
number that popularized maternity-leave
of women workers. Employers did (30) policies. Yet perhaps because the
create maternity-leave programs Supreme Court later struck down
in the 1970’s and 1980’s, but not as the ruling, politicians and scholars
a purely voluntary response in the have failed to recognize its effects,
(15) absence of any government assuming that employers adopted
mandate. (35) maternity-leave policies in response
In 1972, the Equal Employment to the growing feminization of the
Opportunity Commission (EEOC) ruled workforce.
that employers who allowed leaves
--------------------------------------------------------------------------------
Q25:
It can be inferred that the author of the passage would be most likely to agree with which
of the following statements about government policy?
A. Government policy is generally unaffected by pressures in the labor market.
B. The impact of a given government policy is generally weakened by sustained
press attention.
C. It is possible for a particular government policy to continue to have an impact
after that policy has been eliminated.
D. A given government policy can be counterproductive when that policy has
already unofficially been implemented.
E. The impact of a given government policy is generally weakened when the ruling
is contested in court.
Answer:
--------------------------------------------------------------------------------
Q26: Missing!
--------------------------------------------------------------------------------
Q27:
NOTE: You must scroll to read the answer choices for this question.
The passage suggests that the relationship between the view of the author with respect to
maternity leave policy prior to passage of the FMLA and the view of the politicians,
business leaders, and scholars mentioned in lines 1-2 can best be characterized by which
of the following statements?
A. They agree that both the 1972 EEOC ruling on maternity-leave policy and the
increasing feminization of the workplace had an impact on employers’ creation of
maternity-leave programs but disagree about the relative importance of each
factor.
B. They agree that the EEOC ruling on maternity-leave policy had an initial impact
on employers’ creation of maternity-leave programs but disagree over whether the
Supreme Court’s striking down of the EEOC ruling weakened that impact.
C. They agree that creating maternity-leave programs was a necessary response to
the needs of the increasing number of women workers but disagree about whether
maternity should be classified as a disabling medical condition.
D. They agree that employers created maternity-leave programs prior to passage of
the FMLA but disagree about employers’ motivations for doing so.
E. They agree that employers created maternity-leave programs prior to passage of
the FMLA but disagree about how widespread those programs were.
Answer:
*******************
Q28:
Which of the following most logically completes the argument?
A certain cultivated herb is one of a group of closely related plants that thrive in soil with
high concentrations of metals that are toxic to most other plants. Agronomists studying
the herb have discovered that it produces large amounts of histidine, an amino acid that,
in test-tube solutions, renders these metals chemically inert. Possibly, therefore, the
herb’s high histidine production is what allows it to grow in metal-rich soils, a hypothesis
that would gain support if ______.
A. histidine is found in all parts of the plant—roots, stem, leaves, and flowers
B. the herb’s high level of histidine production is found to be associated with an
unusually low level of production of other amino acids
C. others of the closely related group of plants are also found to produce histidine in
large quantities
D. cultivation of the herb in soil with high concentrations of the metals will, over an
extended period, make the soil suitable for plants to which the metals are toxic
E. the concentration of histidine in the growing herb declines as the plant approaches
maturity
Answer:
*******************
Q29:
Lofgren’s disease has been observed frequently in commercially raised cattle but very
rarely in chickens. Both cattle and chickens raised for meat are often fed the type of feed
that transmits the virus that causes the disease. Animals infected with the virus take more
than a year to develop symptoms of Lofgren’s disease, however, and chickens
commercially raised for meat, unlike cattle, are generally brought to market during their
first year of life.
Which of the following is most strongly supported by the information provided?
A. The virus that causes Lofgren’s disease cannot be transmitted to human beings by
chickens.
B. There is no way to determine whether a chicken is infected with the Lofgren’s
disease virus before the chicken shows symptoms of the disease.
C. A failure to observe Lofgren’s disease in commercial chicken populations is not
good evidence that chickens are immune to the virus that causes this disease.
D. An animal that has been infected with the virus that causes Lofgren’s disease but
that has not developed symptoms cannot transmit the disease to an uninfected
animal of the same species.
E. The feed that chickens and cattle are fed is probably not the only source of the
virus that causes Lofgren’s disease.
Answer:
*******************
Q30:
Unlike the body’s inflammatory response to cuts and sprains, with widespread swelling
and stiffness immobilizing the injured area until it heals, the body’s response to sunburn
is more localized and resulting in a distinct line dividing affected and unaffected areas of
the skin.
A. with widespread swelling and stiffness immobilizing the injured area until it
heals, the body’s response to sunburn is more localized and resulting
B. where the injured area is immobilized with widespread swelling and stiffness until
it has healed, sunburn generates a more localized response in the body, which
results instead
C. with the injured area immobilized by means of widespread swelling and stiffness
until healing, sunburn generates a more localized response in the body, one
resulting
D. in which widespread swelling and stiffness immobilize the injured area until it has
healed, the body’s more localized response to sunburn results
E. in which widespread swelling and stiffness immobilize the injured area until
healing, instead, the body’s more localized response to sunburn results
Answer:
*******************
Q31:
Many of the earliest known images of Hindu deities in India date from the time of the
Kushan empire, fashioned either from the spotted sandstone of Mathura or Gandharan
grey schist.
A. empire, fashioned either from the spotted sandstone of Mathura or
B. empire, fashioned from either the spotted sandstone of Mathura or from
C. empire, either fashioned from the spotted sandstone of Mathura or
D. empire and either fashioned from the spotted sandstone of Mathura or from
E. empire and were fashioned either from the spotted sandstone of Mathura or from
Answer:
*******************
Q32:
A prominent investor who holds a large stake in the Burton Tool Company has recently
claimed that , citing as evidence
company’s failure to
slow production in response to a recent rise in its inventory of finished products. It is
doubtful whether an investor’s sniping at management can ever be anything other than
counterproductive, but
It is true that an increased
inventory of finished products often indicates that production is outstripping demand, but
in Burton’s case it indicates no such thing. Rather, the increase in inventory is entirely
attributable to products that have already been assigned to orders received from
customers.
In the argument given, the two portions play which of the following
roles?
A. The first states the position that the argument as a whole opposes; the second
provides evidence to undermine the support for the position being opposed.
B. The first states the position that the argument as a whole opposes; the second is
evidence that has been used to support the position being opposed.
C. The first states the position that the argument as a whole opposes; the second
states the conclusion of the argument as a whole.
D. The first is evidence that has been used to support a position that the argument as
a whole opposes; the second provides information to undermine the force of that
evidence.
E. The first is evidence that has been used to support a position that the argument as
a whole opposes; the second states the conclusion of the argument as a whole.
Answer:
*******************
Q33 to Q36:
Customer loyalty programs are
attempts to bond customers to a special deal. Most buyers who change
company brands are not lost forever; usually,
and its products and services by they are heavy consumers who simply
Line offering incentives—such as airline (35) prefer to buy a number of brands.
(5) frequent flyer programs or special Such multibrand loyalty means that one
credit cards with valuable benefits—to company’s most profitable customers
loyal customers. In support of loyalty will probably be its competitors’ most
programs, companies often invoke the profitable customers as well.
“80/20” principle, which states that (40) Still, advocates of loyalty programs
(10) about 80 percent of revenue contend that such programs are
typically beneficial
comes from only about 20 percent of because the costs of serving
customers. However, this profitable highly loyal customers are lower, and
20 percent are not necessarily loyal because such loyal customers are less
buyers, especially in the sense of (45) price sensitive than other customers.
(15) exclusive loyalty. Studies have It is true that when there are start-up
demonstrated costs, such as credit checks, involved
that only about 10 percent in serving a new customer, the costs
of buyers for many types of frequently exceed those of serving a repeat cus-
purchased consumer goods are (50) tomer. However, it is not at all clear
100 percent loyal to a particular brand why the costs of serving a highly loyal
(20) over a one-year period. Moreover, customer should in principle be different
100-percent-loyal buyers tend to be from those of serving any other
light buyers of the product or service. type of repeat customer. The key
“Divided loyalty” better describes (55) variables driving cost are size and
actual consumer behavior, since type of order, special versus standard
(25) customers typically vary the brands order, and so on, not high-loyalty
they buy. The reasons for this versus divided-loyalty customers.
behavior are fairly straightforward: As for price sensitivity, highly loyal
people buy different brands for different (60) customers may in fact come to
occasions or for variety, or a expect
(30) brand may be the only one in stock a price discount as a reward for their
or loyalty.
may offer better value because of a
--------------------------------------------------------------------------------
Q33:
The primary purpose of the passage is to
A. question the notion that customer loyalty programs are beneficial
B. examine the reasons why many customers buy multiple brands of products
C. propose some possible alternatives to customer loyalty programs
D. demonstrate that most customers are not completely loyal to any one brand of
product or service
E. compare the benefits of customer loyalty programs with those of other types of
purchase incentive programs
Answer:
--------------------------------------------------------------------------------
Q34:
Missing!
--------------------------------------------------------------------------------
Q35:
Missing!
--------------------------------------------------------------------------------
Q36:
Missing!
*******************
Q37:
Roughly one-half of the world’s population, including virtually all of East and Southeast
Asia also, is wholly dependent on rice to be its staple food.
A. including virtually all of East and Southeast Asia also, is wholly dependent on
rice to be
B. including virtually all of East and Southeast Asia, is wholly dependent on rice as
C. virtually all of East and Southeast Asia as well, wholly dependent on rice as
D. which includes virtually all of East and Southeast Asia’s, being wholly dependent
on rice as
E. which includes virtually all of East and Southeast Asia also, is wholly dependent
on rice to be
Answer:
*******************
Q38:
A significant number of complex repair jobs carried out by Ace Repairs have to be
reworked under the company’s warranty. The reworked jobs are invariably satisfactory.
When initial repairs are inadequate, therefore, it is not because the mechanics lack
competence; rather, there is clearly a level of focused concentration that complex repairs
require that is elicited more reliably by rework jobs than by first-time jobs.
The argument above assumes which of the following?
A. There is no systematic difference in membership between the group of mechanics
who do first-time jobs and the group of those who do rework jobs.
B. There is no company that successfully competes with Ace Repairs for complex
repair jobs.
C. Ace Repairs’ warranty is good on first-time jobs but does not cover rework jobs.
D. Ace Repairs does not in any way penalize mechanics who have worked on
complex repair jobs that later had to be reworked.
E. There is no category of repair jobs in which Ace Repairs invariably carries out
first-time jobs satisfactorily.
Answer:
*******************
Q39:
After increasing steadily for centuries, the total annual catch of all wild fish peaked in
1989; since that time, the total catch for most species has declined or plateaued,
prompting conservation efforts designed to help prevent population crashes and possible
extinctions.
A. fish peaked in 1989; since that time, the total catch for most species has declined
or plateaued, prompting
B. fish peaked in 1989, but with the total catch since then declining or plateauing in
the case of most species, prompting
C. fish had peaked in 1989; since that time, the decline or plateau of the total catch
for most species, which prompted
D. fish, which peaked in 1989, and, in the case of most species, it has declined or
plateaued since, prompted
E. fish, which peaked in 1989 but has since declined or plateaued for most species,
and this prompted
Answer:
*******************
Q40:
In Kravonia, the average salary for jobs requiring a college degree has always been
higher than the average salary for jobs that do not require a degree. Current enrollments
in Kravonia’s colleges indicate that over the next four years the percentage of the
Kravonian workforce with college degrees will increase dramatically. Therefore, the
average salary for all workers in Kravonia is likely to increase over the next four years.
Which of the following is an assumption on which the argument depends?
A. Kravonians with more than one college degree earn more, on average, than do
Kravonians with only one college degree.
B. The percentage of Kravonians who attend college in order to earn higher salaries
is higher now than it was several years ago.
C. The higher average salary for jobs requiring a college degree is not due largely to
a scarcity among the Kravonian workforce of people with a college degree.
D. The average salary in Kravonia for jobs that do not require a college degree will
not increase over the next four years.
E. Few members of the Kravonian workforce earned their degrees in other countries.
Answer:
*******************
Q41:
During the eighteenth century, widespread changes in agriculture, known as the agrarian
revolution, involving the large-scale introduction of enclosed fields, of new farming
techniques and crops, and the substitution of commercial for subsistence farming.
A. revolution, involving the large-scale introduction of enclosed fields, of new
farming techniques and crops, and
B. revolution, involved the large-scale introduction of enclosed fields and of new
farming techniques and crops, and
C. revolution, which involved the large-scale introduction of enclosed fields and of
new farming techniques and crops, and
D. revolution involved the large-scale introduction of enclosed fields, new farming
techniques and crops, and of
E. revolution that involved the large-scale introduction of enclosed fields and new
farming techniques and crops, and of
Answer:
*******************
Answer:
EEC()E,ECCBC,CDCED,BAEDE,ADCEC,()BECD,ECA()(),()BAAC,B__

MATH.
Q1 C. BOTH statements TOGETHER are
what is 35 percent of the sum of 1.4 and sufficient, but NEITHER statement
1/5? ALONE is
A. 0.42 sufficient.
B. 0.56 D. EACH statement ALONE is
C. 0.85 sufficient.
D. 1.55 E. Statements (1) and (2) TOGETHER
E. 1.95 are NOT sufficient.
answer: Answer:
------------------------------------------------- -------------------------------------------------
----------------------------------------- -----------------------------------------
Q2 Q4
If x..y, is x+y=xy? How many seconds will it take for a car
(1) (1-x)(1-y)=1 that is traveling
(2) x^2-y^2=x^2y-xy^2 at a constant rate of 45 miles per hour to
________________________________ travel a
________________________________ distance of 220 yards? (1 mile=1,760
A. Statement (1) ALONE is sufficient, yards)
but statement (2) alone is not sufficient. A. 8
B. Statement (2) ALONE is sufficient, B. 9
but statement (1) alone is not sufficient. C. 10
C. BOTH statements TOGETHER are D. 11
sufficient, but NEITHER statement E. 12
ALONE is answer
sufficient. -------------------------------------------------
D. EACH statement ALONE is -----------------------------------------
sufficient. Q5
E. Statements (1) and (2) TOGETHER If x^3y^4=5,000, is y=5?
are NOT sufficient. (1)y is a positive integer.
Answer: (2)x is an integer.
------------------------------------------------- ________________________________
-------------------------------------------- ____________________________
Q3 A. Statement (1) ALONE is sufficient,
What is the value of the product xy? but statement (2) alone is not sufficient.
(1) x/y=1/5 B. Statement (2) ALONE is sufficient,
(2) x+y=60 but statement (1) alone is not sufficient.
________________________________ C. BOTH statements TOGETHER are
________________________________ sufficient, but NEITHER statement
A. Statement (1) ALONE is sufficient, ALONE is
but statement (2) alone is not sufficient. sufficient.
B. Statement (2) ALONE is sufficient, D. EACH statement ALONE is
but statement (1) alone is not sufficient. sufficient.
E. Statements (1) and (2) TOGETHER
are NOT sufficient.
Answer: (2) The manager distributed a total of
------------------------------------------------- 18 pens, 27 pencils, and 36 pads.
----------------------------------------- ________________________________
Q6 ________________________________
A string of 10 lightbulbs is wired in such A. Statement (1) ALONE is sufficient,
a way that if but statement (2) alone is not sufficient.
any individual lightbulb fails, the entire B. Statement (2) ALONE is sufficient,
string fails. If for but statement (1) alone is not sufficient.
each individual lightbulb the probability C. BOTH statements TOGETHER are
of failing during sufficient, but NEITHER statement
time period T id 0.06, what is the ALONE is
probability that the sufficient.
string of lightbulbs will fail during time D. EACH statement ALONE is
period T? sufficient.
A.0.06 E. Statements (1) and (2) TOGETHER
B.(0.06)^10 are NOT sufficient.
C.1-(0.06)^10 Answer:
D.(0.94)^10 -------------------------------------------------
E.1-(0.94)^10 -----------------------------------------
Answer Q9
------------------------------------------------- In the xy-plane, if line k passes through
----------------------------------------- the points
Q7 (3.-4) and (a,b), where b=4a-16 and a..3,
If n is a positive integer, what is the what is the slope of k?
remainder A. –4
When (7^(4n+3))(6^n) is divided by 10? B. –1/2
F. 1 C. 0
G. 2 D. 2
H. 4 E. 4
I. 6 answer
J. 8 -------------------------------------------------
answer ----------------------------------------
------------------------------------------------- Q10
----------------------------------------- The sides of a square region, measured
Q8 to the
A department manager distributed a nearest centimeter, are 6 centimeters
number of pens, pencils, long. The
and pads among the staff in the least possible value of the actual area of
department, with each staff the
member receiving x pens, y pencils, and square region is
z pads. How many A.36.00 sq cm
staff members were in the department? B.35.00 sq cm
(1) The numbers of pens, pencils, and C.33.75 sq cm
pads D.30.25 sq cm
that each staff member received were in E.25.00 sq cm
the retio 2:3:4, respectively. Answer
------------------------------------------------- (2) x=2y
----------------------------------------- A. Statement (1) ALONE is sufficient,
Q11 but statement (2) alone is not sufficient.
Of the three-digit positive integers that B. Statement (2) ALONE is sufficient,
have no digits equal but statement (1) alone is not sufficient.
to zero, how many have two digits that C. BOTH statements TOGETHER are
are equal to each sufficient, but NEITHER statement
other and the remaining digit different ALONE is
from the other two? sufficient.
A. 24 D. EACH statement ALONE is
B. 36 sufficient.
C. 72 E. Statements (1) and (2) TOGETHER
D. 144 are NOT sufficient.
E. 216 Answer:
answer Answer
------------------------------------------------- -------------------------------------------------
----------------------------------------- ----------------------------------------
Q12 Q14
Is y^2=0? If a certain coin is flipped, the
(1) 3y=0 probability that the coin will
(2) y^3=0 land heads up is 1/2. If the coin is
A. Statement (1) ALONE is sufficient, flipped 5 times, what is
but statement (2) alone is not sufficient. the probability that it will land heads up
B. Statement (2) ALONE is sufficient, on the first 3 flips
but statement (1) alone is not sufficient. and not on the last 2 flips?
C. BOTH statements TOGETHER are A. 3/5
sufficient, but NEITHER statement B. 1/2
ALONE is C. 1/5
sufficient. D. 1/8
D. EACH statement ALONE is E. 1/32
sufficient. answer
E. Statements (1) and (2) TOGETHER -------------------------------------------------
are NOT sufficient. ------------------------------------------
Answer: Q15
------------------------------------------------- Company A has a total of n employees,
------------------------------------- where n is an
Q13 odd integer, and no two employees have
If the average (arithmetic mean) of the the same annual
assessed values salary. The annual salaries of the n
of x houses is $212,000 and the average employees are listed
of the assessed in increasing order, and the 16th salary
values of y other houses is $194,000, in the list is the
what is the average median of the annual salaries. If the sum
of the assessed values of the x+y of the annual
houses? salaries of Company A’s employees is
(1) x+y=36 $942,400, what
is the average (arithmetic mean) of the An equal number of desks and
annual salaries bookcases are to be placed
of Company A’s employees? along a library wall that is 15 meters
A.$29,450 long. Each desk is
B.$30,400 2 meters long, and each bookshelf is 1.5
C.$32,500 meters long. If
D.$47,120 the maximum possible number of desks
E.$58,900 and bookcases
Answer are to be placed along the wall, then the
------------------------------------------------- space along the
---------------------------------------- wall that is left over will be how many
Q16 meters long?
If ..x is an integer, what is the value of A.0.5
..x? B.1
(3) 11<x<17 C.1.5
(4) 2<..x<5 D.2
________________________________ E.3
________________________________ Answer
A. Statement (1) ALONE is sufficient, -------------------------------------------------
but statement (2) alone is not sufficient. -----------------------------------------
B. Statement (2) ALONE is sufficient, Q19
but statement (1) alone is not sufficient. Square ABCD is inscribed in circle O.
C. BOTH statements TOGETHER are What is
sufficient, but NEITHER statement the area of square region ABCD?
ALONE is (1) The area of circular
sufficient. region O is 64...
D. EACH statement ALONE is (2) The circumference of
sufficient. circle O is 16 ...
E. Statements (1) and (2) TOGETHER ________________________________
are NOT sufficient. ____________________________-
Answer: A. Statement (1) ALONE is sufficient,
------------------------------------------------- but statement (2) alone is not sufficient.
---------------------------------------- B. Statement (2) ALONE is sufficient,
Q17 but statement (1) alone is not sufficient.
If P(r)=8r/(1-r), for what value of r does C. BOTH statements TOGETHER are
P(r)=1/2(P(3))? sufficient, but NEITHER statement
K. 6 ALONE is
L. 3 sufficient.
M. 0 D. EACH statement ALONE is
N. –3 sufficient.
O. –6 E. Statements (1) and (2) TOGETHER
answer are NOT sufficient.
------------------------------------------------- Answer
----------------------------------------- -------------------------------------------------
Q18 -----------------------------------------
Q20.
Based on this year’s costs, an orcharde E. 1
grower budgets P dollars for planting N answer
new trees next year. -------------------------------------------------
If the average cost of planting each tree -----------------------------------------
were to increase 25 percent from this Q23.
year’s cost ,then the At a fruit stand yesterday, the price of
greatest number of trees that the orchard each apple was $0.10 more than the
grower could plant next year using P price of each orange.
dollars would be. What was the total revenue from the sale
A. 20% less than N of oranges at the fruit stand yesterday?
B. 25%less than N (1) The number of oranges sold at the
C. equal to N fruit stand
D. 20%greater than N yesterday was 5 more than the number
E. 25% greater than N of
answer apples.
------------------------------------------------- (2) The total revenue from the sale of
---------------------------------------- apples
Q21. at the fruit stand yesterday was $15.00
0 1/4 1/2 Statement (1) ALONE is sufficient, but
3/4 1 statement (2) alone is not sufficient
Time(hours) . Statement (2) ALONE is sufficient, but
Reid took 1 hour to cycle from his home statement (1) alone is not sufficient
to his college. 24
The graph shows Reed’s distance from 18
his college during 12
That hour. Reid’s average speed for the 0
last 15 minutes 6
Of the hour was how many kifometers Dislance
per hour greater kilometers
Than his average speed for the first 45 . BOTH statements TOGETHER are
minutes? sufficient, but NEITHER statement
A .3 ALONE
B. 4 is sufficient.
C. 5 . EACH statement ALONE is sufficient
D. 6 . Statements (1) and (2) TOGETHER are
E.7 NOT sufficient
answer Answer
------------------------------------------------- -------------------------------------------------
----------------------------------------- -----------------------------------------
Q22. Q24.
which of the following is a value of X if R. S. and T are numbers on the
for which X^11-X^3>0 number line, not necessarity
A. –2 in that order, what is the distance
B. –1 between R and T?
C. –1/2 (1) The distance between R and S is 50.
D. 1/2 (2) The distance between S and T is 41
________________________________ is sufficient.
________________________________ . EACH statement ALONE is sufficient
Statement (1) ALONE is sufficient, but . Statements (1) and (2) TOGETHER are
statement (2) alone is not sufficient NOT sufficient
. Statement (2) ALONE is sufficient, but answer
statement (1) alone is not sufficient -------------------------------------------------
. BOTH statements TOGETHER are -----------------------------------------
sufficient, but NEITHER statement Q27.
ALONE If M is a positive integer, then M^3 has
is sufficient. how many digits?
. EACH statement ALONE is sufficient (1) M has 3 digits.
. Statements (1) and (2) TOGETHER are (2) M^2 has 5 digits
NOT sufficient ________________________________
Answer _______________________________-
------------------------------------------------- Statement (1) ALONE is sufficient, but
----------------------------------------- statement (2) alone is not sufficient
Q25. . Statement (2) ALONE is sufficient, but
A farmer produced 750 bushels of a statement (1) alone is not sufficient
certain crop at a cost . BOTH statements TOGETHER are
of $20 per bushel. If the farmer sold 2/3 sufficient, but NEITHER statement
of the bushels for ALONE
double their production cost and sold the is sufficient.
remaining bushels . EACH statement ALONE is sufficient
at 25 percent above their production . Statements (1) and (2) TOGETHER are
cost, what was the NOT sufficient
farmer’s gross profit on the sale of the Answer
crop? -------------------------------------------------
A. $11250 -----------------------------------------
B. $13375 Q28.
C. $15000 if the function F is defined by
D. $18750 F(x)=x^2+1/x^2 for all nonzero numbers
E. $26250 X, then F(-1/X)=
answer A. –1/f(x)
------------------------------------------------- B. 1/f(x)
----------------------------------------- C. 1-f(x)
Q26. D. –f(x)
IS (X+Y)^3 an even integer? E. f(x)
(1)X and Y are integers. answer
(2)XY=9 -------------------------------------------------
Statement (1) ALONE is sufficient, but -----------------------------------------
statement (2) alone is not sufficient Q29.
. Statement (2) ALONE is sufficient, but if X>1 and Y>1, is X<Y?
statement (1) alone is not sufficient (1) X^2/(XY+X)<1
. BOTH statements TOGETHER are (2) XY/Y^2-Y<1
sufficient, but NEITHER statement Statement (1) ALONE is sufficient, but
ALONE statement (2) alone is not sufficient
. Statement (2) ALONE is sufficient, but Last year in a group of 30 businesses, 21
statement (1) alone is not sufficient reported a net profit and
. BOTH statements TOGETHER are 15 had investments in foreign markets.
sufficient, but NEITHER statement How many of the businesses
ALONE did not report a net profit nor invest in
is sufficient. foreign markets last year?
. EACH statement ALONE is sufficient (1) last year 12 of the 30 businesses
. Statements (1) and (2) TOGETHER are reported a net profit and had investments
NOT sufficient in foreign markets.
answer (2) last year 24 of the 30 businesses
------------------------------------------------- reported a net profit or invested in
----------------------------------------- foreign markets, or both.
Q30. Statement (1) ALONE is sufficient, but
Yesterday it took a certain plane 3 hours statement (2) alone is not sufficient
to fly from City A to . Statement (2) ALONE is sufficient, but
City B at an average speed of 400 miles statement (1) alone is not sufficient
per hour. Today the . BOTH statements TOGETHER are
same plane flew from City A to City B sufficient, but NEITHER statement
along the same route at ALONE
an average speed of 450 miles per hour. is sufficient.
How many more . EACH statement ALONE is sufficient
yesterday than it took today? . Statements (1) and (2) TOGETHER are
A. 10 NOT sufficient
B. 15 Answer
C. 20 -------------------------------------------------
D. 25 -----------------------------------------
E. 30 Q33.
answer E
------------------------------------------------- A pump started filling an empty pool
----------------------------------------- with water and continued at
Q31. a constant rate until the pool was full. At
BC noon the pool was 1/3 full,
AD and 1+1/4 hours later it was 3/4 full.
In the figure, each side of square ABCD What was the total number of
has length 1, the length of line hours that it took the pump to fill the
Segment CE is 1, and the length of line pool?
segment BE is equal to the length A. 2+1/3 B. 2+2/3 C. 3
Of line segment DE. What is the area of D. 3+1/2 E. 3+2/3
the triangular region BCE? Answer
a. 1/3 b. (2^-2 )/4 c. 1/2 -------------------------------------------------
d. (2^-2)/2 e. 3/4 -----------------------------------------
answer Q34.
------------------------------------------------- If M+4 =2, what is the value of M?
----------------------------------------- (1) M<0 (2) M^2+8M+12=0
Q32. Statement (1) ALONE is sufficient, but
statement (2) alone is not sufficient
. Statement (2) ALONE is sufficient, but Q36.
statement (1) alone is not sufficient In a certain quiz that consists of 10
. BOTH statements TOGETHER are questions, each
sufficient, but NEITHER statement question after the first is worth 4 points
ALONE more than
is sufficient. the preceding question. If the 10
. EACH statement ALONE is sufficient questions on the
. Statements (1) and (2) TOGETHER are quiz are worth a total of 360 points, how
NOT sufficient many points
answer is the third question worth?
------------------------------------------------- A. 18
----------------------------------------- B. 24
Q35. C. 26
Linda purchased 3 books at a book fair. D. 32
What was the median price of the 3 E. 44
books? Answer
(1) The average (arithmetic mean) price -------------------------------------------------
of the 3 books was $1.5 -----------------------------------------
(2) The price of one of the 3 books was Q37.
$1.5 AB
________________________________ +BA
________________________________ AAC
Statement (1) ALONE is sufficient, but In the correctly worked addition problem
statement (2) alone is not sufficient shown, where
. Statement (2) ALONE is sufficient, but the sum of the two-digit positive integers
statement (1) alone is not sufficient AB and BA is
. BOTH statements TOGETHER are the three-digit integer AAC, and A, B,
sufficient, but NEITHER statement and C are different digits, what is the
ALONE units digit of the
is sufficient. integer AAC?
. EACH statement ALONE is sufficient A. 9
. Statements (1) and (2) TOGETHER are B. 6
NOT sufficient C. 3
answer D. 2
------------------------------------------------- E. 0
----------------------------------------- Answer
------------------------------------------------------------------------------------------
Answer:
BDCCC,EEEEC,EDCEB,ADBDA,CCCEA,CEEEC,DBCEC,CE

MATH.
1 D. 1
If 25(5^x)=1 then x= E. 2
A. –2 Answer:
B. –1
C. 0
------------------------------------------------- BOTH statements TOGETHER are
------------------------------------------------- sufficient, but NEITHER statement
----- ALONE
2 is sufficient.
if p, s, and t are positive prime numbers, EACH statement ALONE is sufficient
what is the value of p^3s^3t^3? Statements (1) and (2) TOGETHER
(1) p^3st=728 are NOT sufficient
(2) t=13 Answer
Statement (1) ALONE is sufficient, -------------------------------------------------
but statement (2) alone is not sufficient -------------------------------------------------
Statement (2) ALONE is sufficient, -----
but statement (1) alone is not sufficient 4
BOTH statements TOGETHER are A shipment of 8 television sets contains
sufficient, but NEITHER statement 2 black-and-white sets and 6color sets. If
ALONE 2
is sufficient. television sets are to be chosen at
EACH statement ALONE is sufficient random from this shipment, what is the
Statements (1) and (2) TOGETHER probability
are NOT sufficient that at least 1 of the 2 sets chosen will be
Answer a black-and-white set?
------------------------------------------------- A. 1/7 B. ¼ C. 5/14 D. 11/28 E. 13/26
------------------------------------------------- Answer:
----- -------------------------------------------------
3 -------------------------------------------------
in a certain game played with red chips -----
and blue chips, each red chip has a point 5
value The tens digit of positive integer X is 3.
of X and each blue chip has a point Is the units digit of X greater than 4?
value of Y, where X>Y and X and Y are (1) The units digit of 2X is less than the
positive units digit of X.
integers. If a player has 5 red chips and 3 (2) The tens digit of 2X is 7.
blue chips, what is the average -------------------------------------------------
(arithmetic -------------------------------------------------
mean ) point value of the 8 chips that the -----
player has? ---
(1) The average point value of one red Statement (1) ALONE is sufficient,
chip and one blue chip is 5. but statement (2) alone is not sufficient
(2) The average point value of the 8 Statement (2) ALONE is sufficient,
chips that the player has is an integer. but statement (1) alone is not sufficient
------------------------------------------------- BOTH statements TOGETHER are
------------------------------------------------- sufficient, but NEITHER statement
--- ALONE
Statement (1) ALONE is sufficient, is sufficient.
but statement (2) alone is not sufficient EACH statement ALONE is sufficient
Statement (2) ALONE is sufficient, Statements (1) and (2) TOGETHER
but statement (1) alone is not sufficient are NOT sufficient
Answer The operation* represents either
------------------------------------------------- addition, subtraction, or multiplication of
------------------------------------------------- integers,
----- what is the value of 1*0?
6 (1) 0*2=2
|-4| (|-20|-|5|)= (2) 2*0=2
A. –100 B. –60 C. 60 D. 75 E. 100 Statement (1) ALONE is sufficient,
Answer but statement (2) alone is not sufficient
------------------------------------------------- Statement (2) ALONE is sufficient,
------------------------------------------------- but statement (1) alone is not sufficient
----- BOTH statements TOGETHER are
7 sufficient, but NEITHER statement
What is the maximum integer value of ALONE
M for which 12!/2^M is an integer? is sufficient.
A. 2 B. 4 C. 6 D. 10 E. 12 EACH statement ALONE is sufficient
Answer Statements (1) and (2) TOGETHER
------------------------------------------------- are NOT sufficient
------------------------------------------------- Answer
----- -------------------------------------------------
8 -------------------------------------------------
Can a certain rectangular sheet of glass -----
be positioned on a rectangular tabletop 10
so that If each of the 12 teams participating in a
it covers the entire tabletop and its edges certain tournament plays exactly one
are parallel to the edges of the tabletop? game
(1) The tabletop is 36 inches wide by 60 with each of the other teams, how many
inches long. games will be played?
(2) The area of one side of the sheet of A. 144 B. 132 C. 66 D. 33 E. 23
glass is 2400 square inches. Answer
Statement (1) ALONE is sufficient, -------------------------------------------------
but statement (2) alone is not sufficient -------------------------------------------------
Statement (2) ALONE is sufficient, -----
but statement (1) alone is not sufficient 11
BOTH statements TOGETHER are An equilateral triangle that has an area
sufficient, but NEITHER statement of 9 3^1/2 is inscribed in a circle. What
ALONE is the
is sufficient. area of the circle?
EACH statement ALONE is sufficient A . 6pi B. 9pi C. 12 pi D. 9pi 3^1/2 E.
Statements (1) and (2) TOGETHER 18pi 3^1/2
are NOT sufficient Answer
Answer -------------------------------------------------
------------------------------------------------- -------------------------------------------------
------------------------------------------------- -----
----- 12
9 During a 10-week summer vacation, was
the average (arithmetic mean) number of
books that Carolyn read per week greater -------------------------------------------------
than the average number of books that -------------------------------------------------
Jacob -----
read per week? 14
(1) Twice the average number of books Of the families in City X in 1994, 40
that Carolyn read per week was greater percent owned a personal computer. The
than 5 less than twice the average number
number of books that Jacob read per of families in City X owning a computer
week. in 1998 was 30 percent greater than it
(2) During the last 5 weeks of the was in
vacation, Carolyn read a total of 3 books 1994, and the total number of families in
more City X was 4 percent greater in 1998
than Jacob. than it
Statement (1) ALONE is sufficient, was in 1994. what percent of the families
but statement (2) alone is not sufficient in City X owned a personal computer in
Statement (2) ALONE is sufficient, 1998?
but statement (1) alone is not sufficient A. 50%
BOTH statements TOGETHER are B. 52%
sufficient, but NEITHER statement C. 56%
ALONE D. 70%
is sufficient. E. 74%
EACH statement ALONE is sufficient answer
Statements (1) and (2) TOGETHER -------------------------------------------------
are NOT sufficient -------------------------------------------------
Answer -----
------------------------------------------------- 15
------------------------------------------------- Which of the following expressions can
----- be written as an integer?
13 (1) (82^1/2+82^1/2)^2
If X, Y, and Z are positive integers, is (2) (82)(82^1/2)
Y>X? (3) (82^1/2)(82^1/2)
(1) Y^2=XZ 82
(2) Z-X>0 A. None
Statement (1) ALONE is sufficient, B. (1) only
but statement (2) alone is not sufficient C. (3) only
Statement (2) ALONE is sufficient, D. (1)and(2)
but statement (1) alone is not sufficient E. (1)and (3)
BOTH statements TOGETHER are answer
sufficient, but NEITHER statement -------------------------------------------------
ALONE -------------------------------------------------
is sufficient. -----
EACH statement ALONE is sufficient 16
Statements (1) and (2) TOGETHER From May 1 to May 30 in the same year,
are NOT sufficient the balance in a checking account
Answer increased.
What was the balance in the checking percent, what was the annual interest
account on May 30? rate last year?
(1) If, during this period of time, the A. 1% B. 1.1% C. 9.1% D. 10% E.
increase in the balance in the checking 10.8%
account had been 12 percent, then the Answer
balance in the account on May 30 would -------------------------------------------------
have been $504 -------------------------------------------------
(2) During this period of time, the -----
increase in the balance in the checking 19
account Color X ink is created by blending red,
was 8 percent. blue, green, and yellow inks in the ratio
Statement (1) ALONE is sufficient, 6:5:2:2. what is the number of liters of
but statement (2) alone is not sufficient green ink that was used to create a
Statement (2) ALONE is sufficient, certain
but statement (1) alone is not sufficient batch of color X ink?
BOTH statements TOGETHER are (1) The amount of red ink used to create
sufficient, but NEITHER statement the batch is 2 liters more than the
ALONE amount
is sufficient. of blue ink used to create the batch.
EACH statement ALONE is sufficient (2) The batch consists of 30 liters of
Statements (1) and (2) TOGETHER color X ink.
are NOT sufficient Statement (1) ALONE is sufficient,
Answer but statement (2) alone is not sufficient
------------------------------------------------- Statement (2) ALONE is sufficient,
------------------------------------------------- but statement (1) alone is not sufficient
----- BOTH statements TOGETHER are
17 sufficient, but NEITHER statement
If N is positive, which of the following ALONE
is equal to is sufficient.
1 EACH statement ALONE is sufficient
(n+1)^1/2-n^1/2 Statements (1) and (2) TOGETHER
A. 1 B. (2n+1)^1/2 C. (n+1)^1/2 are NOT sufficient
n^1/2 Answer
D.(n+1)^1/2-n^1/2 -------------------------------------------------
E. (n+1)^1/2+n^1/2 -------------------------------------------------
Answer -----
------------------------------------------------- 20
------------------------------------------------- Of the 32 students in a certain class, 15
----- are in a music club and 20 are in a dance
18 club.
The annual interest rate earned by an If 10 of the students are not in either
investment increased by 10 percent from club, how many of the students are in
last only one of
year to this year. If the annual interest the two clubs?
rate earned by the investment this year A. 5 B. 9 C.10 D. 17 E. 25
was 1 Answer
------------------------------------------------- C. A^4+B^4+C^2 D.
------------------------------------------------- A^4+B^4+2A^2B^2+C^2
----- Answer
21 -------------------------------------------------
If P and R are positive, is 25 percent of P -------------------------------------------------
equal to 10 percent of R? -----
(1) R is 150 percent greater than P 24
(2) P is 60 percent less than R. Each week John earns X dollars per hour
Statement (1) ALONE is sufficient, for the first 40 hours that he works that
but statement (2) alone is not sufficient week and Y dollars for each additional
Statement (2) ALONE is sufficient, hour. How much does John earn per hour
but statement (1) alone is not sufficient for
BOTH statements TOGETHER are the first 40 hours?
sufficient, but NEITHER statement (1) Y=15X
ALONE (2) If John works 45 hours in a week, he
is sufficient. earns a total of $570 that week.
EACH statement ALONE is sufficient Statement (1) ALONE is sufficient,
Statements (1) and (2) TOGETHER but statement (2) alone is not sufficient
are NOT sufficient Statement (2) ALONE is sufficient,
Answer but statement (1) alone is not sufficient
------------------------------------------------- BOTH statements TOGETHER are
------------------------------------------------- sufficient, but NEITHER statement
----- ALONE
22 is sufficient.
If X^2+Y^2=1, is X+Y=1? EACH statement ALONE is sufficient
(1) XY=0 Statements (1) and (2) TOGETHER
(2) Y=0 are NOT sufficient
Statement (1) ALONE is sufficient, Answer
but statement (2) alone is not sufficient -------------------------------------------------
Statement (2) ALONE is sufficient, -------------------------------------------------
but statement (1) alone is not sufficient -----
BOTH statements TOGETHER are 25
sufficient, but NEITHER statement Machine A, operating alone at its
ALONE constant rate, produces 500 feet of a
is sufficient. particular fiber
EACH statement ALONE is sufficient in 2 hours. Machine B, operating alone
Statements (1) and (2) TOGETHER at its constant rate ,produces 500 feet of
are NOT sufficient the
Answer same fiber in 3 hours. Machine C,
------------------------------------------------- operating alone at its constant rate.
------------------------------------------------- Produces 500
----- feet of the same fiber in 5 hours. How
23 many hours will it take machines A. B,
If A@B=A^2+B^2 for all real numbers and C,
A and B. then (A@B)@C=
A. A^2+B^2+C^2 B. 2A^2+2B^2+C^2
operating together at their respective Answer
constant rates, to produce 1000 feet of -------------------------------------------------
the fiber? -------------------------------------------------
A. 60/31 B. 15/7 C. 25/11 D. 10/2 E. -----
20/3 29
Answer A total of 22 men and 26 women were at
------------------------------------------------- a party, and the average (arithmetic
------------------------------------------------- mean)
----- age of all of the adults at the party was
26 exactly 35 years. If the average age of
Does the decimal equivalent of P/Q, the men
where P and Q are positive integers, was exactly 38 years, which of the
contain only following was closest to the average age,
a finite number of nonzero digits? in years,
(1) P>Q of the women?
(2) Q=8 A. 31 B. 31.5 C. 32 D. 32.5 E. 33
Statement (1) ALONE is sufficient, Answer
but statement (2) alone is not sufficient -------------------------------------------------
Statement (2) ALONE is sufficient, -------------------------------------------------
but statement (1) alone is not sufficient -----
BOTH statements TOGETHER are 30
sufficient, but NEITHER statement A parent established a college fund for
ALONE his daughter. Each year the parent made
is sufficient. a
EACH statement ALONE is sufficient contribution to the fund, and each year
Statements (1) and (2) TOGETHER he increased his contribution by a
are NOT sufficient constant
Answer amount. If he made a contribution of
------------------------------------------------- $800 in the first year, by what amount
------------------------------------------------- did the
----- parent increase his contribution to the
27 fund each year?
(1/2)^(-2) (1) The parent’s contribution to the fund
(2)^(-2) in the 18th year was $7600.
A. 16 B. 8 C. 4 D. 1/8 E. 1/16 (2) The parent’s contribution to the fund
Answer in the 7th year was twice what it was in
------------------------------------------------- the 3rd year.
------------------------------------------------- Statement (1) ALONE is sufficient,
----- but statement (2) alone is not sufficient
28 Statement (2) ALONE is sufficient,
If k=-1, which of the following is (are) but statement (1) alone is not sufficient
true? BOTH statements TOGETHER are
1. k^k=k 2. | k| =-k. 3.k^0=-k sufficient, but NEITHER statement
A. 1 only B. 1 and 2 only C. 1 and 3 ALONE
only is sufficient.
D. 2 and 3 only E. 1. 2. and 3. EACH statement ALONE is sufficient
Statements (1) and (2) TOGETHER per hour and 45 miles per hour. John
are NOT sufficient traveled from City A to City B along the
Answer same
------------------------------------------------- route in 2 hours, and his speed was
------------------------------------------------- between 45 miles per hour and 60 miles
----- per hour.
31 Which of the following could be the
Each side of a certain parallelogram has distance, in miles, from City A to City
length 6. if the area of the parallelogram B?
is 18. A. 95 B. 115 C. 125 D. 160 E. 180
which of the following is the measure of Answer
one of its angles? -------------------------------------------------
A. 30’ B. 45’ C. 60’ D. 90’ E.120’ -------------------------------------------------
Answer -----
------------------------------------------------- 34
------------------------------------------------- A certain carton holds fewer than 50
----- books. What is the number of books in
32 the
If ¼ of the larger of two numbers is carton?
greater than 5 times the smaller of the (1) The books in the carton can be
same two divided into 3 stacks of X books each,
numbers, is the smaller number less than with 2
4? books left over.
(1) The larger number is greater than 70. (2) The books in the carton can be
(2) The larger number is less than 80. divided into Y stacks of 7 books each,
Statement (1) ALONE is sufficient, with 2
but statement (2) alone is not sufficient books left over.
Statement (2) ALONE is sufficient, Statement (1) ALONE is sufficient,
but statement (1) alone is not sufficient but statement (2) alone is not sufficient
BOTH statements TOGETHER are Statement (2) ALONE is sufficient,
sufficient, but NEITHER statement but statement (1) alone is not sufficient
ALONE BOTH statements TOGETHER are
is sufficient. sufficient, but NEITHER statement
EACH statement ALONE is sufficient ALONE
Statements (1) and (2) TOGETHER is sufficient.
are NOT sufficient EACH statement ALONE is sufficient
Answer Statements (1) and (2) TOGETHER
------------------------------------------------- are NOT sufficient
------------------------------------------------- Answer
----- -------------------------------------------------
33 -------------------------------------------------
Anne traveled from City A to City B in 4 -----
hours, and her speed was between 25 35
miles What is the value of
x^2yz+xy^2z+xyz^2
(1) x+y+z=18
(2) xyz=10 maintained their speeds and both
Statement (1) ALONE is sufficient, remained on the highway for at least 2
but statement (2) alone is not sufficient hours, how
Statement (2) ALONE is sufficient, long after he passed the gas station did
but statement (1) alone is not sufficient Paul catch up with Mary?
BOTH statements TOGETHER are A. 30 min
sufficient, but NEITHER statement B. 45 min
ALONE C. 1 hr
is sufficient. D. 1 hr 15 min
EACH statement ALONE is sufficient E. 1 hr 30 min
Statements (1) and (2) TOGETHER answer
are NOT sufficient -------------------------------------------------
Answer -------------------------------------------------
------------------------------------------------- -----
------------------------------------------------- 37
----- If K is an integer and
36 (0.0025)(0.025)(0.00025)*10^k is an
Mary passed a certain gas station on a integer, what is the least
highway while traveling west at a possible value of K?
constant A. –12
speed of 50 miles per hour. Then, 15 B. –6
minutes later, Paul passed the same gas C. 0
station D. 6
while traveling west at a constant speed E. 12
of 60 miles per hour. If both drivers Answer
-------------------------------------------------------------------------------------------------------
Answer:
AACED,CDEAC,CACAE,AECDB,DEDCA,EAEDD,ABBEC,DE

12
Verbal Section
*******************
Q1:
The Acoma and Hopi are probably the two oldest surviving Pueblo
communities, both
dating back at least a thousand years.
A. both dating
B. both of which have dated
C. and each has dated
D. and each one dating
E. each one of which date
Answer:
*******************
Q2:
Neither First Lady Eleanor Roosevelt nor Secretary of Labor Frances
Perkins sought
recognition by the press, and both cautiously allowed others of the
Roosevelt brain trust
to take credit for the genesis of historic programs in public
employment, relief, and social
security that were in large measure what they had been responsible
for.
A. to take credit for the genesis of historic programs in public
employment, relief,
and social security that were in large measure what they had been
responsible for
B. to take credit for the genesis of historic programs in public
employment, relief,
and social security for which the two women were in large measure
responsible
C. taking credit for the genesis of historic programs in public
employment, relief,
and social security for which the two women were in large measure
responsible
D. taking credit for the genesis of historic programs in public
employment, relief,
and social security that were in large measure what they were
responsible for
E. taking credit for the genesis of historic programs in public
employment, relief,
and social security which were largely their responsibility
Answer:
*******************
Q3 to Q6:
Extensive research has shown brand merely because the price
that the effects of short-term is
price 13
promotions on sales are reduced. They simply avoid
themselves paying
Line short-term. Companies’ more than they have to when
hopes that one of
(5) promotions might have a (15) their customary brands is
positive temporarily
aftereffect have not been borne available at a reduced price. A
out for reasons that researchers price promotion does not
have been able to identify. A increase
price the number of long-term
promotion entices only a customers
brand’s of a brand, as it attracts virtually
(10) long-term or “loyal” (20) no new customers in the
customers; first place.
people seldom buy an unfamiliar
Nor do price promotions have (30) be more of them. And the
lingering bigger
aftereffects for a brand, even the increase in sales at
negative ones such as damage promotion
to prices, the bigger the loss. While
a brand’s reputation or erosion short-term price promotions can
of have legitimate uses, such as
(25) customer loyalty, as is often (35) reducing excess inventory, it
feared. is the
So why do companies spend so recognizable increase in sales
much on price promotions? that
Clearly is their main attraction to
price promotions are generally management,
run which is therefore reluctant
at a loss, otherwise there would to abandon this strategy despite
its effect on the bottom line.
--------------------------------------------------------------------------------
Q3:
The primary purpose of the passage is to
A. compare the arguments in favor of a certain strategy with those
against it
B. attack a certain strategy by enumerating its negative consequences
C. justify the use of a certain strategy in light of certain criticisms that
have been
made against it
D. advocate a particular strategy by arguing against an alternative
E. explain the effects of a certain strategy and the primary motivations
for adopting
it
Answer:
--------------------------------------------------------------------------------
Q4:
According to the passage, which of the following is the reason why
short-term price
promotions do not attract new long-term customers to a brand?
A. Short-term price promotions do not produce an increase in sales.
14
B. Customers come to regard the promotional price as the fair price
and the regular
price as excessive.
C. Most customers select among competing products largely on the
basis of price
and very few are loyal to any particular brand.
D. Customers who have not previously bought the promoted brand are
almost never
persuaded to do so by the short-term price promotions.
E. Any customers that a brand gains by means of a short-term price
promotion are
liable to be lost when a competing brand has a similar promotion.
Answer:
--------------------------------------------------------------------------------
Q5:
The passage suggests that evidence for price promotions’ “effect on
the bottom line” (line
40) is provided by
A. the lack of lingering aftereffects from price promotions
B. the frequency with which price promotions occur
C. price promotions’ inability to attract new customers
D. price promotions’ recognizable effect on sales
E. the legitimate uses to which management can put price promotions
Answer:
--------------------------------------------------------------------------------
Q6:
It can be inferred from the passage that if a company ceased to run
short-term price
promotions for a particular product, an effect of this change would be
to
A. reduce excess inventory of the product
B. lose some of the product’s long-term customers
C. reduce the product’s overall sales
D. inhibit growth in the number of the product’s customers
E. threaten the product’s profitability
Answer:
*******************
Q7:
Although the discount stores in Goreville’s central shopping district are
expected to close
within five years as a result of competition from a SpendLess discount
department store
that just opened, those locations will not stay vacant for long. In the
five years since the
opening of Colson’s, a nondiscount department store, a new store has
opened at the
location of every store in the shopping district that closed because it
could not compete
with Colson’s.
Which of the following, if true, most seriously weakens the argument?
A. Many customers of Colson’s are expected to do less shopping there
than they did
before the SpendLess store opened.
B. Increasingly, the stores that have opened in the central shopping
district since
Colson’s opened have been discount stores.
15
C. At present, the central shopping district has as many stores
operating in it as it
ever had.
D. Over the course of the next five years, it is expected that Goreville’s
population
will grow at a faster rate than it has for the past several decades.
E. Many stores in the central shopping district sell types of
merchandise that are not
available at either SpendLess or Colson’s.
Answer:
*******************
Q8 to Q10:
The idea that equipping homes but ownership of these
with electrical appliances and appliances did
other not correlate with less time
“modern” household spent on
technologies (20) housework by full-time
Line would eliminate drudgery, home workers.
save labor In fact, time spent by these
(5) time, and increase leisure for workers
women remained remarkably
who were full-time home constant―at
workers about 52 to 54 hours per
remained largely unchallenged week―from
until the 1920’s to the 1960’s, a
the women’s movement of the period
1970’s (25) of significant change in
spawned the groundbreaking household
and technology. In surveying two
(10) influential works of centuries of household
sociologist Joann technology
Vanek and historian Ruth in the United States, Cowan
Cowan. argued
Vanek analyzed 40 years of that the “industrialization” of
timeuse the home
surveys conducted by home (30) often resulted in more work
economists to argue that for full-time
electrical home workers because the use
(15) appliances and other of
modern household such devices as coal stoves,
technologies reduced the effort water
required to perform specific pumps, and vacuum cleaners
tasks, tended
to reduce the workload of worker’s duties also shifted to
married- include
(35) women’s helpers (husbands, (40) more household
sons, management, child
daughters, and servants) while care, and the post-Second World
promoting a more rigorous War
standard phenomenon of being “Mom’s
16 taxi.”
of housework. The full-time
home
--------------------------------------------------------------------------------
Q8:
According to the passage, which of the following is true about the idea
mentioned in line
1?
A. It has been undermined by data found in time-use surveys
conducted by home
economists.
B. It was based on a definition of housework that was explicitly rejected
by Vanek
and Cowan.
C. It is more valid for the time period studied by Cowan than for the
time period
studied by Vanek.
D. It is based on an underestimation of the time that married women
spent on
housework prior to the industrialization of the household.
E. It inaccurately suggested that new household technologies would
reduce the effort
required to perform housework.
Answer:
--------------------------------------------------------------------------------
Q9:
The passage is primarily concerned with
A. analyzing a debate between two scholars
B. challenging the evidence on which a new theory is based
C. describing how certain scholars’ work countered a prevailing view
D. presenting the research used to support a traditional theory
E. evaluating the methodology used to study a particular issue
Answer:
--------------------------------------------------------------------------------
Q10:
The passage suggests that Vanek and Cowan would agree that
modernizing household
technology did not
A. reduce the workload of servants and other household helpers
B. raise the standard of housework that women who were full-time
home workers set
for themselves
C. decrease the effort required to perform household tasks
D. reduce the time spent on housework by women who were full-time
home workers
E. result in a savings of money used for household maintenance
Answer:
*******************
Q11:
17
Because ethylene dibromide, a chemical used to fumigate grain, was
blamed for the high
rate of nerve damage suffered by people who work in grain-processing
plants, many such
plants switched to other chemical fumigants two years ago. Since then,
however, the
percentage of workers at these plants who were newly diagnosed with
nerve damage has
not dropped significantly. Therefore, either ethylene dibromide was
wrongly blamed or
else the new chemicals also cause nerve damage.
Which of the following is an assumption on which the argument
depends?
A. If the new chemicals cause nerve damage, the nerve damage
caused would be
different from any nerve damage that ethylene dibromide may cause.
B. There are no chemical fumigants that are completely safe for
workers in grainprocessing
plants.
C. If ethylene dibromide causes nerve damage, it does not take two
years or longer
for that damage to become detectable.
D. Workers at grain-processing plants typically continue to work there
even after
being diagnosed with nerve damage.
E. Workers at grain-processing plants that still use ethylene dibromide
continue to
have a high rate of nerve damage.
Answer:
*******************
Q12:
The globalization of financial-services companies has been a boon to
money launders,
because of allowing money placed in a bank in a less regulated
jurisdiction to be
transferred to a branch in a more regulated one.
A. of allowing money placed in a bank in a less regulated jurisdiction to
be
transferred
B. of allowing the transfer of money placed in a bank in a less
regulated jurisdiction
C. it allows that money placed in a bank in a less regulated jurisdiction
is transferred
D. it allows the transfer of money have been placed in a bank in a less
regulated
jurisdiction
E. it allows money placed in a bank in a less regulated jurisdiction to be
transferred
Answer:
*******************
Q13:
In Japan, a government advisory committee called for the breakup of
Nippon Telephone
and Telegraph Company, the largest telephone company in the world,
so it would be two
local phone companies and one long-distance provider.
A. In Japan, a government advisory committee called for the breakup of
Nippon
Telephone and Telegraph Company, the largest telephone company in
the world,
so it would be
B. The breakup of the world’s largest telephone company, Nippon
Telephone and
Telegraph Company, was called for by a government advisory
committee in
Japan, so it would be
18
C. A government advisory committee in Japan called for the breakup of
Nippon
Telephone and Telegraph Company, the world’s largest telephone
company, into
D. The breakup of Nippon Telephone and Telegraph Company, the
world’s largest
telephone company, was called for by a government advisory
committee in Japan,
so it would be
E. Called for by a government advisory committee, the breakup of
Nippon
Telephone and Telegraph Company in Japan, the world’s largest
telephone
company, was to be into
Answer:
*******************
Q14:
Which of the flowing most logically completes the argument?
The attribution of the choral work Lacrimae to the composer Pescard
(1400 – 1474) has
been regarded as tentative, since it was based on a single treatise
from the early 1500’s
that named Pescard as the composer. Recently, several musical
treatises from the late
1500’s have come to light, all of which name Pescard as the composer
of Lacrimae.
Unfortunately, these newly discovered treatises lend no support to the
attribution of
Lacrimae to Pescard, since _______.
A. the treatise from the early 1500’s misidentifies the composers of
some of the
musical works it considers
B. the author of the treatise from the early 1500’s had no very strong
evidence on
which to base the identification of Pescard as the composer of Lacrimae
C. there are works that can conclusively be attributed to Pescard that
are not even
mentioned in the treatise from the early 1500’s
D. the later treatises probably had no source for their attribution other
than the earlier
treatise
E. no known treatises from the 1600’s identify Pescard as the
composer of Lacrimae
Answer:
*******************
Q15:
Business Consultant: Some corporations shun the use of executive titles
because they
fear that the use of titles indicating position in the corporation tends to
inhibit
communication up and down the corporate hierarchy. Since an
executive who uses a title
is treated with more respect by outsiders, however, use of a title can
facilitate an
executive’s dealings with external businesses. Clearly, corporations should
adopt the
compromise of encouraging their executives to use their corporate
titles externally but not
internally, since even if it is widely known that the corporation’s
executives use titles
outside their organization, this knowledge does not by itself inhibit
communication
within the corporation.
In the consultant’s reasoning, the two portions in boldface play which of
the following
roles?
19
A. The first describes a strategy that has been adopted to avoid a
certain problem; the
second presents a drawback to that strategy.
B. The first describes a strategy that has been adopted to avoid a
certain problem; the
second is a consideration raised to call into question the effectiveness
of that
strategy as a means of achieving that goal.
C. The first describes a strategy that has been adopted to avoid a
certain problem; the
second is a consideration the consultant raises in questioning the
significance of
that problem.
D. The first is part of an explanation that the consultant offers for a
certain
phenomenon; the second is that phenomenon.
E. The first describes a policy for which the consultant seeks to provide
a
justification; the second is a consideration the consultant raises as part
of that
justification.
Answer:
*******************
Q16:
In 1713, Alexander Pope began his translation of the Iliad, a work that,
taking him seven
years until completion, and that literary critic Samuel Johnson, Pope’s
contemporary,
pronounced the greatest translation in any language.
A. his translation of the Iliad, a work that, taking him seven years until
completion,
and that literary critic Samuel Johnson, Pope’s contemporary,
pronounced
B. his translation of the Iliad, a work that took him seven years to
complete and that
literary critic Samuel Johnson, Pope’s contemporary, pronounced
C. his translation of the Iliad, a work that had taken seven years to
complete and that
literary critic Samuel Johnson, Pope’s contemporary, pronounced it as
D. translating the Iliad, a work that took seven years until completion
and that
literary critic Samuel Johnson, Pope’s contemporary, pronounced it as
E. translating the Iliad, a work that had taken seven years to complete
and literary
critic Samuel Johnson, Pope’s contemporary, pronounced it
Answer:
*******************
Q17:
The steel industry has changed radically over the last two decades, as
large, integrated
companies such as Bethlehem Steel once conducted operations from
mining at one end of
the process to shipping at the other have greatly downsized, or in
some cases shut down
altogether.
A. as large, integrated companies such as Bethlehem Steel
B. as large, integrated companies, such as Bethlehem Steel, that
C. with large, integrated companies, such as Bethlehem Steel, that
D. while large, integrated companies, such as Bethlehem Steel, that
E. and large, integrated companies such as Bethlehem Steel
Answer:
*******************
Q18:
20
Unlike the many winter greens that have tough leaves and thus require
a lengthy cooking
time, those of escarole need little or no cooking.
A. many winter greens that have tough leaves and thus require a
lengthy cooking
time, those of escarole need
B. many winter greens that have tough leaves and thus require a
lengthy cooking
time, escarole needs
C. tough leaves of many winter greens and that they require for a
lengthy cooking
time, escarole needs
D. tough leaves of many winter greens and the requirement that they
have a lengthy
cooking time, those of escarole need
E. lengthy cooking time required by the many winter greens with tough
leaves,
escarole needs
Answer:
*******************
Q19:
After several years of rapid growth, the healthy care company became
one of the largest
health care providers in the metropolitan area, while it then proved
unable to handle the
increase in business, falling months behind in its payment to doctors
and hospitals.
A. while it then proved unable to handle the increase in business,
falling months
behind in its payment to
B. while it then proved unable to handle the increase in business and
fell months
behind in its payment to
C. but then it proved unable to handle the increase in business, falling
months behind
in its paying
D. but then proving unable to handle the increase in business, falling
months behind
in paying
E. but then proved unable to handle the increase in business, falling
months behind
in paying
Answer:
*******************
Q20:
During the past year, Pro-Tect Insurance Company’s total payouts on
car-theft claims
were larger than the company can afford to sustain. Pro-Tect cannot
reduce the
number of car-theft policies it carries, so cannot protect itself against
continued large
payouts that way. Therefore, Pro-Tect has decided to offer a discount to
holders of
car-theft policies whose cars have antitheft devices. Many policyholders will
respond
to the discount by installing antitheft devices, since the amount of the
discount will
within two years typically more than cover the cost of installation.
Thus, because cars
with antitheft devices are rarely stolen, Pro-Tect’s plan is likely to
reduce its annual
payouts.
In the argument above, the two portions in boldface play which of the
following roles?
21
A. The first rules out a certain strategy for achieving a goal; the second
presents the
strategy that was adopted instead and whose effectiveness the
argument assesses.
B. The first is a judgment made in support of a certain conclusion; the
second is that
conclusion.
C. The first has been used as a consideration to support adopting a
certain strategy
for achieving a goal; the second reports a decision to adopt an
alternative strategy.
D. The first provides evidence in favor of adopting a certain strategy
for achieving a
goal; the second reports a decision to pursue an alternative goal.
E. The first is a consideration offered against adopting a certain
strategy for
achieving a goal; the second is the main conclusion that the argument
is seeking
to establish.
Answer:
*******************
Q21:
Editorial: The roof of Northtown Council’s equipment-storage building
collapsed under
the weight of last week’s heavy snowfall. The building was constructed
recently and met
local building-safety codes in every particular, except that the nails
used for attaching
roof supports to the building’s columns were of a smaller size than the
codes specify for
this purpose. Clearly, this collapse exemplifies how even a single,
apparently
insignificant, departure from safety standards can have severe
consequences.
Which of the following, if true, most strongly supports the editorial’s
argument?
A. The only other buildings whose roofs collapsed from the weight of
the snowfall
were older buildings constructed according to less exacting standards
than those
in the safety codes.
B. Because of the particular location of the equipment-storage building,
the weight
of snow on its roof was greater than the maximum weight allowed for
in the
safety codes.
C. Because the equipment-storage building was not intended for
human occupation,
some safety-code provisions that would have applied to an office
building did not
apply to it.
D. The columns of the building were no stronger than the building-
safety codes
required for such a building.
E. Because the equipment-storage building was where the council kept
snowremoval
equipment, the building was almost completely empty when the roof
collapsed.
Answer:
*******************
Q22:
Almost a decade after New York State passed laws to protect patients
by reducing the
grueling hours worked by medical residents, twelve hospitals have
been investigated by
state medical officials, finding that all twelve consistently break the
laws, many residents
work longer than 24 hours straight, and that more than half the
surgical residents work
more than 95 hours a week.
22
A. twelve hospitals have been investigated by state medical officials,
finding that all
twelve consistently break the laws, many residents work longer than
24 hours
straight, and that more than half the surgical residents work
B. an investigation by state medical officials of twelve hospitals have
found all
twelve consistently breaking the laws, that many residents work longer
than 24
hours straight, with more than half the surgical residents working
C. an investigation of twelve hospitals by state medical officials has
found that all
twelve consistently break the laws, that many residents work longer
than 24 hours
straight, and that more than half the surgical residents work
D. twelve hospitals were investigated by state medical officials who
found all twelve
breaking the laws, with many residents working longer than 24 hours
straight, and
more than half the surgical residents work
E. an investigation by state medical officials has found that, of twelve
hospitals, all
twelve consistently break the laws, that many residents work longer
than 24 hours
straight, with more than half the surgical residents working
Answer:
*******************
Q23 to Q26:
Recently biologists have been periodic behavior, continuing to
interested in a tide-associated bur-
periodic behavior displayed by (25) row on schedule for several
Line the diatom Hantzschia virgata, weeks.
a This indicates that commuter
(5) microscopic golden-brown diatoms,
alga that rather than relying on
inhabits that portion of a environmental
shoreline cues to keep time, possess an
washed by tides (the intertidal inter23
zone). nal pacemaker or biological
Diatoms of this species, clock
sometimes (30) that enables them to
called “commuter” diatoms, anticipate periodic
remain changes in the environment.
(10) burrowed in the sand during A commuter diatom has an
high unusually
tide, and emerge on the sand accurate biological clock, a
surface consequence of the unrelenting
during the daytime low tide. (35) environmental pressures to
Just before the sand is which
inundated by it is subjected; any diatoms that
the rising tide, the diatoms do
burrow not burrow before the tide
(15) again. Some scientists arrives
hypothesize are washed away.
that commuter diatoms know This is not to suggest that the
that it (40) period of this biological clock
is low tide because they sense is
an immutably fixed. Biologists have
environmental change, such as concluded that even though a
an diatom does not rely on the
alteration in temperature or a environment
change to keep time, environmental
(20) in pressure caused by tidal (45) factors—including changes
movement. in the
However, when diatoms are tide’s hydrostatic pressure,
observed under constant salinity,
conditions mechanical agitation, and
in a laboratory, they still display
temperature—can alter the this relation is complicated in
period intertidal
of its biological clock according dwellers such as commuter
to diatoms
(50) changes in the tidal cycle. In by the fact that these organisms
short, are
the relation between an (60) exposed to the solar-day
organism’s cycle as
biological clock and its well as to the tidal cycle, and
environment sometimes
is similar to that between a display both solar-day and
wristwatch tidal periods in a single
and its owner: the owner cannot behavior.
(55) make the watch run faster Commuter diatoms, for
or slower, example,
but can reset the hands. (65) emerge only during those
However, low tides
that occur during the day.
--------------------------------------------------------------------------------
Q23:
The passage suggests which of the following about the accuracy of the
commuter
diatom’s biological clock?
A. The accuracy of the commuter diatom’s biological clock varies
according to
changes in the tidal cycle.
24
B. The unusual accuracy that characterizes the commuter diatom’s
biological clock
is rare among intertidal species.
C. The commuter diatom’s biological clock is likely to be more accurate
than the
biological clock of a species that is subject to less intense
environmental
pressures.
D. The commuter diatom’s biological clock tends to be more accurate
than the
biological clocks of most other species because of the consistency of
the tidal
cycle.
E. The accuracy of the commuter diatom’s biological clock tends to
fluctuate when
the diatom is observed under variable laboratory conditions.
Answer:
--------------------------------------------------------------------------------
Q24:
According to the passage, the periodic behavior displayed by
commuter diatoms under
constant laboratory conditions is characterized by which of the
following?
A. Greater unpredictability than the corresponding behavior under
natural conditions
B. A consistent periodic schedule in the short term
C. No difference over the long term from the corresponding behavior
under natural
conditions
D. Initial variability caused by the constant conditions of the laboratory
E. Greater sensitivity to environmental factors than is the case under
natural
conditions
Answer:
--------------------------------------------------------------------------------
Q25:
According to the passage, each of the following is characteristic of the
tide-associated
periodic behavior of commuter diatoms EXCEPT:
A. It is triggered when the diatoms are inundated by the tide.
B. It is correlated with the rise and fall of the tide.
C. It adjusts to changes in the tidal cycle.
D. It is influenced by the solar-day cycle.
E. It is regulated by an innate time-keeping mechanism.
Answer:
--------------------------------------------------------------------------------
Q26:
The author of the passage compares the relationship between an
organism’s biological
clock and its environment to the relation between a wristwatch and its
owner most
probably in order to
A. point out a fundamental difference between the function of
biological clocks in
organisms and the use of mechanical clocks by humans
B. illustrate the way in which the period of an organism’s biological
clock can be
altered by environmental factors
25
C. suggest that there are important similarities between the biological
clock in
organisms such as the commuter diatom and the biological clock in
humans
D. support an argument regarding the methods used by certain
organisms to
counteract the influence of the environment on their biological clocks
E. question the accuracy of the biological clock in organisms such as
the commuter
diatom
Answer:
*******************
Q27:
In parts of the Caribbean, the manatee, an endangered marine
mammal, has long been
hunted for its meat. Having noted the manatee hunters’ expert
knowledge of manatees’
habits, local conservationists are encouraging the hunters to stop
hunting and instead to
take tourists on boat rides to see manatees. Tourist interest is high, so
the plan has
promise of achieving the twin goals of giving the former hunters a
good income and
helping ensure the manatees’ survival.
Which of the following, if true, raises the most serious doubt about the
plan’s chance of
success?
A. Many tourists who visit these parts of the Caribbean are
uninterested in manatees
and would not be willing to pay what the former manatee hunters
would have to
charge for boat rides to see manatees.
B. Recovery of the species would enable some hunting to continue
without putting
the manatees’ survival in jeopardy again.
C. In areas where manatees have traditionally been hunted for food,
local people
could easily replace the manatee meat in their diets with other foods
obtained
from the sea.
D. There would not be enough former manatee hunters to act as
guides for all the
tourists who want to see manatees.
E. To maintain their current income, manatee hunters who switched to
guiding
tourists would have to use far larger boats and make many more trips
into the
manatees’ fragile habitat than they currently do.
Answer:
*******************
Q28:
Regulations will not allow a pesticide that is toxic to humans to be used
inside houses
unless the pesticide will dissipate completely from the air within eight
hours after its
application. One test that pesticide manufacturers standardly use to
determine how
quickly anti-termite pesticides dissipate involves spraying the
pesticides on the walls of
room-sized plywood boxes and then timing its dissipation.
Which of the following would it be most useful to know in order to
evaluate whether a
dissipation time of just under eight hours on the manufacturers’ test
indicates that an antitermite
pesticide that is toxic to humans obeys regulations for use in houses?
26
A. Whether anti-termite pesticides dissipate more slowly in furnished
rooms than in
plywood boxes
B. Whether people who apply anti-termite pesticide standardly wear
protective
equipment that prevents them from being exposed to the pesticide
C. Whether people whose house is being treated with anti-termite
pesticide generally
know that they should remain out of their house during the hours
immediately
after the pesticide’s application
D. Whether there are anti-termite pesticides that are toxic to humans
that, when
subjected to the manufacturers’ test, dissipate completely from the air
in the boxes
in well under eight hours
E. Whether anti-termite pesticides that are not toxic to humans tend to
take longer to
dissipate than those that are toxic
Answer:
*******************
Q29:
Floating in the waters of the equatorial Pacific, an array of buoys
collects and transmits
data on long-term interactions between the ocean and the
atmosphere, interactions that
affect global climate.
A. atmosphere, interactions that affect
B. atmosphere, with interactions affecting
C. atmosphere that affects
D. atmosphere that is affecting
E. atmosphere as affects
Answer:
*******************
Q30:
Under high pressure and intense heat, graphite, the most stable form
of pure carbon,
changes into the substance commonly referred to as diamond and
remaining this way
whether or not the heat and pressure are removed.
A. remaining this way whether or not
B. remaining like that even as
C. remaining as such whether or not
D. remains in this way although
E. remains thus even when
Answer:
*******************
Q31:
Frobisher, a sixteenth-century English explorer, had soil samples from
Canada’s
Kodlunarn Island examined for gold content. Because high gold content
was reported,
Elizabeth I funded two mining expeditions. Neither expedition found
any gold there.
Modern analysis of the island’s soil indicates a very low gold content.
Thus the methods
used to determine the gold content of Frobisher’s samples must have
been inaccurate.
Which of the following is an assumption on which the argument
depends?
27
A. The gold content of the soil on Kodlunarn Island is much lower today
than it was
in the sixteenth century.
B. The two mining expeditions funded by Elizabeth I did not mine the
same part of
Kodlunarn Island.
C. The methods used to assess gold content of the soil samples
provided by
Frobisher were different from those generally used in the sixteenth
century.
D. Frobisher did not have soil samples from any other Canadian island
examined for
gold content.
E. Gold was not added to the soil samples collected by Frobisher before
the samples
were examined.
Answer:
*******************
Q32:
The first commercially successful drama to depict Black family life
sympathetically and
the first play by a Black woman to be produced on Broadway, it was
Lorraine
Hansberry’s A Raisin in the Sun that won the New York Drama Critics’
Circle Award in
1959, and was later made into both a film and a musical.
A. it was Lorraine Hansberry’s A Raisin in the Sun that won the New York
Drama
Critics’ Circle Award in 1959, and was later made
B. in 1959 A Raisin in the Sun, by Lorraine Hansberry, won the New York
Drama
Critics’ Circle Award and was later made
C. Lorraine Hansberry won the New York Drama Critics’ Circle Award for
A Raisin
in the Sun in 1959, and it was later made
D. Lorraine Hansberry’s A Raisin in the Sun won the New York Drama
Critics’
Circle Award in 1959 and was later made
E. A Raisin in the Sun, by Lorraine Hansberry, won the New York Drama
Critics’
Circle Award in 1959, and later made it
*******************
Q33:
The global-warming effect of ocean white caps are one of the many
aspects of the ocean
environment that are not yet incorporated in any detail into the
computer models used for
predicting how rising greenhouse gas concentrations could affect
climate.
A. The global-warming effect of ocean white caps are one of the many
aspects of the
ocean environment that are not yet incorporated in any detail into the
computer
models used for predicting
B. The effect on global warming of ocean white caps are one of the
many aspects of
the ocean environment not yet incorporated in any detail into
computer models,
which they use to predict
C. The effect of ocean white caps on global warming is one of the many
aspects of
the ocean environment that are not yet incorporated in any detail into
the
computer models used to predict
28
D. That ocean white caps have an effect on global warming is one of
the many
aspects of the ocean environment not yet having been incorporated in
any detail
into the computer models that are used for predicting
E. That ocean white caps have an effect on global warming is one of
the many
aspects of the ocean environment not yet being incorporated in any
detail into
computer models, which they use to predict
Answer:
*******************
Q34:
The market for recycled commodities like aluminum and other metals
remain strong
despite economic changes in the recycling industry.
A. commodities like aluminum and other metals remain
B. commodities like those of aluminum and other metals are remaining
C. commodities such as aluminum and other metals remains
D. commodities, such as aluminum and other metals, remain
E. commodities, like the commodities of aluminum and other metals,
remains
Answer:
******************* wane; by the 1960’s British
Q35 to Q37: enterprise
The experience of British (10) was of little importance in
business the Iranian
in Iran between the 1860’s and economy. While in Japan and
the India the
1970’s is one example of the decline of British business was
changing primarily
Line importance of British a function of the rise of strong
enterprise in Asia indigenous
(5) as a whole. Before 1914 business groups, in Iran, by
British business (15) contrast, the government
established and dominated played a
Iran’s large role in both challenging
modern industrial and financial British
sector; commercial interests and
in the 1920’s this domination stimulating
began to locally owned enterprise.
Periodic
surges of intense Iranian British government, and that the
economic Anglo-
(20) nationalism must be Iranian Oil Company did not
understood partly take its
as a reaction to the close (30) orders from the British
relations government,
between British business in Iran despite the 51 percent
and government
the British government. In shareholding. However, the
retrospect, relationship
it is possible to see the uneasy between British business and
and the British
(25) ambiguous nature of this government was sufficiently
relationship. close that
It is true that the British Imperial (35) many Iranians
Bank understandably viewed
29 the oil company and the bank as
in Iran was never entirely a tool symbols
of the
of a British imperialist policy.
--------------------------------------------------------------------------------
Q35:
The primary purpose of the passage is to
A. evaluate a country’s solution to a problem
B. describe differing perceptions of a historical event
C. contrast historical events in two countries
D. provide an explanation for a historical phenomenon
E. challenge an accepted explanation for a historical change
Answer:
--------------------------------------------------------------------------------
Q36:
The passage suggests which of the following about British business in
Japan and India as
compared to British business in Iran?
A. British business in Japan and India received less support from the
British
government than did British business in Iran.
B. During the early twentieth century, British business played less of a
role in the
Japanese and Indian economy than it did in the Iranian economy.
C. The governments of Japan and India played less of a role in the
changing status of
British business than did the government of Iran.
D. The types of enterprises conducted by the British in Japan and India
were
significantly different from the enterprises conducted by the British in
Iran.
E. British business in Japan and India declined more gradually than did
British
business in Iran.
Answer:
--------------------------------------------------------------------------------
Q37:
The author of the passage mentions the British government’s shares in
the Anglo-Iranian
Oil Company most probably in order to
A. demonstrate the British enterprise in Iran was controlled by the
British
government
B. contrast British-run businesses in Iran with Iranian-run businesses in
Iran
30
C. show how joint British and Iranian enterprises were encouraged by
the British
government
D. illustrate a point about the financial difficulties faced by British
businesses in Asia
E. suggest a reason for Iranians’ perception of the role British
government played in
British business
Answer:
*******************
Q38:
Many residents of Calovia are committed to using products containing
recycled materials.
Soon these consumers will get help in identifying such products from a
book being
published by the Calovian government. The book offers a
comprehensive listing, by
product type and brand, of goods sold in Calovia that contain recycled
material.
Therefore, publication of the book will almost certainly increase the use
of products
containing recycled materials in Calovia.
Which of the following, if true, most strengthens the argument?
A. Proceeds from the sale of the book are not expected to exceed the
cost of its
publication.
B. For numerous types of products, there are many brands that use
recycled
materials, although their manufacturers and distributors do nothing to
advertise
those brands’ recycled content.
C. For many materials, such as plastics, the recycling process results in
a lower grade
of material with a correspondingly different range of uses.
D. For many types of products listed in the book, all the brands
available in Calovia
use recycled materials.
E. Many manufacturers of products that contain recycled materials
vary the
proportion of recycled materials in those products in response to
changes in price
and availability.
Answer:
*******************
Q39.
Unearthed in China, fossils of feathered dinosaurs offer the most
dramatic evidence yet
discovered of the close evolutionary relationship between dinosaurs
and birds.
A. offer the most dramatic evidence yet discovered of the close
evolutionary
relationship between dinosaurs and birds
B. offer evidence more dramatic than what has yet been discovered of
the close
evolutionary relationship between dinosaurs and birds
C. offer more dramatic evidence of the close evolutionary relationship
than any yet
discovered between dinosaurs and birds
D. have offered the most dramatic evidence of the close evolutionary
relationship
between dinosaurs and birds that have yet been discovered
E. have offered more dramatic evidence than any that has yet been
discovered of the
close evolutionary relationship between dinosaurs and birds
Answer:
31
*******************
Q40.
Many people suffer an allergic reaction to certain sulfites, including
those that
are commonly added to wine as preservatives. However, since there
are several
wine makers who add sulfites to none of the wines they produce,
people who
would like to drink wine but are allergic to sulfites can drink wines
produced by
these wine makers without risking an allergic reaction to sulfites.
Which of the following is an assumption on which the argument
depends?
A. These wine makers have been able to duplicate the preservative
effect
produced by adding sulfites by means that do not involve adding any
potentially allergenic substances to their wine.
B. Not all forms of sulfite are equally likely to produce the allergic
reactions.
C. Wine is the only beverage to which sulfites are commonly added.
D. Apart from sulfites, there are no substances commonly present in
wine that
give rise to an allergic reaction.
E. Sulfites are not naturally present in the wines produced by these
wine
makers in amounts large enough to produce an allergic reaction in
someone
who drinks these wines.
Answer:
*******************
Q41.
In ancient Thailand, much of the local artisans’ creative energy was
expended for the
creation of Buddha images and when they constructed and decorated
the temples that
enshrined them.
A. much of the local artisans’ creative energy was expended for the
creation of
Buddha images and when they constructed and decorated the temples
that
enshrined them
B. much of the local artisans’ creative energy was expended on the
creation of
Buddha images and on construction and decoration of the temples in
which they
were enshrined
C. much of the local artisans’ creative energy was expended on the
creation of
Buddha images as well as constructing and decoration of the temples
in which
they were enshrined
D. creating images of Buddha accounted for much of the local artisans’
creative
energy, and also constructing and decorating the temples enshrining
them
E. the creating of Buddha images accounted for much of the local
artisans’ creative
energy as well as construction and decoration of the temples that
enshrined them
Answer:
Answers:
ABEDB,CBACD, CECDA, BBBEA,ACCBA,BEAAD, EDCED,CEBAE,B
32
AWA
AA
Read the statement and the instructions that follow it,and then make
any notes that will
help you plan your response.Begin typing your response in the box at
the bottom of the
screen.
The following appeared in an article in a college departmental
newsletter.
"Professor Taylor of Jones University is promoting a model of foreign
language
instruction in which students receive ten weeks of intensive training,
then go abroad to
live with families for ten weeks. The superiority of the model, Professor
Taylor contends,
is proved by the results of a study in which foreign language tests
given to students at 25
other colleges show that first-year foreign language students at Jones
speak more fluently
after only ten to twenty weeks in the program than do nine out of ten
foreign language
majors elsewhere at the time of their graduation:'
Discuss how well reasoned. . . etc
AI
Read the statement and the instructions that follow it, and then make any notes
that will help you plan
your response. Begin typing your response in the box at the bottom of the
screen.
"All employees should help decide how the profits of their company or business
should be used."
Discuss the extent to which you agree or disagree with the opinion
stated above. Support
your views with reasons and/or examples from your own experience,
observations, or
reading.
VERBAL
Q1.
The United States minted about 857 million silver-colored “Susan B. Anthony”
dollars between 1979 and 1981, but the coin proved unpopular because it looked and
felt too much like a quarter.
A. The United States minted about 857 million silver-colored “Susan B.
Anthony” dollars between 1979 and 1981, but the coin
B. About 857 million silver-colored “Susan B. Anthony” dollars were minted as
coins in the United States between 1979 and 1981 but
C. About 857 million silver-colored “Susan B. Anthony” dollars that were minted
between 1979 and 1981 in the United States
D. About 857 million silver-colored “Susan B. Anthony” dollars that the United
States minted between 1979 and 1981
E. Between 1979 and 1981 the United States minted about 857 million
silver-colored “Susan B. Anthony” dollars, which
Answer:
------------------------------------------------------------------------------------------------------
Q2:
Electronic computer chips made of tiny silicon wafers now regularly contain millions
of electronic switches. Unfortunately, electronic switches that are this small cannot
withstand intense radiation. Micro-Mechanics plans to produce a chip that, because
it uses only microscopic mechanical switches, will be invulnerable to radiation
damage. The switches will, however, be slower than electronic switches and the
chip will contain only 12,000 switches.
For there to be a market for Micro-Mechanics’ chip as a result of the apparent
advantage described above, each of the following would have to be true EXCEPT:
A. There will be applications in which the speed attainable by an electronic
switch is not essential.
B. Switches used on electronic chips that contain only 12,000 switches are more
vulnerable to radiation damage than the switches on Micro-Mechanics’ chip
will be.
C. There will be applications for computer chips in environments where the chips
may have to survive intense radiation.
D. Some devices in which computer chips will be used will have other
components that will be able to function during or after exposure to radiation.
E. Manufacturers are able to protect electronic computer chips against exposure
to intense radiation, where this protection is necessary.
Answer:
---------------------------------------------------------------------------------------------------
Q 3 to Q6:
Two opposing scenarios, The “arboreal” hypothesis holds
the “arboreal” hypothesis and that bird ancestors began to fly
the “cursorial” hypothesis, have by climbing frees and gliding
Line traditionally been put forward con- down from branches with the
(5) cerning the origins of bird flight. (10) help of incipient feathers: the
height of trees provides a good (45) hypothesis holds that small
starting place for launching flight, dinosaurs ran along the ground
especially through gliding. As and stretched out their arms for
feathers became larger over time, balance as they leaped into the
(15) flapping flight evolved and birds air after insect prey or, perhaps,
finally became fully air-borne. (50) to avoid predators. Even
This hypothesis makes intuitive rudimentary
Sense, but certain aspects are feathers on forelimbs
Troubling. Archaeopteryx (the could have expanded the arm’s
(20) earliest known bird) and its surface area to enhance lift
maniraptoran dinosaur cousins slightly. Larger feathers could
have no obviously arboreal (55) have increased lift incrementally,
adaptations, such as feet fully until sustained flight was gradually
adapted for perching. Perhaps achieved. Of course, a leap
(25) some of them could climb trees, into the air does not provide the
but no convincing analysis has acceleration produced by drop-
demonstrated how Archaeopteryx (60) ping out of a tree; an animal
would have both climbed and would have to run quite fast
flown with its forelimbs, and there to take off. Still, some small
(30) were no plants taller than a few terrestrial animals can achieve
meters in the environments where high speeds. The cursorial
Archaeopteryx fossils have been (65) hypothesis is strengthened by
found. Even if the animals could the fact that the immediate theropod
climb trees, this ability is not dinosaur ancestors of
(35) synonymous with gliding ability. birds were terrestrial, and they
(Many small animals, and even had the traits needed for high
some goats and kangaroos, (70) lift off speeds: they were small,
are capable of climbing trees agile, lightly built, long-legged,
but are not gliders.) Besides, and good runners. And because
(40) Archaeopteryx shows no obvious they were bipedal, their arms
features of gliders, such as were free to evolve flapping flight,
a broad membrane connecting (75) which cannot be said for other
forelimbs and hind limbs. reptiles of their time.
The “cursorial”(running)
-------------------------------------------------------------------------------------------------------
Q 3:
The primary purpose of the passage is to
A. present counterevidence to two hypotheses concerning the origins of bird
flight
B. propose and alternative to two hypotheses concerning the origins of bird flight
correct certain misconceptions about hypotheses concerning the origins of
bird flight
C. (missing)
D. refute a challenge to a hypothesis concerning the origins of bird flight
E. evaluate competing hypotheses concerning the origins of bird flight
Answer:
-------------------------------------------------------------------------------------------------------
Q 4:
The passage presents which of the following facts as evidence that tends to undermine
the arboreal hypothesis?
A. Feathers tend to become larger over time
B. Flapping flight is thought to have evolved gradually over time
C. Many small animals are capable of climbing trees.
D. Plants in Archaeopteryx’s known habitats were relatively small
E. Leaping into the air does not provide as much acceleration as gliding out of a
tree
Answer:
-------------------------------------------------------------------------------------------------------
Q 5:
Which of the following is included in the discussion of the cursorial hypothesis but
not in the discussion of the arboreal hypothesis?
A. A discussion of some of the features of Archaeopteryx
B. A description of the environment known to have been inhabited by bird
ancestors
C. A possible reason why bird ancestors might have been engaging in activities
that eventually evolved into flight
D. A description of the obvious features of animals with gliding ability
E. An estimate of the amount of time it took for bird ancestors to evolve the kind
of flapping flight that allowed them to become completely airborne
Answer:
-------------------------------------------------------------------------------------------------------
Q 6:
The passage suggests which of the following regarding the climbing ability of
Archaeopteryx?
A. Its ability to climb trees was likely hindered by the presence of incipient feathers
on its forelimbs.
B. It was probably better at climbing trees than were its maniraptoran dinosaur
cousins.
C. It had certain physical adaptations that suggest it was skilled at climbing trees.
D. Scientists have recently discovered fossil evidence suggesting it could not climb
trees.
E. Scientists are uncertain whether it was capable of climbing trees
Answer:
------------------------------------------------------------------------------------------------
Q7:
Providing initial evidence that airport are a larger source of pollution than they were
once believed to be, environmentalists in Chicago report that the total amount of
pollutant emitted annually by vehicles at O’Hare International Airport is twice as
much as that which is being emitted annually by all motor vehicles in the Chicago
metropolitan area.
A. as much as that which is being emitted annually by all
B. as much annually as is emitted by the
C. as much compared to what is annually emitted by all
D. that emitted annually by all
E. that emitted annually compared to the
Answer:
*******************
Q8:
Environmental organizations want to preserve the land surrounding the
Wilgrinn Wilderness Area from residential development. They plan to do this by
purchasing that land from the farmers who own it. That plan is ill-conceived: if
the farmers did sell their land, they would sell it to the highest bidder, and developers
would outbid any other bidders. On the other hand, these farmers will never
actually sell any of the land, provided that farming it remains viable. But
farming will not remain viable if the farms are left unmodernized, and most of the
farmers lack the financial resources modernization requires. And that is exactly why
a more sensible preservation strategy would be to assist the farmers to modernize their
farms to the extent needed to maintain viability.
In the argument as a whole, the two boldface proportions play which of the following
roles?
A. The first presents a goal that the argument rejects as ill-conceived; the second
is evidence that is presented as grounds for that rejection.
B. The first presents a goal that the argument concludes cannot be attained; the
second is a reason offered in support of that conclusion.
C. The first presents a goal that the argument concludes can be attained; the
second is a judgment disputing that conclusion.
D. The first presents a goal, strategies for achieving which are being evaluated in
the argument; the second is a judgment providing a basis for the argument’s
advocacy of a particular strategy.
E. The first presents a goal that the argument endorses; the second presents a
situation that the argument contends must be changed if that goal is to be met
in the foreseeable future.
Answer:
*******************
Q9:
In order to raise revenue, the federal government planned a tax amnesty program that
allows tax delinquents to pay all owed tax without added financial penalty. However,
economists projected that the federal government would collect a far lower percentage
of total tax owed by delinquents than did state governments implementing similar
programs.
Which of the following, if true, would most contribute to an explanation of the
economists’ projections?
A. Tax amnesty programs are only successful if they are widely publicized.
B. Most people who honestly pay their state tax are equally honest in paying their
federal tax.
C. Although federal tax delinquents usually must pay high financial penalties, the
states require far lower financial penalties.
D. The state tax rate varies considerably from state to state, but the federal tax is
levied according to laws which apply to citizens of all the states.
E. Unlike most federal tax delinquents, most state tax delinquents fail to pay state
tax because of an oversight rather than a decision not to pay.
Answer:
*******************
Q10 to Q12:
Astronomers theorize that a black (20)whirling gas is not necessarily a
hole forms when a massive object black
shrinks catastrophically under its own hole: the concentration in M87 might
Line gravity, leaving only a gravitational be a cluster of a billion or so dim stars.
(5)field so strong that nothing escapes it. The same hypothesis might have
Astronomers must infer the existence been applied to the galaxy NGC 4258,
of black holes, which are invisible, (25)but the notion of such a cluster’s
from their gravitational influence on existing in NGC 4258 was severely
the visible bodies surrounding them. undermined when astronomers measured
(10)For example, observations indicate the speed of a ring of dust and
that gas clouds in galaxy M87 are gas rotating close to the galaxy’s
whirling unusually fast about the (30)center. From its speed, they
galaxy’s calculated
center. Most astronomers that the core’s density is more
believe that the large concentration than 40 times the density estimated
(15)of mass at the galaxy’s center is a for any other galaxy. If the center of
black hole whose gravity is causing NGC 4258 were a star cluster, the
the gas to whirl. A few skeptics have (35)stars would be so closely spaced
argued that the concentration of mass that collisions between individual
necessary to explain the speed of the stars would have long ago torn the
cluster apart.
--------------------------------------------------------------------------------
Q10:
The skeptics mentioned in the first paragraph would be most likely to agree with the
astronomers mentioned in line 13 about which of the following statements concerning
the galaxy M87?
A. The speed of the gas whirling around the center of M87 is caused by a dense
object that is not a black hole.
B. The concentration of mass at the center of M87 is probably a large cluster of
dim stars.
C. The presence of a black hole at the center of M87 is the most likely
explanation for the speed of the gas whirling about the galaxy’s core.
D. The speed of the gas whirling around the center of M87 is caused by a large
concentration of mass at the core of M87.
E. The gravitational influence of a star cluster would not be strong enough to
account for the speed of the gas whirling around the core of M87.
Answer:
--------------------------------------------------------------------------------
Q11:
The passage asserts which of the following about the existence of black holes?
A. Astronomers first speculated about the existence of black holes when they
observed gas whirling around the center of a particular galaxy.
B. Evidence used to argue for the existence of black holes is indirect, coming
from their presumed effects on other astronomical bodies.
C. Recent observations of certain astronomical bodies have offered proof.
D. A considerable body of evidence suggests the existence of black holes, even
though their behavior is not completely consistent with the laws of physics.
E. Many astronomers are skeptical about certain recent evidence that has been
used to argue for the existence of black holes.
Answer:
--------------------------------------------------------------------------------
Q12:
Which of the following, if true, would most clearly undermine the possible
explanation for the whirling gas in M87 that is mentioned in the last sentence of the
first paragraph?
A. The stars in a star cluster at the center of M87 could exert a strong
gravitational force without tearing the cluster apart.
B. A cluster of stars at the center would preclude the existence of certain other
astronomical phenomena that have been observed at the center of M87.
C. The stars within many existing galaxies, such as NGC 4258, are more closely
spaced than are the stars within the core of M87.
D. Only one other galaxy has been observed to contain gas clouds whirling about
its center as they do about the core of M87.
E. The gravitational force of a cluster of a billion or so dim stars would be
sufficient to cause a whirling ring of gas and dust to collect around the center
of a galaxy.
Answer:
------------------------------------------------------------------------------------------------------
Q13:
Although she had been known as an effective legislator first in the Texas Senate and
later in the United States House of Representatives, not until Barbara Jordan’s
participation in the hearings on the impeachment of President Richard Nixon in 1974
was she made a nationally recognized figure, as it was televised nationwide.
A. later in the United States House of Representatives, not until Barbara Jordan’s
participation in the hearings on the impeachment of President Richard Nixon
in 1974 was she made a nationally recognized figure, as it was
B. later in the United States House of Representatives, Barbara Jordan did not
become a nationally recognized figure until 1974, when she participated in the
hearings on the impeachment of President Richard Nixon, which were
C. later in the Untied States House of Representatives, it was not until 1974 that
Barbara Jordan became a nationally recognized figure, with her participation
in the hearings on the impeachment of President Richard Nixon, which was
D. then also later in the United States House of Representatives, not until 1974
did Barbara Jordan become a nationally recognized figure, as she participated
in the hearings on the impeachment of President Richard Nixon, being
E. then also later in the United States House of Representatives, Barbara Jordan
did not become a nationally recognized figure until 1974, when she
participated in the hearings on the impeachment of President Richard Nixon,
which was
Answer:
----------------------------------------------------------------------------------------------------------
Q14:
The quality of early pieces of blown glass excavated in Italy and Western Europe by
far surpass those of pieces from the eastern Mediterranean, when regarded not only in
terms of the variety of shapes represented, but also in terms of decorative techniques
and functionality.
A. by far surpass those of pieces from the eastern Mediterranean, when regarded
not only in terms of
B. surpasses by far those from the eastern Mediterranean, with regard not only to
C. far surpass that of pieces from the eastern Mediterranean, not only regarding
D. far surpasses that of the eastern Mediterranean, with regard to not only
E. far surpasses that of pieces from the eastern Mediterranean, not only with
regard to
Answer:
*******************
Q15:
Recent findings lend strong support to the theory that a black hole lies at the center of
the Milky Way and of many of the 100 billion other galaxies estimated to exist in the
universe.
A. that a black hole lies at the center of the Milky Way and of
B. that a black hole lies at the Milky Way’s center and
C. that there is a black hole lying at the Milky Way’s center and
D. of a black hole lying at the Milky Way’s center and
E. of a black hole that lies at the center of the Milky Way and of
Answer:
----------------------------------------------------------------------------------------------
Q16:
Last year a record number of new manufacturing jobs were created. Will this year
bring another record? Well, any new manufacturing job is created either within
an existing company or by the start-up of a new company. Within existing firms,
new jobs have been created this year at well below last year’s record pace. At the
same time, there is considerable evidence that the number of new companies starting
up this year will be no higher than it was last year and there is no reason to think
that the new companies starting up this year will create more jobs per company
than did last year’s start-ups. So clearly, the number of new jobs created this year
will fall short of last year’s record.
In the argument given, the two portions in boldface play which of the following
roles?
A. The first provides evidence in support of the main conclusion of the argument;
the second is a claim that argument challenges.
B. The first is a generalization that the argument seeks to establish; the second is
a conclusion that the argument draws in order to support that generalization.
C. The first is a generalization that the argument seeks to establish; the second is
a judgment that has been advanced in order to challenge that generalization.
D. The first is presented as obvious truth on which the argument is based; the
second is a claim that has been advanced in support of a position that the
argument opposes.
E. The first is presented as obvious truth on which the argument is based; the second is a
judgment advanced in support of the main conclusion of the argument.
Answer:
*******************
Q17:
So dogged were Frances Perkins’ investigations of the garment industry, and her
lobbying for wage and hour reform was persistent, Alfred E. Smith and Franklin D.
Roosevelt recruited Perkins to work within the government, rather than as a social
worker.
A. and her lobbying for wage and hour reform was persistent,
B. and lobbying for wage and hour reform was persistent, so that
C. her lobbying for wage and hour reform persistent, that
D. lobbying for wage and hour reform was so persistent,
E. so persistent her lobbying for wage and hour reform, that
Answer:
--------------------------------------------------------------------------------------------------
Q18:
The Near Earth Asteroid Rendezvous (NEAR) spacecraft will orbit the asteroid Eros
for a year, slowly moving closer to the surface of the object to make ever more precise
measurements that scientists hope will enable them to understand how the solar
system formed some four billion years ago.
A. to make ever more precise measurements that scientists hope will enable them
to
B. to make ever more and more precise measurements, which scientists are
hoping to enable them
C. for making ever more precise measurements, and scientists hope that they will
be able to
D. with the purpose of making more precise measurements than ever, and which
scientists hope will enable them to
E. in order to make more precise measurements than it ever did, and scientists are
hoping they will be able to
Answer:
*******************----
Q19:
Humans have been damaging the environment for centuries by overcutting trees and
farming too intensively, and though some protective measures, like the establishment
of national forests and wildlife sanctuaries, having been taken decades ago, great
increases in population and in the intensity of industrialization are causing a
worldwide ecological crisis.
A. though some protective measures, like the establishment of national forests
and wildlife sanctuaries, having been taken decades ago, great increases in
population
B. though some protective measures, such as the establishment of national forests
and wildlife sanctuaries, were taken decades ago, great increases in population
C. though some protective measures, such as establishing national forests and
wildlife sanctuaries having been taken decades ago, great population increases
D. with some protective measures, like establishing national forests and wildlife
sanctuaries that were taken decades ago, great increases in population
E. with some protective measures, such as the establishment of national forests
and wildlife sanctuaries, having been taken decades ago, great population
increases
Answer:
-----------------------------------------------------------------------------------------------
Q20:
Critics of certain pollution-control regulations have claimed that the money spent
over the last decade in order to reduce emissions of carbon monoxide and of
volatile organic compounds has been wasted. The evidence they offer in support
of this claim might appear compelling: despite the money spent, annual emissions of
these pollutants have been increasing steadily. This evidence is far from adequate,
however, since over the last decade a substantial number of new industrial
facilities that emit these pollutants have been built.
In the reasoning given, the two portions in boldface play which of the following
roles?
A. The first identifies a claim that the reasoning seeks to show is false; the second
is evidence that has been cited by others in support of that claim.
B. The first identifies a claim that the reasoning seeks to show is false; the second
is a position for which the reasoning seeks to provide support.
C. The first is a position that the reasoning contends is inadequately supported by
the evidence; the second is a position for which the reasoning seeks to provide
support.
D. The first is a position that the reasoning contends is inadequately supported by
the evidence; the second is evidence used to support the reasoning’s
contention.
E. The first is a position that the reasoning contends is inadequately supported by
the evidence; the second is evidence that has been used to support that
position.
Answer:
*******************
Q21:
The coyote is one of several recent ecological success stories: along with the
white-tailed deer, the moose, and other species that are enlarging their natural
domains, they have established themselves as supreme adapters in an era when the
capability to adjust to the environmental changes wrought by human beings has
created a whole new class of dominant large mammals.
A. they have established themselves as supreme adapters in an era when the
capability
B. they have established themselves as being supreme adapters in an era when
being able
C. it has established itself as a supreme adapter in an era when to be able
D. it has established itself as being a supreme adapter in an era when its ability
E. it has established itself as a supreme adapter in an era when the ability
Answer:
--------------------------------------------------------------------------------------------------
Q22:
Whales originated in the freshwater lakes and rivers of ancient Asia about sixty
million years ago. Not until about ten million years later did species of whales
develop specialized kidneys enabling them to drink salt water. Although fossil
evidence shows that some early whale species that lacked such kidneys sometimes
swam in the Earth’s saltwater oceans, these species must have had to return frequently
to freshwater rivers to drink.
Which of the following is most strongly supported by the information given?
A. Fossils of whale species dating from between sixty million and fifty million
years ago will not be found on continents that were at the time separated from
ancient Asia by wide expanses of ocean.
B. Among whale fossils that date from later than about fifty million years ago,
none are fossils of whale species that drank only fresh water.
C. Fossils of whale species that drank fresh water will not be found in close
proximity to fossils of whale species that drank salt water.
D. The earliest whales that drank salt water differed from fresh-water-drinking
whales only in their possession of specialized kidneys.
E. Between sixty million and fifty million years ago, the freshwater lakes and
rivers in which whales originated were gradually invaded by salt water.
Answer:
------------------------------------------------------------------------------------------------
Q 23 to Q 26: assumption.
When the history of women (25) Lash’s biography revealed a
began to receive focused attention Complicated woman who sought
in the 1970’, Eleanor Roosevelt Through political activity both to
Line was one of a handful of female flee inner misery and to promote
(5) Americans who were well known causes in which she passionately
to both historians and the general (30) believed. However, she still
public. Despite the evidence that appeared to be an idiosyncratic
she had been important in socialreform figure, somehow self-generated
circles before her husband not amenable to any generalized
(10) was elected President and that explanation. She emerged from
she continued to advocate different (35) the biography as a mother to the
causes than he did, she held entire nation, or as a busybody.
a place in the public imagination but hardly as a social type, a
largely because she was the wife figure comprehensible in terms
(15) of a particularly influential of broader social developments.
President. (40) But more recent work on the
Her own activities were feminism of the post-suffrage
seen as preparing the way for her years (following 1920) allows us
husband’s election or as a complement to see Roosevelt in a different
to his programs. Even light and to bring her life into a
(20) Joseph Lash’s two volumes of (45) more richly detailed context. Lois
Sympathetic biography, Eleanor and Scharf’s Eleanor Roosevelt, written
Franklin (1971) and Eleanor: The In 1987, depicts a generation of
Years Alone (1972), reflected this Privileged women, born in the late
Nineteenth century and maturing Answer:
(50) in the twentieth, who made the -------------------------------------------------
transition from old patterns of -------------------------------------------------
female association to new ones. -----
Their views and their lives were full Q 24:
Of contradictions. They maintained The author indicates that, according to
(55) female social networks but began Scharf’s
to integrate women into mainstream biography, which of the following was
politics; they demanded equal NOT
treatment but also argued that characteristic of feminists of Eleanor
women’s maternal responsibilities Roosevelt’s
(60) made them both wards and generation?
representatives A. Their lives were full of contradictions
of the public interest. B. Their policies identified them as
Thanks to Scharf and others, idiosyncratic.
Roosevelt’s activities—for example, C. They were from privileged
her support both for labor laws backgrounds.
(65) protecting women and for D. They held that women had unique
appointments responsibilities.
of women to high public E. They made a transition from old
office—have become intelligible in patterns
terms of this social context rather of a association to new ones.
than as the idiosyncratic career of Answer:
a famous man’s wife. -------------------------------------------------
------------------------------------------------- -------------------------------------------------
------------------------------------------------- -----
----- Q 25:
Q 23: Which of the following studies would
The passage as a whole is primarily proceed in a
concerned way most similar to the way in which,
with which of the following? according to
A. Changes in the way in which Eleanor the passage. Scharf’s book interprets
Roosevelt’s life is understood Eleanor
B. Social changes that made possible the Roosevelt’s career?
role A. An exploration of the activities of a
Played by Eleanor Roosevelt in social wealthy
reform social reformer in terms of the ideals
C. Changes in the ways in which held
historians have by the reformer
viewed the lives of American women B. A history of the leaders of a political
D. Social changes that resulted from the party
activities which explained how the conflicting
of Eleanor Roosevelt aims
E. Changes in the social roles that of its individual leaders thwarted and
American diverted the activities of each leader
women have played
C. An account of the legislative career of The author cites which of the following
a conservative as evidence
senator which showed his goals to against the public view of Eleanor
have been derived from a national Roosevelt held
conservative in the 1970’s?
movement of which the senator was A. She had been born into a wealthy
a part family.
D. A biography of a famous athlete B. Her political career predated the
which adoption
explained her high level of motivation in of women’s suffrage.
terms C. She continued her career in politics
of the kind of family in which she grew even
up After her husband’s death.
E. A history of the individuals who led D. She was one of a few female
the movement historical
to end slavery in the United States which Figures who were well known to
attributed the movement’s success to the historians
efforts of those exceptional individuals By the 1970’s.
Answer: E. Her activism predated her husband’s
------------------------------------------------- presidency
------------------------------------------------- and her projects differed from his.
----- Answer:
Q 26:
-------------------------------------------------------------------------------------------------------
Q27:missing
-----------------------------------------------------------------------------------------
Q28:
Which of the following most logically completes the argument?
A significant number of Qualitex Corporation’s department heads are due to retire this
year. The number of employees other than current department heads who could take
on the position of department head is equal to only about half of the expected
vacancies. Oualitex is not going to hire department heads from outside the company
or have current department heads take over more than one department, so some
departments will be without department heads next year unless Qualitex ______.
A. promotes some current department heads to higher-level managerial positions
B. raises the salary for department heads
C. reduces the number of new employees it hires next year
D. reduces the average number of employees per department
E. reduces the number of its departments
Answer:
*******************
Q29:missing
-----------------------------------------------------------------------------------------------
Q30:
According to some botanists, invasive plants are the second most serious threat, after
habitat loss, to native species of plants and animals and to the maintenance of
biologically diverse ecosystems.
A. threat, after habitat loss, to native species of plants and animals and to the
maintenance of biologically diverse ecosystems
B. threat, after habitat loss, to native species of plants and animals and for
maintaining biologically diverse ecosystems
C. threat, after losing their habitat, to native species of plants and animals and
also to maintenance of biologically diverse ecosystems
D. threat to native species of plants and animals and for maintaining biologically
diverse ecosystems, after habitat loss
E. threat to native species of plants and animals as well as to maintaining
biologically diverse ecosystems, after losing their habitat
Answer:
-----------------------------------------------------------------------------------------------------
Q31:
Maize contains the vitamin niacin, but not in a form the body can absorb. Pellagra is
a disease that results from niacin deficiency. When maize was introduced into
southern Europe from the Americas in the eighteenth century, it quickly became a
dietary staple, and many Europeans who came to subsist primarily on maize
developed pellagra. Pellagra was virtually unknown at that time in the Americas,
however, even among people who subsisted primarily on maize.
Which of the following, if true, most helps to explain the contrasting incidence of
pellagra described above?
A. Once introduced into southern Europe, maize became popular with
landowners because of its high yields relative to other cereal crops.
B. Maize grown in the Americas contained more niacin than maize grown in
Europe did.
C. Traditional ways of preparing maize in the Americas convert maize’s niacin
into a nutritionally useful form.
D. In southern Europe many of the people who consumed maize also ate
niacin-rich foods.
E. Before the discovery of pellagra’s link with niacin, it was widely believed that
the disease was an infection that could be transmitted from person to person.
Answer:
---------------------------------------------------------------------------------------------------
Q 32 to 34: predict. For example, most predators
Grassland songbirds often nest in of waterfowl nests prey
the same grassland-wetland complexes opportunitistically
as waterfowl, particularly in a certain on songbird nests, and removing
Line part of those complexes, namely, (15) these predators could directly
(5) upland habitats surrounding increase
wetlands. songbird nesting success. Alternatively,
Although some wildlife management small mammals such as mice
procedures directed at waterfowl, such and ground squirrels are important
as habitat enhancement or restoration, in the diet of many waterfowl-nest
may also benefit songbirds , the impact (20) predators and can themselves be
(10) of others, especially the control of important predators of songbird
waterfowl predators, remains difficult to nets. Thus. Removing waterfowl-nest
predators could affect songbird nesting songbird populations
success through subsequent increases E . a resulting increase in small-mammal
(25) in small-mammal populations. populations could increase
In 1995 and 1996, researchers smallmammal
trapped and removed certain predation on songbirds
waterfowlnest answer:
predators. primarily raccoons and -------------------------------------------------
striped skunks, then observed subse- -------------------------------------------------
(30) quent survival rates for songbird -----
nests. Q33
Surprisingly. They observed no it can be inferred that the habitat
significant preferences
effect on songbird nesting of raccoons and striped skunks affected
Success. This may be due to several the
Factors. Neither raccoons nor striped results of the experiment described in the
(35) skunks consume ground squirrels, passage for which of the following
which are important predators of reasons?
songbird A . Songbird nests in the wetlands are
nests. Thus, their removal may usually located in places that most
not have led to significant increases waterfowl-nest predators cannot reach.
in populations of smaller predators. B. Raccoons and striped skunks are not
(40) Additionally. Both raccoons and usually found in areas where songbird
striped nests tend to be located.
skunks prefer wetlands and spend little C. Mice and ground squirrels tend to
time in upland habitats; removing these avoid
species may not have increased the predation by raccoons and striped
nesting success of songbirds in the skunks by remaining exclusively in
uplands enough to allow detection. the uplands.
------------------------------------------------- D. The populations of small mammals in
------------------------------------------------- the wetlands are usually controlled by
Q32 larger waterfowl-nest predators such
The passage suggests that removing as raccoons and striped skunks.
Waterfowl-nest predators could possibly E. The waterfowl on which raccoons and
Have a negative effect on songbird striped skunks prey in the wetlands
populations compete with songbirds for food.
because Answer:
A. songbird populations could then -------------------------------------------------
grow to unsustainable numbers -------------------------------------------------
B. small-mammal population could -----
then move out of the uplands Q34
into wetland areas the primary purpose of the passage is to
C. competition among remaining A. describe some procedures used for
waterfowl-nest predators could wildlife management and consider
decrease significantly some problems associated with the
D. a resulting increase in waterfowl execution of those procedures
populations could crowd out B. outline a problem related to a wildlife
management procedure and offer for wildlife management should be
potential explanations for the results of modified because of its unintended
an experiment bearing on that problem consequences.
C. present experimental results that E. propose that further experiments be
illustrate the need for certain wildlife performed to assess the long-term
management procedures and point out effects of certain wildlife management
some inconsistencies in those results procedures.
D. argue that a certain procedure used Answer:
-------------------------------------------------------------------------------------------------------
Q35:
In polluted environments, dolphins gradually accumulated toxins in their body fat,
and the larger the dolphin the more accumulated toxin it can tolerate. Nearly 80
percent of the toxins a female dolphin has accumulated pass into the fat-rich milk her
nursing calf ingests. Therefore, the unusually high mortality rate among dolphin
calves in the industrially contaminated waters along Florida’s Gulf Coast is probably
the result of their being poisoned by their mother’s milk.
Which of the following, if true, most strengthens the argument?
A. The survival rate of firstborn dolphin calves in the area along Florida’s Gulf
Coast is highest for those whose mothers were killed before they were
weaned.
B. The rate at which adult dolphins living in the waters along Florida’s Gulf
Coast accumulate toxins is no higher than that of adult dolphins in comparably
polluted waters elsewhere.
C. Among dolphin calves born in the area along Florida’s Gulf Coast, the
mortality rate is highest among those with living siblings.
D. As dolphins age, they accumulate toxins from the environment more slowly
than when they were young.
E. Dolphins, like other marine mammals, have a higher proportion of body fat
than do most land mammals.
Answer:
*******************
Q36:
Scholars who once thought Native American literatures were solely oral narratives
recorded by missionaries or anthropologists now understand this body of work to
consist of both oral literatures and the written works of Native American authors, who
have been publishing since 1772.
A. Scholars who once thought Native American literatures were solely oral
narratives
B. Scholars thinking of Native American literatures once solely as oral narratives,
and
C. Scholars who once had thought of Native American literatures solely as oral
narratives and
D. Native American literatures, which some scholars once thought were solely
oral narratives
E. Native American literatures, which some scholars once, thinking they were
solely oral narratives
Answer:
------------------------------------------------------------------------------------------------------
Q37:missing
-----------------------------------------------------------------------------------------------------
Q38
People who have spent a lot of time in contact with animals often develop
animal-induced allergies, a significant percentage of which are quite serious. In a
survey of current employees in major zoos, about 30 percent had animal-induced
allergies. However, a zoo employee who develops a serious animal-induced allergy
is very likely to switch to some other occupation.
Which of the following hypotheses receives the strongest support from the
information given?
A. The incidence of serious animal-induced allergies among current zoo
employees is lower than that among the general population.
B. Zoo employees tend to develop animal-induced allergies that are more serious
than those of other people who spend equally large amounts of time with
animals.
C. Exposure to domestic pets is, on the whole, less likely to cause
animal-induced allergy than is exposure to the kinds of animals that are kept
in zoos.
D. There is no occupation for which the risk of developing an animal-induced
allergy is higher than 30 percent.
E. Among members of the general population who have spent as much time with
animals as zoo employees typically have, the percentage with animal-induced
allergies is significantly more than 30 percent.
Answer:
----------------------------------------------------------------------------------------------------
Q39:missing
----------------------------------------------------------------------------------------------------
Q40:
Many entomologists say that campaigns to eradicate the fire ant in the United States
have failed because the chemicals that were used were effective only in wiping out the
ant’s natural enemies, which made it easier for them to spread.
A. which made it easier for them
B. which makes it easier for it
C. thus making it easier for them
D. thus making it easier for the ant
E. thereby, it was made easier for the ant
Answer:
-------------------------------------------------------------------------------------------------
Q41
Which of the following most logically completes the argument?
Yorco and Zortech are two corporations that employ large numbers of full-time
workers who are paid by the hour. Publicly available records indicate that Yorco
employs roughly the same number of such hourly wage workers as Zortech does but
spends a far higher total sum per year on wages for such workers. Therefore, hourly
wages must be higher, on average, at Yorco than at Zortech, since _____.
A. Zortech spends a higher total sum per year than Yorco does to provide its
hourly wage workers with benefits other than wages
B. the work performed by hourly wage workers at Zortech does not require a
significantly higher level of skill than the work performed by hourly wage
workers at Yorco does
C. the proportion of all company employees who are hourly wage workers is
significantly greater at Yorco than it is at Zortech
D. overtime work, which is paid at a substantially higher rate than work done
during the regular work week, is rare at both Yorco and Zortech
E. the highest hourly wages paid at Yorco are higher than the highest hourly
wages paid at Zortech
Answer:
--------------------------------------------------------------------------------------------------
Answer:
AEEDC,EDEED,BABEA,EECBD,EAABD,E()E()A,CEBBA,A()E()D,D

1
Math Section
*******************
Q1: (2) The average height of the
Of the fruit that arrives at a students in class X and class Y
cannery, 20 percent by weight is combined is 126
rejected before processing. centimeters.
Of the fruit that is processed, 15 A. Statement (1) ALONE is
percent by weight is rejected sufficient, but statement (2)
before canning. Of the alone is not sufficient.
fruit that arrives at the cannery, B. Statement (2) ALONE is
what percent by weight is sufficient, but statement (1)
canned? alone is not sufficient.
A. 32% C. BOTH statements TOGETHER
B. 35% are sufficient, but NEITHER
C. 65% statement ALONE is
D. 68% sufficient.
E. 70% D. EACH statement ALONE is
Answer: sufficient.
******************* E. Statements (1) and (2)
Q2: TOGETHER are NOT sufficient.
What is the ratio of the average Answer:
(arithmetic mean) height of *******************
students in class X to the Q3:
average height of students in If the length of a certain
class Y? rectangle is 2 greater than the
(1) The average height of the width of the rectangle, what is
students in class X is 120 the
centimeters. perimeter of the rectangle?
(1) The length of each diagonal B. 15 to 1
of the rectangle is 10. C. 16 to 5
(2) The area of the rectangular D. 15 to 6
region is 48. E. 5 to 4
A. Statement (1) ALONE is Answer:
sufficient, but statement (2) *******************
alone is not sufficient. Q6:
B. Statement (2) ALONE is If a company allocates 15
sufficient, but statement (1) percent of its budget to
alone is not sufficient. advertising, 10 percent to
C. BOTH statements TOGETHER capital
are sufficient, but NEITHER improvements, and 55 percent
statement ALONE is to salaries, what fraction of its
sufficient. budget remains for other
D. EACH statement ALONE is allocations?
sufficient. A. 4/5
E. Statements (1) and (2) B. 3/5
TOGETHER are NOT sufficient. C. 3/10
Answer: D. 1/5
******************* E. 1/10
Q4: Answer:
2 *******************
For a nonnegative integer n, if Q7:
the remainder is 1 when 2n is If x(x - 5)(x + 2) = 0, is x
divided by 3, then which of negative?
the following must be true? (1) x2 – 7x ≠ 0
I. n is greater than zero. (2) x2 –2x –15 ≠ 0
II. 3n = (-3)n A. Statement (1) ALONE is
III. √2n is an integer. sufficient, but statement (2)
A. I only alone is not sufficient.
B. II only 3
C. I and II B. Statement (2) ALONE is
D. I and III sufficient, but statement (1)
E. II and III alone is not sufficient.
Answer: C. BOTH statements TOGETHER
******************* are sufficient, but NEITHER
Q5: statement ALONE is
A certain club has 20 members. sufficient.
What is the ratio of the member D. EACH statement ALONE is
of 5-member sufficient.
committees that can be formed E. Statements (1) and (2)
from the members of the club to TOGETHER are NOT sufficient.
the number of 4-member Answer:
committees that can be formed *******************
from the members of the club? Q8:
A. 16 to 1 QR
30° B. Statement (2) ALONE is
PS sufficient, but statement (1)
In parallelogram PQRS shown, if alone is not sufficient.
PQ = 4 and QR = 6, what is the 4
area of PQRS? C. BOTH statements TOGETHER
A. 8 are sufficient, but NEITHER
B. 12 statement ALONE is
C. 24 sufficient.
D. 8√3 D. EACH statement ALONE is
E. 12√3 sufficient.
Answer: E. Statements (1) and (2)
******************* TOGETHER are NOT sufficient.
Q9: Answer:
A certain farmer pays $30 per *******************
acre per month to rent Q11:
farmland. How much does the On Saturday morning, Malachi
farmer pay per month to rent a will begin a camping vacation
rectangular plot of farmland that and he will return home at
is 360 feet by 605 feet? the end of the first day on which
(43,560 square feet = 1 acre) it rains. If on the first three days
A. $5,330 of the vacation the
B. $3,630 probability of rain on each day is
C. $1,350 0.2, what is the probability that
D. $360 Malachi will return
E. $150 home at the end of the day on
Answer: the following Monday?
******************* A. 0.008
Q10: B. 0.128
When 200 gallons of oil were C. 0.488
removed from a tank, the D. 0.512
volume of oil left in the tank was E. 0.640
3/7 of the tank’s capacity. What Answer:
was the tank’s capacity? *******************
(1) Before the 200 gallons were Q12:
removed, the volume of oil in If x and y are integers and x >
the tank was 1/2 of 0, is y > 0?
the tank’s capacity. (1) 7x – 2y > 0
(2) After the 200 gallons were (2) -y < x
removed, the volume of oil left A. Statement (1) ALONE is
in the tank was 1,600 sufficient, but statement (2)
gallons less than the tank’s alone is not sufficient.
capacity. B. Statement (2) ALONE is
A. Statement (1) ALONE is sufficient, but statement (1)
sufficient, but statement (2) alone is not sufficient.
alone is not sufficient.
C. BOTH statements TOGETHER C. 25y
are sufficient, but NEITHER D. 100y
statement ALONE is E. 400y
sufficient. Answer:
D. EACH statement ALONE is *******************
sufficient. Q15:
E. Statements (1) and (2) If n is a positive integer and r is
TOGETHER are NOT sufficient. the remainder when (n – 1)(n +
Answer: 1) is divided by 24, what
******************* is the value of r?
Q13: (1) 2 is not a factor of n.
Lists S and T consist of the same (2) 3 is not a factor of n.
number of positive integers. Is A. Statement (1) ALONE is
the median of the sufficient, but statement (2)
integers in S greater than the alone is not sufficient.
average (arithmetic mean) of B. Statement (2) ALONE is
the integers in T? sufficient, but statement (1)
(1) The integers in S are alone is not sufficient.
consecutive even integers, and C. BOTH statements TOGETHER
the integers in T are are sufficient, but NEITHER
consecutive odd integers. statement ALONE is
(2) The sum of the integers in S sufficient.
is greater than the sum of the D. EACH statement ALONE is
integers in T. sufficient.
A. Statement (1) ALONE is E. Statements (1) and (2)
sufficient, but statement (2) TOGETHER are NOT sufficient.
alone is not sufficient. Answer:
B. Statement (2) ALONE is *******************
sufficient, but statement (1) Q16:
alone is not sufficient. In the sequence of nonzero
C. BOTH statements TOGETHER numbers t1, t2, t3, …, tn, …, tn+1 =
are sufficient, but NEITHER tn / 2 for all positive integers
statement ALONE is n. What is the value of t5?
sufficient. (1) t3 = 1/4
D. EACH statement ALONE is (2) t1 - t5 = 15/16
sufficient. A. Statement (1) ALONE is
E. Statements (1) and (2) sufficient, but statement (2)
TOGETHER are NOT sufficient. alone is not sufficient.
Answer: B. Statement (2) ALONE is
5 sufficient, but statement (1)
******************* alone is not sufficient.
Q14: C. BOTH statements TOGETHER
If x percent of 40 is y, then 10x are sufficient, but NEITHER
equals statement ALONE is
A. 4y sufficient.
B. 10y
D. EACH statement ALONE is E. Statements (1) and (2)
sufficient. TOGETHER are NOT sufficient.
E. Statements (1) and (2) Answer:
TOGETHER are NOT sufficient. *******************
Answer: Q19:
******************* Which of the following is equal
Q17: to (212 – 26) / (26 – 23)?
Last year the price per share of A. 26 + 23
Stock X increased by k percent B. 26 - 23
and the earnings per share C. 29
of Stock X increased by m D. 23
percent, where k is greater than E. 2
m. By what percent did the Answer:
ratio of price per share to *******************
earnings per share increase, in Q20:
terms of k and m? Of the students in a certain
A. k m % school, 15 percent are enrolled
B. (k – m) % in an art class and 10 percent
6 are enrolled in a music class.
C. [100(k – m)] / (100 + k) % What percent of the students in
D. [100(k – m)] / (100 + m) % the school are enrolled in
E. [100(k – m)] / (100 + k + m) % neither an art class nor a music
Answer: D class?
******************* (1) 2/3 of the students who are
Q18: enrolled in an art class are also
What is the total value of enrolled in a music
Company H’s stock? class.
(1) Investor P owns 1/4 of the (2) There are more than 100
shares of Company H’s total students in the school.
stock. A. Statement (1) ALONE is
(2) The total value of Investor sufficient, but statement (2)
Q’s shares of Company H’s stock alone is not sufficient.
is $16,000. B. Statement (2) ALONE is
A. Statement (1) ALONE is sufficient, but statement (1)
sufficient, but statement (2) alone is not sufficient.
alone is not sufficient. C. BOTH statements TOGETHER
B. Statement (2) ALONE is are sufficient, but NEITHER
sufficient, but statement (1) statement ALONE is
alone is not sufficient. sufficient.
C. BOTH statements TOGETHER D. EACH statement ALONE is
are sufficient, but NEITHER sufficient.
statement ALONE is E. Statements (1) and (2)
sufficient. TOGETHER are NOT sufficient.
D. EACH statement ALONE is Answer:
sufficient. *******************
Q21:
If x < 0, then √(-x│x│) is D. EACH statement ALONE is
7 sufficient.
A. -x E. Statements (1) and (2)
B. -1 TOGETHER are NOT sufficient.
C. 1 Answer:
D. x *******************
E. √x 8
Answer: Q24:
******************* Month Number of Days Worked
Q22: June 20
July 17
For any integer k greater than 1, August 19
the symbol k* denotes the The table above shows the
product of all the fractions of number of days worked by a
the form 1/t, where t is an certain sales representative in
integer between 1 and k, each of three months last year.
inclusive. What is the value of If the number of sales calls that
5*/4*? the representative made
A. 5 each month was proportional to
B. 5/4 the number of days worked in
C. 4/5 that month and if the
D. 1/4 representative made a total of
E. 1/5 168 sales calls in the three
Answer: months shown, how many sales
******************* calls did the representative
Q23: make in August?
l1 l2 A. 50
y° p B. 51
q C. 56
r D. 57
x° E. 60
s Answer:
t *******************
If l1 ║ l2 in the figure above, is x Q25:
= y? Pat invested x dollars in a fund
(1) p║r and r║t that paid 8 percent annual
(2) q║s interest, compounded annually.
A. Statement (1) ALONE is Which of the following
sufficient, but statement (2) represents the value, in dollars,
alone is not sufficient. of Pat’s investment plus interest
B. Statement (2) ALONE is at the end of 5 years?
sufficient, but statement (1) A. 5(0.08x)
alone is not sufficient. B. 5(1.08x)
C. BOTH statements TOGETHER C. [1 + 5(0.08)]x
are sufficient, but NEITHER D. (1.08)5x
statement ALONE is E. (1.08x)5
sufficient.
Answer: Answer:
******************* *******************
Q26: Q29:
If n = 3k, is k an integer? If x > 0.9, which of the following
(1) n is an integer. could be the value of x?
(2) n/6 is an integer. A. √0.81
A. Statement (1) ALONE is B. √0.9
sufficient, but statement (2) C. (0.9)2
alone is not sufficient. D. (0.9)(0.99)
B. Statement (2) ALONE is E. 1 - √0.01
sufficient, but statement (1) Answer:
alone is not sufficient. *******************
C. BOTH statements TOGETHER Q30:
are sufficient, but NEITHER Development planners
statement ALONE is determined the number of new
sufficient. housing units needed in a
D. EACH statement ALONE is certain
sufficient. area by using the formula H =
E. Statements (1) and (2) kJ, where H is the number of
TOGETHER are NOT sufficient. new housing units needed in
Answer: the area, J is the number of new
******************* jobs to be created in the area,
Q27: and k is a constant. How
9 many new housing units did the
In the xy-plane, what is the slope planners determine were
of the line with equation 3x + 7y needed?
= 9? (1) The number of new jobs to
A. – 7/3 be created was 60,000.
B. – 3/7 (2) According to the formula
C. 3/7 used by the planners, if 37,500
D. 3 jobs were to be created,
E. 7 then 7,500 new housing units
Answer: would be needed.
******************* A. Statement (1) ALONE is
Q28: sufficient, but statement (2)
In a certain English class, 1/4 of alone is not sufficient.
the number of girls is equal to B. Statement (2) ALONE is
1/6 of the total number of sufficient, but statement (1)
students. What is the ratio of alone is not sufficient.
the number of boys to the C. BOTH statements TOGETHER
number of girls in the class? are sufficient, but NEITHER
A. 1 to 4 statement ALONE is
B. 1 to 3 sufficient.
C. 1 to 2 D. EACH statement ALONE is
D. 2 to 3 sufficient.
E. 2 to 1
E. Statements (1) and (2) If 30!/10! is written as the
TOGETHER are NOT sufficient. product of consecutive integers,
Answer: the largest of which is 30,
10 what is the smallest of the
******************* integers?
Q31: A. 1
A solid yellow stripe is to be B. 3
painted in the middle of a C. 7
certain highway. If 1 gallon of D. 11
paint covers an area of p square E. 20
feet of highway, how many Answer:
gallons of paint will be *******************
needed to paint a stripe t inches Q34:
wide on a stretch of highway m If Antonio bought two half-gallon
miles long? (1 mile = cartons of ice cream during a
5,280 feet and 1 foot = 12 special sale, what percent
inches) of the total regular price of the
A. (5,280 mt) / 12p two cartons did he save?
B. (5,280 pt) / 12m (1) Antonio paid the regular
C. (5,280 pmt) / 12 price for the first carton and
D. (5,280)(12m) / pt received the second carton
E. (5,280)(12p) / mt for half the regular price.
Answer: (2) The regular price of the ice
******************* cream Antonio bought was
Q32: $4.00 per half-gallon
Is the integer n even? carton.
(1) n – 5 is an odd integer. 11
(2) n/5 is an even integer. A. Statement (1) ALONE is
A. Statement (1) ALONE is sufficient, but statement (2)
sufficient, but statement (2) alone is not sufficient.
alone is not sufficient. B. Statement (2) ALONE is
B. Statement (2) ALONE is sufficient, but statement (1)
sufficient, but statement (1) alone is not sufficient.
alone is not sufficient. C. BOTH statements TOGETHER
C. BOTH statements TOGETHER are sufficient, but NEITHER
are sufficient, but NEITHER statement ALONE is
statement ALONE is sufficient.
sufficient. D. EACH statement ALONE is
D. EACH statement ALONE is sufficient.
sufficient. E. Statements (1) and (2)
E. Statements (1) and (2) TOGETHER are NOT sufficient.
TOGETHER are NOT sufficient. Answer:
Answer: *******************
******************* Q35:
Q33:
If n is a positive integer, which D. 1 1/4
of the following is a possible E. 1 1/2
value of |56 - 5n|? Answer:
A. 7 *******************
B. 9 Answers:
C. 12 DEDEC, DCBED, BECCC, DDEAA,
D. 15 AEEDD, BBCBC, ADDAB, EC
E. 20
Answer: Verbal
******************* .
Q36: Q1.
Is y – x positive? Because of wireless service costs
(1) y > 0 plummeting in the last year, and as
(2) x = 1 - y mobile
A. Statement (1) ALONE is Phones are increasingly common, many
sufficient, but statement (2) people now using their mobile phones
alone is not sufficient. To make calls across a wide region at
B. Statement (2) ALONE is night and on weekends, when numerous
sufficient, but statement (1) Wireless companies provide unlimited
alone is not sufficient. airtime for a relatively small monthly
C. BOTH statements TOGETHER fee.
are sufficient, but NEITHER A. Because of wireless service costs
statement ALONE is plummeting in the last year, and as
sufficient. Mobile phones are increasingly
D. EACH statement ALONE is common, many people.
sufficient. B. As the cost of wireless service
E. Statements (1) and (2) plummeted in the last year and as mobile
TOGETHER are NOT sufficient. Phones became increasingly common,
Answer: many people.
******************* C. In the last year, with the cost of
Q37: wireless service plummeting, and mobile
Working alone at their Phones have become increasingly
respective constant rates, common, there are many people.
machine A and machine B can fill D. With the cost of wireless service
a plummeting in the last year and mobile
certain order in 3 hours and 6 Phones becoming increasingly common,
many people are
hours, respectively. If the two
E. While the cost of wireless service has
machines work
plummeted in the last year and
simultaneously at their
Mobile phones are increasingly
respective constant rates, how
common, many people are.
many hours does it take the two
Answer:
machines to fill 1/2 of that
-------------------------------------------------
order?
-----------------------------------------
A. 1/2
Q2.
B. 3/4
C. 1
In two months, the legal minimum wage done nothing to improve the situation,
in the country of Kirlandia will increase because it was coupled with
from five Kirlandic dollars(KD5.00) Per a ban on receiving money for lectures
hour to KD5.50 per hour. Opponents of and teaching engagements.
this increase have argued that the Pat: No, the raise in salary really does
resulting rise in wages will drive the improve the situation. Since very few
inflation judges teach or give lectures, the ban
rate up. In fact its impact on wages will will have little or no negative
probably be negligible, since only a effect.
very small proportion of all Kirfandic Pat’s response to Mel is inadequate in
workers are currently receiving less than that it
KD5.50 per hour. A. attempts to assess how a certain
Which of the following, if true, most change will affect potential members
seriously weakens the argument? of a group by providing evidence about
A. Most people in kirlandia who are its effect on the current members.
currently earning the minimum wage B. mistakenly takes the cause of a
have certain change to be an effect of that
been employed at their current jobs for change
less than a year. C. attempts to argue that a certain
B. Some firms in Kirlandia have paid change will have a positive effect merely
workers considerably less than KD5.00 by pointing to the absence of negative
per hour, in violation of kirlandic effects
employment regulations. D. simply denies Mel’s claim without
C. Many businesses hire trainees at or putting forward any evidence in support
near the minimum wage but must of that denial
reward trained workers by keeping their E. assumes that changes that benefit the
paylevels above the pay level most able members of a group
of trainees. necessarily benefit all members of that
D. The greatest growth in Kirlandia’s group.
economy in recent years has been in Answer:
those sectors where workers earn wages -------------------------------------------------
that tend to be much higher -------------------------------------------------
than the minimum wage. ---
E The current minimum wage is Q4.
insufficient for a worker holding only Recent findings lend strong support to
one job the theory that a black hole lies at the
to earn enough to support a family ,even center of
when working full time at that job. the Milky Way and of many of the 100
Answer: billion other galaxies estimated to exist
------------------------------------------------- in the
---------------------------------------- universe.
Q3. A. that a black hole lies at the center of
Mel: The official salary for judges has the Milky Way and of
always been too low to attract the best B. that a black hole lies at the Milky
Candidates to the job. The legislature’s Way’s center and
move to raise the salary has
C. that there is a black hole lying at the (1775-1783), an ideology of “republican
milky Way’s center and Line motherhood” resulted in a surge of
D. of a black hole lying at the Milky edu-
Way’s center and (5)cational opportunities for women in
E. of a black hole that lies at the center the
of the Milky Way and of United States. Kerber maintained that
answer: the leaders of the new nation wanted
------------------------------------------------- women to be educated in order to raise
------------------------------------------------- oolitically virtuous sons. A virtuous citi-
---- (10)zenry was considered essential to the
Q5. success of the country’s republican form
For many revisionist historians, of government; virtue was to be instilled
Christopher Columbus has come to not only by churches and schools, but
personify devastation and enslavement by families, where the mother’s role
in the name of progress that (15) was crucial. Thus, according to
has decimated native peoples of the Kerber,
Western Hemisphere. motherhood became pivotal to the fate
A. devastation and enslavement in the of the republic, providing justification
name of progress that for
has decimated native peoples of the an unprecedented attention to female
Western Hemisphere education.
B. devastation and enslavement in the (20) Introduction of the republican
name of progress by motherhood
which native peoples of the Western thesis dramatically changed
Hemisphere historiography. Prior to Kerber’s work,
decimated educational historians barely mentioned
C. devastating and enslaving in the name women and girls; Thomas Woody’s 1929
of progress those native (25) work is the notable exception.
peoples of the Western Hemisphere Examining
which in the name of progress are newspaper advertisements for
decimated. academies.
D. devastating and enslaving those Woody found that educational
native peoples of the western opportunities increased for both girls
Hemisphere which in the name of and boys around 1750. pointing to “An
progress are decimated. (30) Essay on Woman” (1753) as
E. the devastation and enslavement in reflecting
the name of progress that a shirt in view. Woody also claimed that
have decimated the native peoples of the practical education for females had
Western Hemisphere. many advocates before the Revolution,
Answer: Woody’s evidence challenges the notion
------------------------------------------------- (35) that the Revgolution changed
--------------------------------------- attiludes
Q6 TO 9 regarding female education, although it
Linda Kerber argued in the mid- may have accelerated earlier trends.
1980’s that after the American Historians’ reliance on Kerber’s
Revolution “republican
mother hood” thesis may have Which of the following for its success?
(40) obscured the presence of these A. Women assuming the sole
trends, responsibility
making it difficult to determine to what for instilling political virtue in
extent the Revolution really changed Children.
women’s lives. B. Girls becoming the primary focus of
------------------------------------------------- A reformed educational system that
------------------------------------------------- emphasized political virtue
----- C. The family serving as one of the pri-
Q6. Mary means by which children were
According to the passage, kerber main- Imbued with political virtue
Tained that which of the following led to D. The family assuming many of the
An increase in educational opportunities Functions previously performed by
For women in the United States after the Schools and churches.
American Revolution? E. Men and women assuming equal
A. An unprecedented demand by women responsibility for the management
For greater educational opportunities of schools, churches, and the family
In the decades following the Revolution answer:
B. A new political ideology calling for -------------------------------------------------
Equality of opportunity between -------------------------------------------------
Women and men in all aspects of life -----
C. A belief that the American Q8.-
educational According to the passage, within the
system could be reformed only if field
Women participated more fully in Of educational history, Thomas Woody’s
that system 1929 work was
D A belief that women needed to be A. innovative because it relied on
educated if they were to contribute newspaper
to the success of the nation’s new advertisements as evidence
form of government. B. exceptional in that it concentrated on
E. A recognition that women needed to the period before the American
be educated if they were to take an Revolution
active role in the nation’s schools C. unusual in that it focused on
and churches. educational
Answer: attitudes rather than on
------------------------------------------------- educational practices
------------------------------------------------- D. controversial in its claims regarding
----- educational opportunities for boys.
Q7. E. atypical in that it examined the
According to the passage. Kerber argued education of girls.
That political leaders thought that the Answer
form -------------------------------------------------
Of government adopted by the United -------------------------------------------------
States -----
After the American Revolution depended Q9.
on The passage suggests that, with regard
to the history of women’s education in two hours faster than anyone had yet
the done.
United States, Kerber’s work differs C. but also swam the distance in almost
from two hours faster than anyanyone
Woody’s primarily concerning which of had yet done, and setting a record for
the following? speed
A. The extent to which women were D. but also setting a record for speed by
interested in pursuing educational swimming the distance in
opportunities in the eighteenth century almost two hours faster than anyone had
B. The extent of the support for yet done.
educational E. but, swimming the distance almost
opportunities for girls prior to two hours faster than anyone
the American Revolution. had yet done, she also set a record for
C. The extent of public resistance to speed.
educational Answer:
opportunities for women after -------------------------------------------------
the American Revolution. -------------------------------------------------
D. Whether attitudes toward women’s -----
educational opportunities changed Q11.
during the eighteenth century. which of the following most logically
E. Whether women needed to be completes the passage?
educated On the whole, scientists do their most
in order to contribute to the success creative work before age forty, a
of a republican form of government tendency that has been taken to show
answer: that aging carries with it a loss of
------------------------------------------------- creative capacity. An alternative
------------------------------------------------- explanation is that by age forty most
----- scientists have worked in their field for
Q10. fifteen or more years and that by
In 1926, in her second attempt to swim then they have exhausted the opportunity
across the English Channel, for creative work in that field.
Gertrude Ederle not only crossed the Supporting this explanation is the
Channel against currents that finding that
Forced her to swim thirty-five miles A. the average age of recipients of
instead of the minimal twenty-one, scientific research grants is significantly
but she set a record for speed as well, by greater than forty.
swimming the distance in B. a disproportionately large number of
almost two hours faster than anyone had the scientists who produce
yet done. highly creative work beyond age forty
A. but she set a record for speed as well, entered their field at an
by swimming the distance older age than is common.
in almost two hours faster than anyone C. many scientists temper their own
had yet done. expectations of what they can
B. but also set a record for speed, achieve in their research work by their
swimming the distance almost belief that their creativity.
will decline as they age.
D. scientists who are older than forty responsibility of night and weekend
tend to find more satisfaction in work.
other activities, such as teaching and D. Most patients would rather trust their
mentoring, than they do in physicians than their insurance
pursuing their own research. companies to make decisions about their
E. there is a similar diminution of treatment.
creativity with age in nonscientific E. Since the insurance companies pay
fields, such as poetry and musical physicians a set amount for each
composition. office visit, it is to physicians’ financial
Answer: advantage to see as many
------------------------------------------------- Patients as possible.
------------------------------------------------- Answer:
----- -------------------------------------------------
Q12. -------------------------------------------------
In Kantovia, physicians’ income comes -----
from insurance companies, which Q 13 to 15:
require Grassland songbirds often nest in
physicians to document their decisions the same grassland-wetland complexes
in treating patients and to justify as waterfowl, particularly in a certain
deviations Line part of those complexes, namely,
from the companies’ treatment (5) upland habitats surrounding
guidelines. Ten years ago physicians wetlands.
were allowed Although some wildlife management
more discretion. Most physicians believe procedures directed at waterfowl, such
that the companies’ requirements now as habitat enhancement or restoration,
prevent them from spending enough may also benefit songbirds , the impact
time with patients. Yet the average (10) of others, especially the control of
amount of waterfowl predators, remains difficult to
time a patient spends with a physician predict. For example, most predators
during an office visit has actually of waterfowl nests prey opportunistically
increased on songbird nests, and removing
somewhat over the last ten years. (15) these predators could directly
Which of the following, if true, most increase
helps to resolve the apparent discrepancy songbird nesting success. Alternatively,
between physicians’ perceptions and the small mammals such as mice
change in the actual time spent? and ground squirrels are important
A. Patients are more likely to be in a in the diet of many waterfowl-nest
hurry nowadays and are less willing to (20) predators and can themselves be
wait a long time to see their physician. important predators of songbird
B. Physicians today typically have a nets. Thus. Removing waterfowl-nest
wider range of options in diagnosis and predators could affect songbird nesting
treatment to consider with the patient success through subsequent increases
before prescribing. (25) in small-mammal populations.
C. Physicians are increasingly likely to In 1995 and 1996, researchers
work in group practices, sharing the trapped and removed certain
waterfowlnest
predators. primary raccoons and such as mice and ground squirrels,
striped skunks, then observed subse- populations of ground squirrels tend
(30) quent survival rates for songbird to increase quickly enough to
nests. compensate
Surprisingly. They observed no for this level of predation.
significant D. Although ground squirrels have been
effect on songbird nesting known to prey on songbird nests, a
success. This may be due to several larger portion of their diets is usually
factors. Neither raccoons nor striped provided by predation on waterfowl
(35) skunks consume ground squirrels, nests.
which are important predators of E. Since larger predators tend to prefer
songbird small mammals to songbird eggs as
nests. Thus, their removal may a food source, a large population of
not have led to significant increases ground squirrels plays an important
in populations of smaller predators. role in controlling opportunistic
(40) Additionally, both raccoons and predation on songbird nests.
striped Answer:
skunks prefer wetlands and spend little -------------------------------------------------
time in upland habitats; removing these -------------------------------------
species may not have increased the Q 14:
nesting success of songbirds in the Which of the following best describes
uplands enough to allow detection. the function of the sentence “Neither
------------------------------------------------- raccoons…songbird nests” (lines 34-37)
------------------------------------------------- in the context of the passage as a whole?
----- A. It raises questions about the validity
Q 13: of a
NOTE: You must scroll to read the theory described in the first paragraph.
answer B. It points out an oversimplification
choices for this question. that is
According to the passage, which of the inherent in the argument presented in
following is true about the role played the first paragraph.
by ground squirrels in the ecology of C. It introduces information that may
grassland-wetland complexes? help
A. While not important in the diet of explain the results of the experiment
raccoons or striped skunks, ground that are presented earlier in the
squirrels are a significant source paragraph.
of food for other waterfowl-nest D. It provides a specific example of the
predators. type of data collected in the experiment
B. Whereas ground squirrels are described earlier in the paragraph.
typically important as predators of E. It anticipates a potential objection to
songbird nests, their opportunistic the
predation on waterfowl nests also conclusions drawn by the researchers
has an observable effect on waterfowl involved in the experiment described
nesting success. earlier in the paragraph.
C. Although most waterfowl-nest Answer:
predators prey on small mammals
------------------------------------------------- Years, explaining hominid features like
------------------------------------- D fire used to cook food could date back
Q 15: almost two million
The primary purpose of the passage is to years ,explaining hominid features such
A. describe some procedures used for as having.
wildlife management and consider E fire used for cooking food could be
some problems associated with the dated back to almost
execution of those procedures two million years and explain hominid
B. outline a problem related to a wildlife features like.
management procedure and offer Answer:
potential explanations for the results of -------------------------------------------------
an experiment bearing on that problem -----------------------------------------
C. present experimental results that Q17
illustrate the need for certain wildlife Until recently, the Inuit people led a
management procedures and point out nomadic existence, sheltering in
some inconsistencies in those results igloos, the ice-block domes that are
D. argue that a certain procedure used peculiar to north-central Canada,
for wildlife management should be and in structures made of stones, bones,
modified because of its unintended driftwood, and skins.
consequences A. Until recently, the Inuit people led a
E. propose that further experiments be nomadic existence, sheltering
performed to assess the long-term B. During recent times, the Inuit people
effects of certain wildlife management lead a nomadic existence,
procedures sheltering.
answer: C. In the times that are recent, the Inuit
------------------------------------------------- people led a nomadic existence,
----------------------------------------- sheltered.
Q 16: D. Up until recently, the Inuit people,
A Harvard anthropologist has proposed leading a nomadic existence,
that using fire to cook have sheltered.
food could be dated back to almost two E. Until recent times, leading a nomadic
million years and that existence, the Inuit people
it could explain hominid features like were sheltered.
having a large brain and Answer:
small teeth. -------------------------------------------------
A using fire to cook food could be dated -----------------------------------------
back to almost two Q18
million years and that it could explain In contrast to environmentalists
hominid features proposals to limit emissions of certain
like having pollutants, the administration proposed
B the use of fire to cook food could date calling for mandatory restrictions of
back almost two only three such pollutants from power
million years and could explain such plants-mercury, sulfur dioxide, and
hominid features as nitrogen oxides-and the plan would
C cooking food with fire could date back delay such cuts until 2010 or later.
to almost two million
A. administration proposed calling for -------------------------------------------------
mandatory restriction of -----------------------------------------
B. administration proposed a call for Q20
mandatory restrictions, including those Most states impose limitations on the
for authority of the legislature to borrow
C. administration, proposing mandatory money,
restrictions on with their objectives being to protect
D. administration’s proposal was a call taxpayers and the credit of the state
for mandatory restrictions, which government.
include A. to borrow money, with their
E. administration’s proposal would call objectives being to protect.
for mandatory restrictions on B. to borrow money, the objectives of
answer: which are the protecting of
------------------------------------------------- C. to borrow money, limitations intended
-------------------------------------- to protect.
Q19 D. for borrowing money, of which the
From 1980 to 1989, total consumption of objective is protecting.
fish in the country of Jurania increased E. for borrowing money, limitations with
by 4.5 percent, and total consumption of the intent of protecting.
poultry products there increased by Answer:
9.0 percent. During the same period, the -------------------------------------------------
population of Jurania increased by ----------------------------------------
6 percent, in part due to immigration to Q21
Jurania from other countries in the Which of the following most logically
region. completes the editorial below?
If the statements above are true, which Editorial in Golbindian Newspaper: For
of the following must also be true on the almost three months, opposition parties
basis of them? have been mounting daily street
A. During the 1980’s in Jurania, profits demonstrations in the capital in an effort
of wholesale distributors of poultry to
products increased at a greater rate than pressure the ruling party into calling an
did profits of wholesale distributors election. Though the demonstrations
of fish. were well attended at first, attendance
B. For people who immigrated to Jurania has declined steadily in recent weeks.
during the 1980’s , fish was less However, the decline in attendance does
likely to be a major part of their diet than not indicate that popular support for
was poultry. the opposition’s demands is dropping,
C. In 1989 Juranians consumed twice as since
much poultry as fish. A. the opposition’s demands have not
D. For a significant of jurania’s changed during the period when the
population, both fish and poultry street demonstrations have been
products were a regular part of their diet mounted.
during the 1980’s. B. No foreign governments have
E. Per capita consumption of fish in expressed any support for the
Jurania was lower in 1989 than in 1980. opposition’s
Answer: demands.
C. The state-controlled media have D. Responding to false alarms
ceased any mention of the significantly reduces the fire
demonstrations, department’s
leaving many citizens outside the capital capacity for responding to fires.
with no way of knowing that E. On any given day, a significant
demonstrations continue. percentage of the public telephones
D. There have not recently been any in Springfield are out of service.
antigovernment demonstrations in cities Answer:
other than the capital. -------------------------------------------------
E. A recent sharp decrease in -----------------------------------------
unemployment has led to increased Q23.
popular Because fish look through water, their
support for the government. eyes are very
Answer: different from a mammal.
------------------------------------------------- A. from a mammal
----------------------------------------- B. from a mammal’s
Q22. C. from that of a mammal
Springfield Fire Commissioner: the vast D. than that of a mammal
majority of false fire alarms E. than is a mammal’s.
are prank calls made anonymously from answer:
fire alarm boxes on street -------------------------------------------------
corners. Since virtually everyone has -----------------------------------------
access to a private telephone, Q24
these alarm boxes have outlived their Although improved efficiency in
usefulness. Therefore, we converting harvested trees into wood
propose to remove the boxes. Removing products
the boxes will reduce the may reduce harvest rates, it will
number of prank calls without stimulate demand by increasing supply
hampering people’s ability to report a and
fire. lowering prices, thereby boosting
Which of the following, if true, most consumption.
strongly supports the claim that the A. in converting harvested trees into
proposal, if carried out, will have the wood products may reduce harvest
announced effect? rates, it will stimulate demand by
A. The fire department traces all alarm increasing supply and lowering prices,
calls made from private telephones thereby boosting.
and records where they came from. B. In converting harvested trees into
B. Maintaining the fire alarm boxes wood products may reduce harvest
costs Springfield approximately rates, demand will be stimulated because
five million dollars annually. of increasing supply and
C. A telephone call can provide the fire lowering prices, which boost.
department with more information C. Of converting harvested trees into
about the nature and size of a fire than wood products may reduce harvest
can an alarm placed rates, it will stimulate demand by
from an alarm box. increasing supply and lowering prices,
which boosts.
D. Of harvested trees being converted itself. While some Socialists did
into wood products may reduce view price protests as a direct
harvest rates, it will stimulate demand, (35) step toward socialism, most
because it will increase supply Socialists ultimately sought to
and lower prices, thereby boosting. divert the cost-of-living movement
E. When harvested trees are converted into alternative channels of protest.
into wood products may reduce Union organizing, they argued,
harvest rates, demand will be stimulated (40) was the best method through which
because of increasing supply to combat the high cost of living.
and lowering prices, which boost. For others, cost-of-living or oganizing
Answer: was valuable insofar as it led
------------------------------------------------- women into the struggle for suf-
----------------------------------------- (45) frage, and similarly, the suffrage
Q25 to 28 struggle was valuable insofar as
In mid-February 1917 a it moved United States society
women’s movement independent one step closer to socialism.
of political affiliation erupted in Although New York’s Social-
Line New York City, the stronghold of (50) ists saw the cost-of-living issue
(5) the Socialist party in the United as, at best ,secondary or tertiary
states. Protesting against the high to the real task at hand, the boycotters,
cost of living, thousands of women by sharp contrast, joined
refused to buy chickens, fish, and the price protest movement out of
vegetables. The boycott shut. (55) an urgent and deeply felt
(10) down much of the City’s foodstuffs commitment
marketing for two weeks, riveting to the cost-of-living issue.
public attention on the issue of A shared experience of swiftly
food prices, which had increased declining living standards caused
partly as a result of increased by rising food prices drove these
(15) exports of food to Europe that had (60) women to protest. Consumer
been occurring since the outbreak organizing spoke directly to their
of the First World War. daily lives and concerns; they
By early 1917 the Socialist saw cheaper food as a valuable
party had established itself as a end in itself. Food price protests
(20) major political presence in New (65) were these women’s way of
York City. New York Socialists, organizing
whose customary spheres of at their own workplace, as
struggle were electoral work and workers whose occupation was
trade union organizing, seized the shopping and preparing food for
(25) opportunity and quickly organized their families.
an extensive series of cost-ofliving -------------------------------------------------
protests designed to direct -----------------------------------------
the women’s movement toward Q25
Socialist goals. Underneath the The author suggests which of the
(30) Socialists’ brief commitment to following about
cost-of-living organizing lay a the New York Socialists’ commitment to
basic indifference to the issue the costof-
living movement? Which of the following best states the
A. It lasted for a relatively short period function of the
of time. passage as a whole?
B. It was stronger than their commitment A. To contrast the views held by the
to the Socialist party
Suffrage struggle. and by the boycotting women of New
C. It predated the cost-of-living protests York City
that on the cost-of-living issue
Erupted in 1917. B. To analyze the assumptions
D. It coincided with their attempts to underlying opposing
bring more viewpoints within the New York
Women into union organizing. Socialist
E. It explained the popularity of the party of 1917
Socialist C. To provide a historical perspective on
party in New York City. different
Answer: approaches to the resolution of the cost-
------------------------------------------------- ofliving
---------------------------------------- issue.
Q26 D. To chronicle the sequence of events
It can be inferred from the passage that that led
the goal to the New York Socialist party’s
of the boycotting women was the emergence
A. achievement of an immediate as a political power
economic E. To analyze the motivations behind the
outcome Socialist
B. development of a more socialistic party’s involvement in the women’s
society suffrage
C. concentration of widespread movement.
consumer Answer:
protests on the more narrow issue of -------------------------------------------------
food prices -----------------------------------------
D. development of one among a number Q28.
of According to the passage ,most New
different approaches that the women York
wished to employ in combating the high Socialists believed which of the
cost of living. following about
E. attraction of more public interest to the cost-of-living movement?
issues A. It was primarily a way to interest
that the women and the New York women
Socialists in joining the Socialist party.
considered important. B. It was an expedient that was useful
Answer: only
------------------------------------------------- insofar as it furthered other goals.
----------------------------------------- C. It would indirectly result in an
Q27 increase in
the number of women who belonged to
labor unions. Laboratory tests have shown that the
D. It required a long-term commitment kind of oil paint used in these paintings
but actually adjusts to climatic changes quite
Inevitably represented a direct step well. If, as some museum directors
Toward socialism. believe,
E. It served as an effective complement paint is the most sensitive substance in
to these works, then by relaxing the
union organizing. standards
Answer: for temperature and humidity control,
------------------------------------------------- museums can reduce energy costs
---------------------------------------- without risking damage to these
Q29. paintings. Museums would be rash to
The army cutworm moth is a critical relax those
source of fat for many standards, however, since results of
of Yellowstone National Park’s grizzly preliminary tests indicate that gesso, a
bears; they overturn compound
rocks to find them and consuming as routinely used by Renaissance artists to
many as 40,000 apiece help paint adhere to the canvas, is unable
in a single day. to
A. bears; they overturn rocks to find withstand significant variations in
them and consuming humidity.
as many as. In the argument above, the two portions
B. bears; overturning rocks to find the in boldface play which of the following
insects, consuming roles?
up to A. The first is an objection that has been
C. bears, overturning rocks to find them raised against the position taken by the
and they consume argument; the second is the position
as many as. taken by the argument.
D. bears, and they overturn rocks to find B. The first is the position taken by the
them and consume argument; the second is the position that
up to the
E. bears, which overturn rocks to find argument calls into question.
the insects, consuming C. The first is a judgment that has been
as many as. offered in support of the position that the
Answer: argument
------------------------------------------------- calls into question; the second is a
----------------------------------------- circumstance on which that judgment is,
Q30. in part based.
Museums that house Renaissance oil D. The first is a judgment that has been
paintings typically store them in offered in support of the position that the
envbironments argument
that calls into question; the second is that
are carefully kept within narrow margins positon.
of temperature and humidity to inhibit E. The first is a claim that the argument
any calls into question; the second is the
deterioration. position
taken by the argument. C. Pollutants other than nitrogen dioxide
Answer: that are emitted by automobiles have
------------------------------------------------- also
----------------------------------------- been significantly reduced in Donia
Q31 since 1993.
New items developed for automobiles in D. Many Donians who own cars made
the 1997 model year included before 1993 have had catalytic
a safer air bag , which, unlike previous converters
air bags, eliminated the possibility installed in their cars.
that a burst of smoke would appear when E. Most car trips in Donia’s capital city
the bag inflated, and making an are too short for the catalytic converter
already terrified passenger think the car to
was on fire. reach its effective working temperature.
A. inflated, and making. Answer:
B. Inflated, so that it could make. -------------------------------------------------
C. Inflated and made. ----------------------------------------
D. Inflated and make. Q33 to 35
E. Inflated to make. In corporate purchasing,
Answer: competitive scrutiny is typically
------------------------------------------------- limited to suppliers of items that are
----------------------------------------- Line directly related to end products.
Q32 (5) With “indirect” purchases (such as
Nitrogen dioxide is a pollutant emitted computers, advertising, and legal
by automobiles. Catalytic converters, services), which are not directly
devices designed to reduce nitrogen related to production, corporations
dioxide emissions, have been required often favor “supplier partnerships”
in all new cars in Donia since 1993, and (10) (arrangements in which the
as a result, nitrogen dioxide emissions purchaser forgoes the right to
have been significantly reduced pursue alternative suppliers), which
throughout most of the country. Yet can inappropriately shelter suppliers
althouth the proportion of new cars in from rigorous competitive scrutiny
Donia’s capital city has always been (15) that might afford the purchaser
comparatively high, nitrogen dioxide economic leverage. There are two
emissions there have showed only an independent variables—availability
insignificant decline since 1993. of alternatives and ease of changing
Which of the following, if true, most suppliers—that companies should.
helps to explain the insignificant decline (20) use to evaluate the feasibility of
in nitrogen dioxide emissions in Donia’s subjecting suppliers of indirect
capital city? purchases to competitive scrutiny.
A. More of the cars in Donia’s capital This can create four possible
city were made before 1993 than after Situations.
1993. (25) In Type 1 situations, there are
B. The number of new cars sold per year many alternatives and change is
in Donia has declined slightly since 1993 relatively easy. Open pursuit of
alternatives—by frequent cornpetitive
bidding, if possible—will
(30) likely yield the best results. In Which of the following can be inferred
Type 2 situations, where there about
are many alternatives but change supplier partnerships, as they are
is difficult—as for providers of described
employee health-care benefits—it in the passage?
(35) is important to continuously test A. They cannot be sustained unless the
the market and use the results to goods
secure concessions from existing or services provided are available form a
suppliers. Alternatives provide a large number of suppliers.
credible threat to suppliers, even if B. They can result in purchasers paying
(40) the ability to switch is constrained. more
In Type 3 situations, there are few for goods and services than they would
alternatives, but the ability to switch in
without difficulty creates a threat that a competitive-bidding situation.
companies can use to negotiate. C. They typically are instituted at the
(45) concession from existing suppliers. urging of
In Type 4 situations, where there the supplier rather than the purchaser.
are few alternatives and change D. They are not feasible when the goods
is difficult, partnerships may be or
unavoidable/. Services provided are directly related to
------------------------------------------------- The purchasers’ end products
----------------------------------------- E. They are least appropriate when the
Q33. purchasers’ ability to change suppliers is
Which of the following best describes limited.
the Answer:
relation of the second paragraph to the -------------------------------------------------
first? ----------------------------------------
A. The second paragraph offers proof of Q35
an According to the passage, which of the
assertion made in the first paragraph. following
B. The second paragraph provides an factors distinguishes an indirect purchase
explanation for the occurrence of a from
situation described in the first paragraph. other purchases?
C. The second paragraph discusses the A. The ability of the purchasing
application of a strategy proposed in the company
first paragraph. to subject potential suppliers of the
D The second paragraph examines the purchased item to competitive scrutiny
scope of a problem presented in the first B. The number of suppliers of the
paragraph. purchased
F. The second paragraph discusses the item available to the purchasing
contradictions inherent in a relationship company
described in the first paragraph. C. The methods of negotiation that are
Answer: available t other purchasing company.
------------------------------------------------- D. The relationship of the purchased
----------------------------------------- item to
Q34 the purchasing company’s end product
E. The degree of importance of the operations
purchased Answer:
item in the purchasing company’s
business
------------------------------------------------------------------------------------------
Q36
The chemical adenosine is released by brain cells when those cells are active.
Adenosine then binds to more and more sites on cells in certain areas of the
brain, as the total amount released gradually increases during wakefulness.
During sleep, the number of sites to which adenosine is bound decreases. Some
researchers have hypothesized that it is the cumulative binding of adenosine to a
large number of sites that causes the onset of sleep.
Which of the following, if true, provides the most support for the researchers’
hypothesis?
A. Even after long periods of sleep when adenosine is at its lowest concentration
in the brain, the number of brain cells bound with adenosine remains
very large.
B. Caffeine, which has the effect of making people remain wakeful, is known to
interfere with the binding of adenosine to sites on brain cells.
C. Besides binding to sites in the brain, adenosine is known to be involved in
biochemical reactions throughout the body.
D. Some areas of the brain that are relatively inactive nonetheless release
some adenosine.
E. Stress resulting from a dangerous situation can preserve wakefulness even
when brain levels of bound adenosine are high.
Answer:
----------------------------------------------------------------------------------------
Q37.
Though the law will require emissions testing of all diesel vehicles, from
tractor trailers to excursion buses, it will have no effect on sport utility
vehicles, almost all of which are gasoline powered, and will not be subjected
to emissions-control standards as stringent as they are for
diesel-powered vehicles.
A. powered, and will not be subjected to emissions-control standards as
stringent as they are for
B. powered, and therefore not subjected to emissions-control standards
that are as stringent as those of
C. powered and therefore not subject to emissions-control standards as
stringent as those for
D. powered, which are not subject to emissions-control standards as
stringent as they are for
E. powered and therefore they are not subject to emissions-control
standards as stringent as those of
answer:
-----------------------------------------------------------------------------------
Q38
Doctors hope that one day the body’s master cells, called stem
cells, can be directed to grow in organs or tissues appropriate for
transplant, use them to test drugs and potentially toxic chemicals,
and may study them to gain insight into basic human biology.
A. transplant, use them to test drugs and potentially toxic
chemicals, and may study them
B. transplant, using them to test drugs and potentially toxic
chemicals, and studied.
C. Transplant, used to test drugs and potentially toxic chemicals,
and studied.
D. A transplant, use them for testing drugs and potentially toxic
chemicals, and for studying.
E. A transplant, used to test drugs and potentially toxic chemicals,
and may study them.
Answer:
--------------------------------------------------------------------------------------
Q39.
Navigators have known for thousands of years that the ocean
has variable currents, but it is only in the last half century that
a reasonably clear picture has emerged of the patterns and
causes of ocean currents.
A. a reasonably clear picture has emerged of the patterns
and causes of ocean currents.
B. a reasonably clear picture of the patterns of ocean
currents and their causes have emerged.
C. a reasonably clear picture emerged of ocean currents,
their patterns and the causes of them
D. there have emerged a reasonably clear picture of the
patterns of ocean currents and what caused them.
E. there had emerged a reasonably clear picture of the
patterns of ocean currents and their causes.
Answer:
------------------------------------------------------------------------------------------
Q40
For similar cars and drivers, automobile insurance for collision damage has
always cost more in Greatport than in Faimont. Police studies, however,
show that cars owned by Greatport residents are, on average, slightly less
likely to be involved in a collision than cars in Fairmont. Clearly, therefore,
insurance companies are making a greater profit on collision-damage
insurance in Greatport than in Fairmont.
Which of the following is an assumption on which the argument depends?
A. Repairing typical collision damage does not cost more in Greatport
than in Fairmont.
B. There are no more motorists in Greatport than in Fairmont.
C. Greatport residents who have been in a collision are more likely to
report it to their insurance company than Fairmont residents are.
D. Fairmont and Greatport are the cities with the highest collisiondamage
insurance rates.
E. The insurance companies were already aware of the difference in the
like lihood of collisions before the publication of the police reports.
Answer:
-------------------------------------------------------------------------------------
Q41.
Prospecting for gold during the California gold rush was a relatively
easy task, because of erosion, prehistoric glacier movement, and
ancient, gold-bearing riverbeds thrust to the surface by volcanic
activity put gold literally within reach for anybody with a pan or shovel.
A. because of erosion, prehistoric glacier movement,
and ancient, gold-bearing riverbeds thrust to the surface by volcanic activity
put gold literally within reach for
B. because of erosion, prehistoric glacier movement, and volcanic
activity that thrust ancient, gold-bearing riverbeds to the surface,
and putting gold literally within reach of
C. owing to erosion, prehistoric glacier movement, and volcanic activity
that had thrust ancient, gold-bearing riverbeds to the surface,
and putting gold literally within reach of
D. since erosion, prehistoric glacier movement, and volcanic activity
that thrust ancient, gold-bearing riverbeds to the surface, putting
gold literally within reach for
E. since erosion, prehistoric glacier movement, and ancient, goldbearing
riverbeds thrust to the surface by volcanic activity put
gold literally within reach of
Answer:
-----------------------------------------------------------------------------------------------
answer:
DBAAA,DCEDB,BBACB,BDEEC,CCBBD,CCBED,EECBD,BCCAA,E

Verbal E. During which the city was


1 transformed.
Tom Bradley was mayor of Los Angeles Answer:
from 1973 to 1993, an era when the city -------------------------------------------------
had -------------------------------------------------
transformed from a collection of -----
suburban neighborhoods to the second- 2
largest city in According to public health officials, in
the United States. 1998 Massachusetts became the first
A. an era when the city had transformed. state in
B. An era during which the city was which more babies were born to women
transformed. over the age of thirty than under it.
C. An era that transformed it. A. than
D. During which era the city B. than born
transformed. C. than they were
D. than there had been are used is difficult, especially when a
E. than had been born. nonnative agent is introduced, because,
Answer: unlike a chemical pesticide, a biocontrol
------------------------------------------------- agent may adapt in unpredictable ways.
------------------------------------------------- (40) so that it can feed on or otherwise
----- harm
Q3 TO 6 new hosts.
More selective than most chemical -------------------------------------------------
pesticides in that they ordinarily destroy ---------------
only unwanted species, biocontrol. 3
Line agents (such as insects, fungi, and The passage is primarily concerned with
(5) viruses) eat, infect, or parasitize A. explaining why until recently
targeted scientists
plant or animal pests. However, failed to recognize the risks presented
biocontrol agents can negatively affect by biocontrol agents.
nontarget species by, for example, B. emphasizing that biocontrol agents
competing with them for resources: a and
(10) biocontrol agent might reduce the chemical pesticides have more
benefits similarties
conferred by a desirable animal than differences.
species by consuming a plant on which C. suggesting that only certain
the animal prefers to lay its eggs. biocontrol
Another example of indirect negative. agents should be used to control plant
(15) consequcnces occurred in England or animal pasts.
when a virus introduced to control D. arguing that biocontrol agents involve
rabbits reduced the amount of open risks, some of which may not be readily
ground (because large rabbit populations discerned.
reduce the ground cover), in E. suggesting that mishaps involving
(20) turn reducing underground ant nests biocontroll
and triggering the extinction of a blue agents are relatively commonplace.
butterfly that had depended on the Answer:
nests to shelter its offspring. The -------------------------------------------------
paucity of known extinctions or disrup- ---------------------------------------------
(25) tions resulting from indirect 4
interactions The passage suggests that the author
may reflect not the infrequency of such would
mishaps but rather the failure to look be most likely to agree with which of the
for or to detect them: most organisms following statements about the use of
likely to be adversely affected by biocontrol
(30) indirect interactions are of little or agents?
no known commercial value and the A. Biocontrol agent should be used only
events linking a biocontrol agent with in cases
an adverse effect are often unclear. where chemical pesticides have proven
Moreover, determining the potential ineffective or overly dangerous.
(35) risks of biocontrol agents before B. Extinctions and disruptions resulting
they from
the use of biocontrol agents are likely to A. a situation in which a species is less
have increasingly severe commercial vulnerable to biocontrol agents than it
consequences. would have been to chemical pesticides.
C. The use of biocontrol agents does not B. a way in which the introduction of a
require regulation as stringent as that biocontrol
required by the use of chemical agent can affect a nontarget
pesticides. species.
D. The use of biocontrol agents may C. a nonnative agent’s adapting in an
evenfinally unpredictable
supersede the use of chemical pesticides way that results in damage to a
in controlling unwanted species. new host.
E. The risks of using native biocontrol D. The contention that biocontrol agents
agents can
may be easier to predict than the risks of harm nontarget species by competing
using nonnative biocontrol agents. with them for resources
Answer: E. the way in which indirect
------------------------------------------------- consequences
------------------------------------------------- from the use of biocontrol agents are
----- most likely to occurs.
5 Answer:
Which of the following is mentioned in -------------------------------------------------
the passage as an indirect effect of using -------------------------------------------------
a -----
biocontrol agent? 7
A. Reduction of the commercial value of Which of the following most logically
a completes the argument below?
desirable animal species The expansion of large-scale farming in
B. An unintended proliferation of a Africa and Asia has destroyed much of
nontarget the
animal species natural vegetation on which elephants
C. An unforeseen mutation in a target have historically depended, forcing them
species to
D. Diminution of the positive effects turn to cultivated land to satisfy their
conferred enormous appetites. As a result, farmers
by a nontarget animal species. have
E. Competition for resources with a lost millions of dollars worth of crops
largest annually. Yet even if elephant
species. sanctuaries were
Answer: created on a widespread basis to
------------------------------------------------- guarantee elephants sufficient natural
------------------------------------------------- vegetation, the
----- raiding would likely persist, since
6 A. when elephants forage for food, they
The example presented by the author in typically travel in herds.
lines 14-23 most clearly serves to B. Foraging elephants have been known
illustrate to cause substantial damage even to
plants that they do not eat. Part of the proposed increase in state
C. Some of the land where crops have education spending is due to higher
suffered extensive damage from enrollment,
elephants since the number of students in public
has been allowed to return to its natural schools have grown steadily since the
state. mid-1980’s and, at nearly 47 million, are
D. Elephants tend to prefer cultivated at a record high.
crops to wild vegetation as a food A. enrollment, since the number of
source. students in public schools have grown
E. Elephant sanctuaries are usually steadily
created in areas that are rich in the since the mid-1980’s and, at nearly 47
natural million, are at.
vegetation on which elephants have B. Enrollment, with a number of
historically depended. students in public schools growing
Answer: steadily
------------------------------------------------- since the mid-1980’s and, at nearly 47
------------------------------------------------- million, reaching.
----- C. Enrollment: since students in public
8 schools have grown steadily in number
The results of the company’s cost- since the mid-1980’s and, at nearly 47
cutting measures are evident in its million, have reached.
profits, which D. Enrollment: the number of students in
increased five percent during the first public schools has grown steadily since
three months of this year after it fell over the mid-1980’s and, at nearly 47 million,
the has reached.
last two years. E. Enrollment: students in public schools
A. which increased five percent during have grown steadily in number since
the first three months of this year after it the mid-1980’s and , at nearly 47million,
fell are at
B. which had increased five percent Answer:
during the first three months of this year -------------------------------------------------
after it had fallen -------------------------------------------------
C. which have increased five percent -
during the first three months of this year Q10 TO 12
after falling Is it possible to decrease
D. with a five percent increase during inflation without causing a recession
the first three months of this year after and its concomitant increase
falling line in unemployment? The orthodox
E. with a five percent increase during the (5) answer is “no.” whether they
first three months of this year after support the “inertia” theory of
having fallen inflation (that today’s inflation rate
Answer: is caused by yesterday’s inflation,
------------------------------------------------- the state of the economic
------------------------------------------------- (10) cycle, and external influences
- such as import prices) or the
9 “rational expectations” theory
(that inflation is caused by A. It fit the rational expectations theory
workers’ and employers’ expec- of inflation
(15) tations, coupled with a lack of but not the inertia theory of inflation.
credible monetary and fiscal B. It was possible to control without
policies), most economists causing a
agree that tight monetary and recession.
fiscal policies, which cause C. It was easier to control in those
(20) recessions, are necessary to countries by
decelerate inflation. They point applying tight monetary and fiscal
out that in the 1980’s, many policies
European countries and the than it would have been elsewhere.
United States conquered high D. It was not caused by workers’ and
(25) (by these countries’ standards) employers’
inflation, but only by applying tight expectations.
monetary and fiscal policies that E. It would not necessarily be considered
sharply increased unemployment. high
Nevertheless, some govern- elsewhere.
(30) ments’ policymakers insist that Answer:
direct controls on wages and -------------------------------------------------
prices, without tight monetary and -------------------------------------------------
fiscal policies, can succeed in -----
decreasing inflation. Unfortu- 11
(35) nately, because this approach Which of the following, if true, would
fails to deal with the underlying most strengthen
causes of inflation, wage and The author’s conclusion about the use of
price controls eventually collapse, wage and
the hitherto-repressed price controls?
(40) inflation resurfaces, and in the A. Countries that repeatedly use wage
meantime, though the policymakers and price
succeed in avoiding a controls tend to have lower long-term
recession, a frozen structure of economic
relative prices imposes distor- growth rates than do other countries.
(45) tions that do damage to the B. Countries that have extremely high
economy’s prospects for longterm inflation
growth. frequently place very stringent controls
------------------------------------------------- on
------------------------------------------------- wages and prices in an attempt to
10 decrease
The passage suggests that the high the inflation.
inflation in the C. Some countries have found that the
United States and many European use of wage
countries in the and price controls succeeds in
1980’s differed from inflation elsewhere decreasing
in which of inflation but also causes a recession.
the following ways? D. Policymakers who advocate the use
of wage
and price controls believe that these D. Who, having had no formal medical
controls training and struggled against
will deal with the underlying causes of overwhelming odds, became
inflation. E. Who had no formal medical training,
E. Policymakers who advocate the use o struggled against overwhelming odds to
wage become
and price controls are usually more Answer:
concerned 14
about long-term economic goals than In 1983 Argonia’s currency, the argon,
about underwent a reduction in value relative
short-term economic goals to the
answer: world’s strongest currencies. This
------------------------------------------------- reduction resulted in a significant
------------------------------------------------- increase in
- Argonia’s exports over 1982 levels. In
12 1987 a similar reduction in the value of
The primary purpose of the passage is to the
A. apply two conventional theories. argon led to another increase in
B examine a generally accepted position Argonia’s exports. Faced with the need
C. support a controversial policy to increase
D. explain the underlying causes of a exports yet again, Argonia’s finance
phenomenon minister has proposed another reduction
E. propose an innovative solution in the
Answer: value of the argon.
------------------------------------------------- Which of the following, if true, most
------------------------------------------------- strongly supports the prediction that the
----- finance
13 minister’s plan will not result in a
Vivien Thomas, who had no formal significant increase in Argonia’s exports
medical training, in struggling against next year?
overwhelming odds, he became a cardiac A. The value of the argon rose sharply
surgeon and eventually to receive an last year against the world’s strongest
honorary doctorate from Johns Hopkins currencies.
University. B. In 1988 the argon lost a small amount
A. who had no formal medical training, of its value, and Aronian exports rose
in struggling against over whelming slightly in 1989.
odds, C. The value of Argonia’s exports was
he became lower last year than it was the year
B. Having had no formal medical before.
training, in struggling against D. All of Argonia’s export products are
overwhelming made by factories that were operating at
odds to become full capacity last year, and new factories
C. Who, having no formal medical would take years to build.
training, he struggled against E. Reductions in the value of the argon
overwhelming have almost always led to significant
odds in becoming
reductions in the amount of goods and been reduced to levels typical of healthy
services that Argonians purchase from lungs.
abroad. D. The lungs of people who suffer from
Answer: cystic fibrosis are unable to fight off
------------------------------------------------- harmful bacteria even when the salt
------------------------------------------------- concentration is reduced to levels typical
- of healthy lungs.
15. E. The salt concentration in the airway-
Healthy lungs produce a natural surface fluid of people whose lungs
antibiotic that protects them from produce lower-than-average amounts of
infection by the antibiotic is generally much lower
routinely killing harmful bacteria on than that typical of healthy lungs.
airway surfaces. People with cystic Answer:
fibroses, 16
however, are unable to fight off such which of the following most logically
bacteria, even though their lungs completes the argument below?
produce normal According to promotional material
amounts of the antibiotic. Since the fluid published by the city of Springfield,
on airway surfaces in the lungs of people more tourists
with cystic fibrosis bas an abnormally stay in hotels in Springfield than stay in
high salt concentration, scientists the neighboring city of Harristown. A
hypothesize brochure from the largest hotel in
that in high salt environments the Harristown claims that more tourists stay
antibiotic becomes ineffective at killing in that
harmful hotel than stay in the Royal Arms Hotel
bacteria. in Springfield. If both of these sources
Which of the following, if it were are
obtained as an experimental result, accurate, however, the “Report on
would most Tourism” for the region must be in error
decisively undermine the scientists’ in stating
hypothesis? that .
A. Healthy lungs in which the salt A. the average length of stay is longer at
concentration of the airway-surface fluid the largest hotel in Harristown than it is
has at the Royal Arms Hotel.
been substantially increased are able to B. There is only one hotel in Harristown
reestablish their normal salt that is larger than the Royal Arms Hotel.
concentration within a relatively short C. More tourists stay in hotels in
period of time. Harristown than stay in the Royal Arms
B. The antibiotic produced by the lungs Hotel.
is effective at killing harmful bacteria D. The Royal Arms hotel is the largest
even when salt concentrations are below hotel in Springfield
levels typical of healthy lungs. E. The royal arms hotel is the only hotel
C. The salt concentration of the airway- in Springfield.
surface fluid in the lungs of people who Answer:
suffer from cystic fibrosis tends to return
to its former high levels after having
------------------------------------------------- Less. On average, than do comparable
------------------------------------------------- households shoes microwave oven is
----- other-
17 Wise similar but has a built-in clock.
The yield of natural gas from Norway’s Which of the following is an assumption
Troll gas field is expected to increase on which the argument depends?
annually until the year 2005 and then to A. Households that do not have a
stabilize at six billion cubic feet a day , microwave oven use less energy per
which year, on
will allow such an extraction rate at least average, than do households that have a
for 50 years’ production microwave oven.
A. 2005 and then to stabilize at six B. Microwave ovens with a built-in
billion cubic feet a day, which clock do not generally cost more to buy
will allow such an extraction rate at least than
for microwave ovens without a built-in
B. 2005 and then to stabilize at six clock.
billion cubic feet a day, an C. All households that have a microwave
extraction rate that will allow at least oven also have either a gas oven or a
C. 2005 and then stabilizing at six billion Conventional electric oven.
cubic feet a day, with D. Households whose microwave oven
such an extraction rate at the least does not have a built-in clock are no
allowing more
D. 2005, then stabilizing at six billion likely to have a separate electric clock
cubic feet a day, allowing plugged in than households whose
such an extraction rate for at least microwave oven has one.
E. 2005, then stabilizing at six billion E. There are more households that have
cubic feet a day, which will a microwave oven with a built-in clock
allow such an extraction rate for at least than there are households that have a
answer: microwave oven without a built-in clock.
------------------------------------------------- Answer:
------------------------------------------------- -------------------------------------------------
----- -------------------------------------------------
Q 18: -----
Most household appliances use Q 19:
electricity only when in use. Many In archaeology, there must be a balance
microwave ovens, between explanation
However, have built-in clocks and so use of the value and workings of
some electricity even when they are not archaeology, revealing the mysteries
in of past and present cultures, and to
Use. The clocks each consume about 45 promote respect for
kilowatt-hours per year. Therefore, archaeological sites.
house A. between explanation of the value and
Holds whose microwave oven has no workings of archaeology,
built-in clock use 45 kilowatt-hours per revealing the mysteries of past and
year present cultures,
and to promote
B. among explaining the value and and the sub-languages or dialects within
workings of archaeology, it, but those who have tried
revealing the mysteries of past and to count typically have found about five
present cultures, thousand.
and promoting A. and the sub-languages or dialects
C. between explaining the value and within it, but those who have
workings of archaeology, tried to count typically have found
revealing the mysteries of past and B. and the sub-languages or dialects
present cultures, within them, with those who
and when promoting have tried counting typically finding
D. among explaining the value and C. and the sub-languages or dialects
workings of archaeology, the within it, but those who have
revelation of the mysteries of past and tried counting it typically find
present cultures, D. or the sub-languages or dialects
and to promote within them, but those who tried
E. between explaining archaeology’s to count them typically found
value and workings, in the E. or the sub-languages or dialects
revealing the mysteries of past and within them, with those who have
present cultures, tried to count typically finding
and in promoting answer:
answer: -------------------------------------------------
------------------------------------------------- -------------------------------------------------
------------------------------------------------- -----
----- Q 22:
Q 20: Often patients with ankle fractures that
According to a 1996 survey by the are stable, and thus do not
National Association of College and require surgery, are given follow-up x-
University Business Officers, more than rays because their orthopedists
three times as many independent are concerned about possibly having
institutions of higher education charge misjudged the stability of the fracture.
tuition and fees of under $8,000 a When a number of follow-up x-rays
year than those that charge over $16,000. were reviewed, however, all
A. than those that charge the fractures that had initially been
B. than are charging judged stable were found to have
C. than to charge healed correctly. Therefore, it is a waste
D. as charge of money to order follow-up
E. as those charging x-rays of ankle fracture initially judged
answer: stable.
------------------------------------------------- Which of the following, if true, most
------------------------------------------------- strengthens the argument?
----- A. Doctors who are general practitioners
Q 21: rather than orthopedists are
Nobody knows exactly how many less likely than orthopedists to judge the
languages there are in the world, stability of an ankle fracture
partly because of the difficulty of correctly.
distinguishing between a language
B. Many ankle injuries for which an E. extremes of temperature,
initial x-ray is ordered are revealed precipitation, and storminess, which are
by the x-ray not to involve any fracture the most obvious effects, are those to
of the ankle. impact the most on people
C. X-rays of patients of many different answer:
orthopedists working in several -------------------------------------------------
hospitals were reviewed. -------------------------------------------------
D. The healing of ankle fractures that -----
have been surgically repaired is Q 24:
always checked by means of a follow-up Many winemakers use cork stoppers; but
x-ray. cork stoppers can leak, crumble, or
E. Orthopedists routinely order follow- become
up x-rays for fractures of bone moldy, so that those winemakers must
other than ankle bones. often discard a significant proportion of
Answer: their
------------------------------------------------- inventory of bottled wine. Bottlemaster
------------------------------------------------- plastic stoppers, which cannot leak,
Q 23: crumble,
Discussion of greenhouse effects has or mold, have long been available to
usually focused on whether the winemakers, at a price slightly higher
Earth would warm and by how much, than that
but climatologists have indicated all of traditional cork stoppers. Cork prices,
along that the most obvious effects, and however, are expected to rise
those that would have the largest dramatically in
impact on people, would be extremes of the near future. Clearly, therefore,
temperature, precipitation, winemakers who still use cork but
and storminess. wish to keep
A. the most obvious effects, and those production costs from rising will be
that would have the largest forced to reconsider plastic stoppers.
impact on people, would be extremes of And
temperature, since the wine-buying public’s
precipitation, and storminess association of plastic stoppers with poor-
B. the effects that are the most obvious quality wine
ones, extremes of temperature, is weakening, there is an excellent
precipitation, and storminess, would be chance that the Bottlemaster plastic
those impacting stopper
the most on people will gain an increased share of the
C. those effects to have the largest marked for wine-bottle stoppers.
impact on people, extremes of In the argument given, the two portions
temperature, precipitation, and in boldface play which of the following
storminess, are what are the roles?
most obvious effects A. The first is a judgment that has been
D. extremes of temperature, advanced in support of a position that
precipitation, and storminess, the most the
obvious effects, that they would have the argument opposes; the second is the
largest impact on people main conclusion of the argument.
B. The first is a judgment that has been with Trotter, Washington’s militant
advanced in support of a position that opponent, less for ideological reasons
the (25) than because Trotter had described
argument opposes; the second is a to him Washington’s efforts to silence
conclusion drawn in order to support the those in the African American press
main conclusion of the argument. who opposed Washington’s positions.
C. The first is the main conclusion of the (30) reflected not a change in his long-
argument; the second provides evidence term
in support of that main conclusion. goals but rather a pragmatic response
D. The first is the main conclusion of the in the face of social pressure:
argument; the second is a restatement government officials had threatened
of that main conclusion. African American journalists with
E. The first is a conclusion drawn in (35) censorship if they continued to
order to support the main conclusion of voice
the grievances. Furthermore, Du Bois
argument; the second is that main believed that African Americans’
conclusion. contributions to past war efforts had
Answer: brought them some legal and political
------------------------------------------------- (40) advances. Du Bois’
------------------------------------------------- accommodationism
----- did not last, however. Upon
Q 25 learning of systematic discrimination
In a 1918 editorial, W.E.B. Du Bois experienced by African Americans
advised African Americans to stop in the military, he called on them to
agitating for equality and to proclaim “return fighting” from the war.
Line their solidarity with White -------------------------------------------------
Americans -------------------------------------------------
(5) for the duration of the First World -----
War. Q 25:
The editorial surprised many African According to the passage, which of the
Americans who viewed Du Bois as following is true of the strategy that Du
an uncompromising African American Bois’
leader and a chief opponent of the 1918 editorial urged African Americans
(10) accommodationist tactics urged by to
Booker, T.Washington. In fact, adopt during the First World War?
however, Du Bois often shifted positions A. It was a strategy that Du Bois had
along the continuum between consistently
Washington and confrontationists rejected in the past.
(15) such as William Trotter. In 1895, B. It represented a compromise between
when Washington called on African Du Bois’ own views and those of Trotter.
Americans to concentrate on improving C. It represented a significant
their communities instead of opposing redefinition of
discrimination and agitating for the long-term goals Du Bois held prior
(20) political rights, Du Bois praised to the war.
Washington’s speech. In 1903, D. It was advocated by Du Bois in
however, Du Bois aligned himself response
to his recognition of the discrimination B. It underwent a shift in 1903 for
faced by African Americans during reasons
the war. Other than Du Bois’ disagreement with
E. It was advocated by Du Bois in part Washington’s accommodationist views.
because of his historical knowledge C. It underwent a shift as Du Bois made
of gains African Americans had made a
during past wars. long-term commitment to the strategy
Answer: of accommodation.
------------------------------------------------- D. It remained consistently positive even
------------------------------------------------- Though Du Bois disagreed with
----- Washington’s efforts to control the
26. African American press.
The passage is primarily concerned with E. It was shaped primarily by Du Bois’
A. identifying historical circumstances appreciation of Washington’s pragmatic
that approach to the advancement of the
led Du Bois to alter his long-term goals. interests of African Americans.
B. defining “accommodationism” and Answer:
showing -------------------------------------------------
how Du Bois used this strategy to -------------------------------------------------
achieve certain goals -----
C. accounting for a particular position 28.
adopted by Du Bois during the First Unlike most severance packages, which
World War. require workers to stay
D. contesting the view that Du Bois was until the last day scheduled to collect,
significantly workers at the automobile
influenced by either Washington company are eligible for its severance
or Trotter. package even if they find
E. assessing the effectiveness of a a new job before they are terminated.
strategy A. the last day scheduled to collect,
that Du Bois urged African Americans to workers at the automobile
adopt. company are eligible for its severance
Answer: package.
------------------------------------------------- B. the last day they are scheduled to
------------------------------------------------- collect, workers are
----- eligible for the automobile company’s
27. severance package
The passage indicates which of the C. their last scheduled day to collect, the
following automobile company
about Du Bois’ attitude toward offers its severance package to workers.
Washington? D. their last scheduled day in order to
A. It underwent a shift during the First collect, the automobile
World company’s severance package is
War as Du Bois became more available to workers.
sympathetic E. the last day that they are scheduled to
with Trotter’s views. collect, the automobile
company’s severance package is Answer:
available to workers.
-------------------------------------------------------------------------------------------------------
29.
Plant scientists have been able to genetically engineer vegetable seeds
to produce crops that are highly resistant to insect damage. Although
these seeds currently cost more than conventional seeds, their cost is
likely to decline. Moreover, farmers planting them can use far less pesticide,
and most consumers prefer vegetables grown with less pesticide,
therefore, for crops for which these seeds can be developed, their use
is likely to become the norm.
which of the following would be most useful to know in evaluating the
argument above?
A. Whether plant scientists have developed insect-resistant seeds
For every crop that is currently grown commercially
B. Whether farmers typically use agricultural pesticides in larger
Amounts than is necessary to prevent crop damage.
C. Whether plants grown from the new genetically engineered seeds
Can be kept completely free of insect damage.
D. Whether seeds genetically engineered to produce insect-resistant
crops generate significantly lower per acre crop yields than do
currently used seeds.
E. Whether most varieties of crops currently grown commercially have
Greater natural resistance to insect damage than did similar varieties
in the past.
Answer:
---------------------------------------------------------------------------------------------------
----
Q30 TO 33 (15) when the seismologist Kiyoo
In most earthquakes the Earth’s Wadati
crust cracks like porcelain, Stress convincingly demonstrated their
builds up until a fracture forms at a existence.
line depth of a few kilometers and the Instead of comparing the arrival
crust times of seismic waves at different
(5) slips to relieve the stress. Some locations, as earlier researchers had
earthquakes, however, take place (20) done, Wadati relied on a time
hundreds difference
of kilometers down in the Earth’s between the arrival of primary(P)
mantle, where high pressure makes waves and the slower secondary(S)
rock so ductile that it flows instead of waves. Because P and S waves
(10) cracking, even under stress severe travel at different but fairly constant
enough to deform it like putty. How can (25) speeds, the interval between their
there be earthquakes at such depths? arrivals increases in proportion to the
That such deep events do occur distance from the earthquake focus,
has been accepted only since 1927. or initial rupture point.
For most earthquakes, wadati dis-
(30) covered, the interval was quite short occur would be most weakened if which
near the epicenter; the point on the of
surface the following were discovered to be
where shaking is strongest. For true?
a few events, however, the delay was A. Deep events are far less common than
long even at the epicenter. Wadati saw shallow events.
(35) a similar pattern when he analyzed B. Deep events occur in places other
data than
on the intensity of shaking. Most where crustal plates meet.
earthquakes C. Mantle rock is more ductile at a depth
had a small area of intense of
shaking, which weakened rapidly with several hundred kilometers than it is
increasing distance from the epicenter. at 50 kilometers.
(40) but others were characterized by a D. The speeds of both P and S waves are
lower peak intensity, felt over a slightly greater than previously thought.
broader area. Both the P-S intervals E. Below 650 kilometers earthquakes
and the intensity patterns suggested cease
two kinds of earthquakes: the more to occur.
(45) common shallow events, in which Answer:
the -------------------------------------------------
focus lay just under the epicenter, and -------------------------------------------------
deep events, with a focus several -----
hundred kilometers down. 31.
The question remained: how can information presented in the passage
(50) such quakes occur, given that suggests
mantle that, compared with seismic activity
rock at a depth of more than 50 at the epicenter of a shallow event,
kilometers seismic
is too ductile to store enough activity at the epicenter of a deep event
stress to fracture? Wadati’s work is
suggested characterized by
that deep events occur in areas A. shorter P-S intervals and higher peak
(55) (now called Wadati-Benioff zones) intensity
where one crustal plate is forced under B. shorter P-S intervals and lower peak
another and descends into the mantle. intensity
The descending rock is substantially C. longer P-S intervals and similar peak
cooler than the surrounding mantle and intensity
(60) hence is less ductile and much more D. longer P-S intervals and higher peak
liable to fracture. intensity
------------------------------------------------- E. longer P-S intervals and lower peak
------------------------------------------------- intensity.
----- Answer:
30. -------------------------------------------------
The author’s explanation of how deep -------------------------------------------------
events -----
32.
The passage supports which of the -------------------------------------------------
following -------------------------------------------------
Statements about the relationship -----
between the 33.
epicenter and the focus of an The passage suggests that which of the
earthquake? following must take place in order for
(A)P waves originate at the focus and any
S waves originate at the epicenter. earthquake to occur?
(B) In deep events the epicenter and the 1.Stress must build up.
focus are reversed. 2.Cool rock must descend
(C) In shallow events the epicenter and into the mantle.
the 3.A fracture must occur
focus coincide (A)1 only
(D)In both deep and shallow events the (B) 2 only
Focus lies beneath the epicenter (C) 3 only
(E) The epicenter is in the crust, whereas (D)1 and 3 only
the focus is in the mantle. (E) 1, 2, and 3
Answer: answer:
-------------------------------------------------------------------------------------------------------
34.
Studying the fruit fly, a household nuisance but a time-honored experimental
subject, has enabled the secrets of how embryos develop to begin to be
unraveled by scientists,
A. Studying the fruit fly, a household nuisance but a time-honored experimental
subject, has enabled the secrets of how embryos develop to
begin to be unraveled by scientists.
B. By the study of the fruit fly, a household nuisance and also a timehonored
experimental subject, it was possible for the secrets of how
embryos develo0p to begin to be unraveled by scientists.
C. By studying a household nuisance but a time-honored experimental subject,
the fruit fly enabled scientists to begin to unravel the secrets of how
embryos develop.
D. By studying the fruit fly, a household nuisance and also a time-honored
experimental subject, the secrets of how embryos develop are beginning
to be unraveled by scientists.
E. The study of the fruit fly, a household nuisance but a time-honored experimental
subject, has enabled scientists to begin to unravel the secrets of
how embryos develop.
Answer:
35
The milk of many mammals contains cannabinoids, substances that are known to
stimulate certain receptors in the brain. To investigate the function of cannabinoids,
researchers injected newborn mice with a chemical that is known to block
cannabinoides from reaching their receptors in the brain. The injected mice showed
far less interest in feeding than normal newborn mice do. Therefore, cannabinoids
probably function to stimulate the appetite.
Which of the following is an assumption on which the argument depends?
A. Newborn mice do not normally ingest any substance other than their
mothers’ milk.
B. Cannabinoids are the only substances in mammals’ milk that stimulate the
appetite.
C. The mothers of newborn mice do not normally make any effort to encourage their
babies to feed.
D. The milk of mammals would be less nutritious if it did not contain cannabinoids.
E. The chemical that blocks cannabinoids from stimulating their brain receptors does
not independently inhibit the appetite.
Answer:
-------------------------------------------------------------------------------------------------------
36.
The Industrial Revolution, making it possible to mass-produce
manufactured goods, was marked by their use of new machines,
new energy sources, and new basic materials.
A. making it possible to mass-produce manufactured goods,
was marked by their use of
B. making possible the mass production of manufactured goods,
marked by the use of
C. which made it possible that manufactured goods were
mass-produced, was marked by their using
D. which made possible the mass production of manufactured
goods, was marked by the use of
E. which made the mass production of manufactured goods
possible and was marked by using.
Answer:
-------------------------------------------------------------------------------------------------------
37.
Fish currently costs about the same at seafood stores throughout Eastville and its
surrounding suburbs. Seafood stores buy fish from the same wholesalers and at the
same prices, and other business expenses have also been about the same. But new tax
breaks will substantially lower the cost of doing business within the city. Therefore, in
the future, profit margins will be higher at seafood stores within the city than at
suburban seafood stores.
For the purposes of evaluating the argument, it would be most useful to know
whether.
(A)more fish wholesalers are located within the city than in the surrounding suburbs.
(B) Any people who currently own seafood stores in the suburbs surrounding Eastville
will relocate their businesses nearer to the city
(C) The wholesale price of fish is likely to fall in the future
(D)Fish has always cost about the same at seafood stores throughout Eastville and its
surrounding suburbs.
(E) Seafood stores within the city will in the future set prices that are lower than those
at suburban seafood stores.
Answer:
-------------------------------------------------------------------------------------------------------
38.
in January 1994 an oil barge ran aground off the coast of San
Juan, Puerto Rico, leaking its cargo of 750000 gallons into the
ocean, while causing the pollution of the city’s beaches.
A. leaking its cargo of 750000 gallons into the ocean, while causing the pollution of
B. with its cargo of 750000 gallons leaking into the ocean, and it polluted
C. and its cargo of 750000 gallons leaked into the ocean, polluting
D. while it leaked its cargo of 750000 gallons into the ocean and caused the pollution
of
E. so that its cargo of 750000 gallons leaked into the ocean, and they were polluting.
Answer:
-------------------------------------------------------------------------------------------------------
39.
Japan’s abundant rainfall and the typically mild temperature throughout most of the
country have produced a lush vegetation cover and, despite the mountainous terrain
and generally poor soils, it has made possible the raising of a variety of crops.
A. it has made possible the raising of
B. has made it possible for them to raise
C. have made it possible to raise
D. have made it possible for raising
E. thus making it possible for them to raise
answer:
----------------------------------------------------------------------------------
40.
Scientists typically do their most creative work before the age of forty. It is commonly
thought that this happens because aging by itself brings about a loss of creative
capacity. However, studies show that a disproportionately large number of the
scientists who produce highly creative work beyond the age of forty entered their
field at an older age than is usual. Since by the age of forty the large majority of
scientists have been working in their field for at least fifteen years, the studies’ finding
strongly suggests that the real reason why scientists over forty rarely produce highly
creative work is not that they have simply aged but rather that they generally have
spent too long in a given field.
In the argument given, the two portions in boldface play which of the following roles?
A. The first is the position that the argument as a whole opposes; the second is an
objection that has been raised against a position defended in the argument.
B. the first is a claim that has been advanced in support of a position that the
argument opposes; the second is a finding that has been used in support of that
position.
C. The first is an explanation that the argument challenges; the second is a finding
that has been used in support of that explanation.
D. The first is an explanation that the argument challenges; the second is a finding on
which that challenge is based.
E. The first is an explanation that the argument defends; the second is a finding that
has been used to challenge that explanation.
Answer:
-------------------------------------------------------------------------------------------------------
41.
In California, a lack of genetic variation in the Argentine ant has allowed the species
to spread widely; due to their being so genetically similar to one another, the ants
consider all their fellows to be a close relative and thus do not engage in the kind of
fierce intercolony struggles that limits the spread of this species in its native
Argentina.
A. due to their being so genetically similar to one another, the ants consider
all their fellows to be a close relative and thus do not engage in the kind
of fierce intercolony struggles that limits.
B. Due to its being so genetically similar the ant considers all its fellows to be
a close relative and thus does not engage in the kind of fierce intercolnny
struggles that limit.
C. Because it is so genetically similar, the ant considers all its fellows to be
close relatives and thus does not engage in the kind of fierce intercolony
struggles that limits.
D. Because they are so genetically similar to one another, the ants consider
all their fellows to be close relatives and thus do not engage in the kind
of fierce intercolony struggles that limit.
E. Because of being so genetically similar to one another, the ants consider
all their fellows to be a close relative and thus do not engage in the kind
of fierce intercolony struggles that limits.
Answer:
-----------------------------------------------------------------------------------------
answer:
baded, bdcde, abedd, ebdbe, abaee, cbedb, eddee, deccd,d

1
Math Section
******************* If 2 different representatives are
Q1: to be selected at random from a
A student’s average (arithmetic group of 10 employees
mean) test score on 4 tests is and if p is the probability that
78. What must be the both representatives selected
student’s score on a 5th test for will be women, is p >
the student’s average score on 2
the 5 tests to be 80? 1
A. 80 ?
B. 82 (1) More than
C. 84 2
D. 86 1
E. 88 of the 10 employees are
Answer: women.
*******************
Q2:
(2) The probability that both E. Statements (1) and (2)
representatives selected will be TOGETHER are NOT sufficient.
men is less than 2
10 Answer:
1 *******************
. Q4:
A. Statement (1) ALONE is For a recent play performance,
sufficient, but statement (2) the ticket prices were $25 per
alone is not sufficient. adult and $15 per child. A
B. Statement (2) ALONE is total of 500 tickets were sold for
sufficient, but statement (1) the performance. How many of
alone is not sufficient. the tickets sold were for
C. BOTH statements TOGETHER adults?
are sufficient, but NEITHER (1) Revenue from ticket sales for
statement ALONE is this performance totaled
sufficient. $10,500.
D. EACH statement ALONE is (2) The average (arithmetic
sufficient. mean) price per ticket sold was
E. Statements (1) and (2) $21.
TOGETHER are NOT sufficient. A. Statement (1) ALONE is
Answer: sufficient, but statement (2)
******************* alone is not sufficient.
Q3: B. Statement (2) ALONE is
Material A costs $3 per kilogram, sufficient, but statement (1)
and material B costs $5 per alone is not sufficient.
kilogram. If 10 kilograms C. BOTH statements TOGETHER
of material K consists of x are sufficient, but NEITHER
kilograms of material A and y statement ALONE is
kilograms of material B, is x > sufficient.
y? D. EACH statement ALONE is
(1) y > 4 sufficient.
(2) The cost of the 10 kilograms E. Statements (1) and (2)
of material K is less than $40. TOGETHER are NOT sufficient.
A. Statement (1) ALONE is Answer:
sufficient, but statement (2) *******************
alone is not sufficient. Q5:
B. Statement (2) ALONE is From a group of 3 boys and 3
sufficient, but statement (1) girls, 4 children are to be
alone is not sufficient. randomly selected. What is the
C. BOTH statements TOGETHER probability that equal numbers
are sufficient, but NEITHER of boys and girls will be
statement ALONE is selected?
sufficient. A.
D. EACH statement ALONE is 10
sufficient. 1
B.
9 A. 72
4 B. 74
C. C. 75
2 D. 78
1 E. 80
D. Answer:
5 *******************
3 Q9:
E. If x and y are positive integers
3 and 1 + x + y + xy = 15, what is
2 the value of x + y?
Answer: A. 3
******************* B. 5
Q6: C. 6
A certain company assigns D. 8
employees to offices in such a E. 9
way that some of the offices Answer:
can be empty and more than *******************
one employee can be assigned Q10:
to an office. In how many If Ann saves x dollars each week
ways can the company assign 3 and Beth saves y dollars each
employees to 2 different offices? week, what is the total
A. 5 amount that they save per
B. 6 week?
C. 7 (1) Beth saves $5 more per
D. 8 week than Ann saves per week.
E. 9 (2) It takes Ann 6 weeks to save
Answer: the same amount that Beth
******************* saves in 5 weeks.
3 A. Statement (1) ALONE is
Q7: sufficient, but statement (2)
√[2√63 + 2/(8+3√7)] = alone is not sufficient.
A. 8 + 3√7 B. Statement (2) ALONE is
B. 4 + 3√7 sufficient, but statement (1)
C. 8 alone is not sufficient.
D. 4 C. BOTH statements TOGETHER
E. √7 are sufficient, but NEITHER
Answer: statement ALONE is
******************* sufficient.
Q8: D. EACH statement ALONE is
The infinite sequence a1, a2,…, sufficient.
an,… is such that a1 = 2, a2 = -3, E. Statements (1) and (2)
a3 = 5, a4 = -1, and an = TOGETHER are NOT sufficient.
an-4 for n > 4. What is the sum of Answer:
the first 97 terms of the *******************
sequence? Q11:
4 What is the value of the integer
If x is a positive integer, is the n?
remainder 0 when 3x + 1 is (1) n(n + 2) = 15
divided by 10? (2) (n + 2)n = 125
(1) x = 4n + 2, where n is a A. Statement (1) ALONE is
positive integer. sufficient, but statement (2)
(2) x > 4 alone is not sufficient.
A. Statement (1) ALONE is B. Statement (2) ALONE is
sufficient, but statement (2) sufficient, but statement (1)
alone is not sufficient. alone is not sufficient.
B. Statement (2) ALONE is C. BOTH statements TOGETHER
sufficient, but statement (1) are sufficient, but NEITHER
alone is not sufficient. statement ALONE is
C. BOTH statements TOGETHER sufficient.
are sufficient, but NEITHER D. EACH statement ALONE is
statement ALONE is sufficient.
sufficient. E. Statements (1) and (2)
D. EACH statement ALONE is TOGETHER are NOT sufficient.
sufficient. Answer:
E. Statements (1) and (2) *******************
TOGETHER are NOT sufficient. Q14:
Answer: If a code word is defined to be a
******************* sequence of different letters
Q12: chosen from the 10 letters
If r, s, and w are positive A, B, C, D, E, F, G, H, I, and J,
numbers such that w = 60r + what is the ratio of the number
80s and r + s = 1, is w < 70? of 5-letter code words to
(1) r > 0.5 the number of 4-letter code
(2) r > s words?
A. Statement (1) ALONE is A. 5 to 4
sufficient, but statement (2) B. 3 to 2
alone is not sufficient. C. 2 to 1
B. Statement (2) ALONE is 5
sufficient, but statement (1) D. 5 to 1
alone is not sufficient. E. 6 to 1
C. BOTH statements TOGETHER Answer:
are sufficient, but NEITHER *******************
statement ALONE is Q15:
sufficient. Which of the following fractions
D. EACH statement ALONE is has a decimal equivalent that is
sufficient. a terminating decimal?
E. Statements (1) and (2) A.
TOGETHER are NOT sufficient. 189
Answer: 10
******************* B.
Q13: 196
15 3
C. 2
225 were basic and the rest were
16 deluxe. If it takes
D. 5
144 7
25 as many
E. hours to produce a deluxe
128 stereo as it does to produce a
39 basic stereo, then the number of
Answer: hours it took to produce the
******************* deluxe stereos last month was
Q16: what fraction of the total
For any integer k greater than 1, number of hours it took to
the symbol k* denotes the produce all the stereos?
product of all the fractions of 6
the form A.
t 17
1 7
, where t is an integer between 1 B.
and k, inclusive. What is the 31
value of * 14
*
C.
4 15
5 7
? D.
A. 5 35
B. 17
4 E.
5 2
C. 1
5 Answer:
4 *******************
D. Q18:
4 A certain machine produces
1 1,000 units of product P per
E. hour. Working continuously at
5 this constant rate, this machine
1 will produce how many units of
Answer: product P in 7 days?
******************* A. 7,000
Q17: B. 24,000
Company S produces two kinds C. 40,000
of stereos: basic and deluxe. Of D. 100,000
the stereos produced by E. 168,000
Company S last month, Answer:
******************* (2) During the show, there was
Q19: an average (arithmetic mean) of
If 5x – 5x - 3 = (124)(5y), what is y 20 unoccupied seats
in terms of x? per row for the back 15 rows.
A. x A. Statement (1) ALONE is
B. x - 6 sufficient, but statement (2)
C. x - 3 alone is not sufficient.
D. 2x + 3 B. Statement (2) ALONE is
E. 2x + 6 sufficient, but statement (1)
Answer: alone is not sufficient.
******************* C. BOTH statements TOGETHER
Q20: are sufficient, but NEITHER
Lines k and m are parallel to each statement ALONE is
other. Is the slope of line k sufficient.
positive? D. EACH statement ALONE is
(1) Line k passes through the sufficient.
point (3, 2). E. Statements (1) and (2)
(2) Line m passes through the TOGETHER are NOT sufficient.
point (-3, 2). Answer:
A. Statement (1) ALONE is *******************
sufficient, but statement (2) Q22:
alone is not sufficient. The number of defects in the
B. Statement (2) ALONE is first five cars to come through a
sufficient, but statement (1) new production line are 9,
alone is not sufficient. 7, 10, 4, and 6, respectively. If
C. BOTH statements TOGETHER the sixth car through the
are sufficient, but NEITHER production line has either 3, 7,
statement ALONE is or
sufficient. 12 defects, for which of theses
D. EACH statement ALONE is values does the mean number
sufficient. of defects per car for the
E. Statements (1) and (2) first six cars equal the median?
TOGETHER are NOT sufficient. I. 3
Answer: II. 7
7 III. 12
******************* A. I only
Q21: B. II only
Theater M has 25 rows with 27 C. III only
seats in each row. How many of D. I and III only
the seats were occupied E. I, II, and III
during a certain show? Answer:
(1) During the show, there was *******************
an average (arithmetic mean) of Q23:
10 unoccupied seats +xyz
per row for the front 20 rows. dm
en
f A. 4
8 B. 7
The figure above shows two C. 8
entries, indicated by m and n, in D. 11
an addition table. What is E. 12
the value of n + m? Answer:
(1) d + y = -3 *******************
(2) e + z = 12 Q26:
A. Statement (1) ALONE is In the xy-plane, the line with
sufficient, but statement (2) equation ax + by + c = 0, where
alone is not sufficient. abc ≠ 0, has slope
B. Statement (2) ALONE is 3
sufficient, but statement (1) 2
alone is not sufficient. . What
C. BOTH statements TOGETHER is the value of b?
are sufficient, but NEITHER (1) a = 4
statement ALONE is (2) c = −6
sufficient. A. Statement (1) ALONE is
D. EACH statement ALONE is sufficient, but statement (2)
sufficient. alone is not sufficient.
E. Statements (1) and (2) B. Statement (2) ALONE is
TOGETHER are NOT sufficient. sufficient, but statement (1)
Answer: alone is not sufficient.
******************* 9
Q24: C. BOTH statements TOGETHER
On a map Town G is 10 are sufficient, but NEITHER
centimeters due east of Town H statement ALONE is
and 8 centimeters due south of sufficient.
Town J. Which of the following is D. EACH statement ALONE is
closest to the straight-line sufficient.
distance, in centimeters, E. Statements (1) and (2)
between Town H and Town J on TOGETHER are NOT sufficient.
the map? Answer:
A. 6 *******************
B. 13 Q27:
C. 18 The total cost of an office dinner
D. 20 was shared equally by k of the n
E. 24 employees who
Answer: attended the dinner. What was
******************* the total cost of the dinner?
Q25: (1) Each of the k employees who
If x is the product of the positive shared the cost of the dinner
integers from 1 to 8, inclusive, paid $19.
and if i, k, m, and p are (2) If the total cost of the dinner
positive integers such that x = had been shared equally by k +
2i3k5m7p, then i + k + m + p = 1 of the n
employees who attended the r
dinner, each of the k + 1 D.
employees would have 3
paid $18. 1
A. Statement (1) ALONE is r
sufficient, but statement (2) E.
alone is not sufficient. 23
B. Statement (2) ALONE is 1
sufficient, but statement (1) r
alone is not sufficient. Answer:
C. BOTH statements TOGETHER *******************
are sufficient, but NEITHER Q29:
statement ALONE is Leo can buy a certain computer
sufficient. for p1 dollars in State A, where
D. EACH statement ALONE is the sales tax is t1 percent,
sufficient. or he can buy the same
E. Statements (1) and (2) computer for p2 dollars in State
TOGETHER are NOT sufficient. B, where the sales tax is t2
Answer: percent. Is the total cost of the
******************* computer greater in State A than
Q28: in State B?
This year Henry will save a 10
certain amount of his income, (1) t1 > t2
and he will spend the rest. (2) p1t1 > p2t2
Next year Henry will have no A. Statement (1) ALONE is
income, but for each dollar that sufficient, but statement (2)
he saves this year, he will alone is not sufficient.
have 1 + r dollars available to B. Statement (2) ALONE is
spend. In terms of r, what sufficient, but statement (1)
fraction of his income should alone is not sufficient.
Henry save this year so that C. BOTH statements TOGETHER
next year the amount he was are sufficient, but NEITHER
available to spend will be equal statement ALONE is
to half the amount that he sufficient.
spends this year? D. EACH statement ALONE is
A. sufficient.
2 E. Statements (1) and (2)
1 TOGETHER are NOT sufficient.
r Answer:
B. *******************
22 Q30:
1 If x < 0 and 0 < y < 1, which of
r the following has the greatest
C. value?
32 A. x2
1 B. (xy)2
C. ( sufficient.
y D. EACH statement ALONE is
x sufficient.
)2 E. Statements (1) and (2)
D. TOGETHER are NOT sufficient.
y Answer:
x2 *******************
E. x2y Q33:
Answer: The surface of a certain planet
******************* reflects 80 percent of the light
Q31: that strikes it. The clouds
Three printing presses, R, S, and around the planet then absorb
T, working together at their 40 percent of the reflected light.
respective constant rates, What percent of the light
can do a certain printing job in 4 that strikes the planet is
hours. S and T, working together reflected from the surface and
at their respective passes through the clouds
constant rates, can do the same without
job in 5 hours. How many hours being absorbed?
would it take R, working A. 32%
alone at its constant rate, to do B. 40%
the same job? C. 48%
A. 8 D. 60%
B. 10 E. 88%
C. 12 Answer:
D. 15 *******************
E. 20 Q34:
Answer: The selling price of an article is
******************* equal to the cost of the article
Q32: plus the markup. The
Is x an odd integer? markup on a certain television
(1) x + 3 is an even integer. set is what percent of the selling
(2) price?
3 (1) The markup on the television
x set is 25 percent of the cost.
is an odd integer. (2) The selling price of the
A. Statement (1) ALONE is television set is $250.
sufficient, but statement (2) A. Statement (1) ALONE is
alone is not sufficient. sufficient, but statement (2)
B. Statement (2) ALONE is alone is not sufficient.
sufficient, but statement (1) B. Statement (2) ALONE is
alone is not sufficient. sufficient, but statement (1)
11 alone is not sufficient.
C. BOTH statements TOGETHER C. BOTH statements TOGETHER
are sufficient, but NEITHER are sufficient, but NEITHER
statement ALONE is statement ALONE is
sufficient. *******************
D. EACH statement ALONE is Q36:
sufficient. What is the sum of a certain pair
E. Statements (1) and (2) of consecutive odd integers?
TOGETHER are NOT sufficient. (1) At least one of the integers is
Answer: negative.
******************* (2) At least one of the integers is
Q35: positive.
If A. Statement (1) ALONE is
3 sufficient, but statement (2)
7x alone is not sufficient.
+ a = 8 and a > 8, then, in terms B. Statement (2) ALONE is
of a, sufficient, but statement (1)
7 alone is not sufficient.
3x C. BOTH statements TOGETHER
= are sufficient, but NEITHER
12 statement ALONE is
A. sufficient.
a D. EACH statement ALONE is
1 sufficient.
+ E. Statements (1) and (2)
8 TOGETHER are NOT sufficient.
1 Answer:
B. *******************
8 Q37:
1 If an automobile averaged 22.5
- miles per gallon of gasoline,
a approximately how many
1 kilometers per liter of gasoline
C. did the automobile average? (1
8a mile = 1.6 kilometers and
1 1 gallon = 3.8 liters, both
D. rounded to the nearest tenth.)
7 A. 3.7
3 B. 9.5
(8 – a) C. 31.4
E. D. 53.4
49 E. 136.8
9 Answer:
(8 – a) *******************
Answer: *******************
Answers:
EEBDD, DDBCC, ADBEE, EAECE, EDCBD, ACEEC, EDCAE, CB

1
Math Section
*******************
Q1: E. Statements (1) and (2) TOGETHER
At the opening of a trading day at a are NOT sufficient.
certain stock exchange, the price per Answer:
share of stock K *******************
was $8. If the price per share of stock K Q4:
was $9 at the closing of the day, what John, Karen, and Luke collected cans of
was the vegetables for a food drive. The number
percent increase in the price per share of of cans
stock K for that day? that John collected was 1/2 the number
A. 1.4% of cans that Karen collected and 1/3 the
B. 5.9% number
C. 11.1% of cans that Luke collected. The number
D. 12.5% of cans that Luke collected was what
E. 23.6% fraction of
Answer: the total number of cans that John,
******************* Karen, and Luke collected?
Q2: 2
Which of the following is equal to (212 – A. 1/5
26) / (26 – 23)? B. 1/3
A. 26 + 23 C. 2/5
B. 26 – 23 D. 1/2
C. 29 E. 2/3
D. 23 Answer:
E. 2 *******************
Answer: Q5:
******************* The sides of a square region, measured
Q3: to the nearest centimeters, are 6
What is the probability that a student centimeters long.
randomly selected from a class of 60 The least possible value of the actual
students will area of the square region is
be a male who has brown hair? A. 36.00 sq cm
(1) One-half of the students have brown B. 35.00 sq cm
hair. C. 33.75 sq cm
(2) One-third of the students are males. D. 30.25 sq cm
A. Statement (1) ALONE is sufficient, E. 25.00 sq cm
but statement (2) alone is not sufficient. Answer:
B. Statement (2) ALONE is sufficient, *******************
but statement (1) alone is not sufficient. Q6:
C. BOTH statements TOGETHER are For all integers x and y, the operation
sufficient, but NEITHER statement is defined by x y = (x+2)2 + (y+3)2.
ALONE is What is the
sufficient. value of integer t?
D. EACH statement ALONE is (1) t 2 = 74
sufficient. (2) 2 t = 80
A. Statement (1) ALONE is sufficient, C. 20
but statement (2) alone is not sufficient. D. 28
B. Statement (2) ALONE is sufficient, E. 35
but statement (1) alone is not sufficient. Answer:
C. BOTH statements TOGETHER are *******************
sufficient, but NEITHER statement Q9:
ALONE is If x and y are nonzero integers, is 18 a
sufficient. factor of xy2?
D. EACH statement ALONE is (1) x is a multiple of 54.
sufficient. (2) y is a multiple of 6.
E. Statements (1) and (2) TOGETHER A. Statement (1) ALONE is sufficient,
are NOT sufficient. but statement (2) alone is not sufficient.
Answer: B. Statement (2) ALONE is sufficient,
******************* but statement (1) alone is not sufficient.
Q7: C. BOTH statements TOGETHER are
Before a certain tire is used, 40 percent sufficient, but NEITHER statement
of its total weight consists of tread. If ALONE is
during a sufficient.
lifetime of use, 50 percent, by weight, of D. EACH statement ALONE is
the tire’s tread is lost and no other parts sufficient.
of the E. Statements (1) and (2) TOGETHER
tire is lost, what per cent of the tire’s are NOT sufficient.
total remaining weight consists of the Answer:
remaining *******************
tread? Q10:
A. 55% Is (t/3) > (w/5)?
B. 20% (1) w > t
C. 25% (2) 5t > 3w
D. 30% A. Statement (1) ALONE is sufficient,
E. 35% but statement (2) alone is not sufficient.
Answer: B. Statement (2) ALONE is sufficient,
3 but statement (1) alone is not sufficient.
******************* C. BOTH statements TOGETHER are
Q8: sufficient, but NEITHER statement
When positive integer x is divided by 5, ALONE is
the remainder is 3; and when x is divided sufficient.
by 7, D. EACH statement ALONE is
the remainder is 4. When positive sufficient.
integer y is divided by 5, the remainder E. Statements (1) and (2) TOGETHER
is 3; and when are NOT sufficient.
y is divided by 7, the remainder is 4. If x Answer:
> y, which of the following must be a *******************
factor of Q11:
x - y? A certain company that sells only cars
A. 12 and trucks reported that revenues from
B. 15 car sales in
1997 were down 11 percent from 1996 Answer:
and revenues from truck sales in 1997 *******************
were up 7 Q14:
percent from 1996. If total revenues For a convention, a hotel charges a daily
from car sales and truck sales in 1997 room rate of $120 for 1 person and x
were up 1 dollars for
4 each additional person. What is the
percent from 1996, what is the ratio of charge for each additional person?
revenue from car sales in 1996 to (1) The daily cost per person for 4
revenue from people sharing the cost of a room
truck sales in 1996? equally is $45.
A. 1 : 2 (2) The daily cost per person for 2
B. 4 : 5 people sharing the cost of a room
C. 1 : 1 equally is $25
D. 3 : 2 more than the corresponding cost for 4
E. 5 : 3 people.
Answer: A. Statement (1) ALONE is sufficient,
******************* but statement (2) alone is not sufficient.
Q12: B. Statement (2) ALONE is sufficient,
Which of the following is equal to but statement (1) alone is not sufficient.
[1/( SQRT(3)- SQRT(2))]2 C. BOTH statements TOGETHER are
A. 1 sufficient, but NEITHER statement
B. 5 ALONE is
C. SQRT(6) sufficient.
D.5 - SQRT(6) 5
D. EACH statement ALONE is
E. 5 + 2 SQRT(6)
sufficient.
Answer:
E. Statements (1) and (2) TOGETHER
*******************
are NOT sufficient.
Q13:
Answer:
On a certain day, Tim invested $1,000 at
*******************
10 percent annual interest, compounded
Q15:
annually, and Lana invested $2,000 at 5
Is xy > x/y?
percent annual interest, compounded
(1) xy > 0
annually.
(2) y < 0
The total amount of interest earned by
A. Statement (1) ALONE is sufficient,
Tim’s investment in the first 2 years was
but statement (2) alone is not sufficient.
how
B. Statement (2) ALONE is sufficient,
much greater than the total amount of
but statement (1) alone is not sufficient.
interest earned by Lana’s investment in
C. BOTH statements TOGETHER are
the first 2
sufficient, but NEITHER statement
years?
ALONE is
A. $5
sufficient.
B. $15
D. EACH statement ALONE is
C. $50
sufficient.
D. $100
E. $105
E. Statements (1) and (2) TOGETHER C. 12
are NOT sufficient. D. 13
Answer: E. 14
******************* Answer:
Q16: *******************
Is r > -1? Q19:
(1) r > -5 In the xy-plane, the sides of a certain
(2) r < 16 rectangle are parallel to the axes. If one
A. Statement (1) ALONE is sufficient, of the
but statement (2) alone is not sufficient. vertices of the rectangle is (-1,-2), what
B. Statement (2) ALONE is sufficient, is the perimeter of the rectangle?
but statement (1) alone is not sufficient. (1) One of the vertices of the rectangle is
C. BOTH statements TOGETHER are (2, -2).
sufficient, but NEITHER statement (2) One of the vertices of the rectangle is
ALONE is (2, 3).
sufficient. A. Statement (1) ALONE is sufficient,
D. EACH statement ALONE is but statement (2) alone is not sufficient.
sufficient. B. Statement (2) ALONE is sufficient,
E. Statements (1) and (2) TOGETHER but statement (1) alone is not sufficient.
are NOT sufficient. C. BOTH statements TOGETHER are
Answer: sufficient, but NEITHER statement
******************* ALONE is
Q17: sufficient.
If the volume of a small container is D. EACH statement ALONE is
14,520 cubic millimeters, what is the sufficient.
volume of the E. Statements (1) and (2) TOGETHER
container in cubic centimeters? are NOT sufficient.
(1 millimeter = 0.1 centimeter) Answer:
A. 0.1452 *******************
B. 1.452 Q20:
C. 14.52 Tom, Jane, and Sue each purchased a
D. 145.2 new house. The average (arithmetic
E. 1,452 mean) price of
Answer: the three houses was $120,000. What
******************* was the median price of the three
Q18: houses?
2x + y = 12 (1) The price of Tom’s house was
|y| <= 12 $110,000.
6 (2) The price of Jane’s house was
For how many ordered pairs (x , y) $120,000.
that are solutions of the system A. Statement (1) ALONE is sufficient,
above are x but statement (2) alone is not sufficient.
and y both B. Statement (2) ALONE is sufficient,
integers? but statement (1) alone is not sufficient.
A. 7
B. 10
C. BOTH statements TOGETHER are A. Statement (1) ALONE is sufficient,
sufficient, but NEITHER statement but statement (2) alone is not sufficient.
ALONE is B. Statement (2) ALONE is sufficient,
sufficient. but statement (1) alone is not sufficient.
D. EACH statement ALONE is C. BOTH statements TOGETHER are
sufficient. sufficient, but NEITHER statement
E. Statements (1) and (2) TOGETHER ALONE is
are NOT sufficient. sufficient.
Answer: D. EACH statement ALONE is
******************* sufficient.
Q21: E. Statements (1) and (2) TOGETHER
A certain circle in the xy-plane has its are NOT sufficient.
center at the origin. If P is a point on the Answer:
circle, *******************
what is the sum of the squares of the Q23:
coordinates of P? One kilogram of a certain coffee blend
(1) The radius of the circle is 4. consists of x kilogram of type I coffee
(2) The sum of the coordinates of P is 0. and y
7 kilogram of type II coffee. The cost of
A. Statement (1) ALONE is sufficient, the blend is C dollars per kilogram,
but statement (2) alone is not sufficient. where C =
B. Statement (2) ALONE is sufficient, 6.5x + 8.5y. Is x < 0.8?
but statement (1) alone is not sufficient. (1) y > 0.15
C. BOTH statements TOGETHER are (2) C >=7.30
sufficient, but NEITHER statement A. Statement (1) ALONE is sufficient,
ALONE is but statement (2) alone is not sufficient.
sufficient. B. Statement (2) ALONE is sufficient,
D. EACH statement ALONE is but statement (1) alone is not sufficient.
sufficient. C. BOTH statements TOGETHER are
E. Statements (1) and (2) TOGETHER sufficient, but NEITHER statement
are NOT sufficient. ALONE is
Answer: sufficient.
******************* D. EACH statement ALONE is
Q22: sufficient.
What is the median number of E. Statements (1) and (2) TOGETHER
employees assigned per project for the are NOT sufficient.
projects at Answer:
Company Z? *******************
(1) 25 percent of the projects at Q24:
Company Z have 4 or more employees Of the goose eggs laid at a certain pond,
assigned to 3
each project. 2
(2) 35 percent of the projects at hatched, and
Company Z have 2 or fewer employees 4
assigned to 3
each project. of the geese that hatched
from those eggs survived the first month. Answer:
Of the geese that survived the first *******************
month, Q27:
5 The interior of a rectangular carton is
3 designed by a certain manufacturer to
8 have a
did not survive the first year. If 120 volume of x cubic feet and a ratio of
geese survived the first year and if no length to width to height of 3 : 2 : 2. In
more than one terms of x,
goose hatched from each egg, how many which of the following equals the height
goose eggs were laid at the pond? of the carton, in feet?
A. 280 A. 3 x
B. 400 B. 3
C. 540 3
D. 600 2x
E. 840 C. 3
Answer: 2
******************* 3x
Q25: 9
If n is a positive integer, what is the D. 3
remainder when 38n+3 + 2 is divided by 3
5? 2
A. 0 x
B. 1 E. 3
C. 2 2
D. 3 3
E. 4 x
Answer: Answer:
******************* *******************
Q26: Q28:
What is the greatest common factor of A certain company sold 800 units of its
the positive integers j and k? product for $8 each and 1,000 units of its
(1) k = j + 1 product
(2) jk is divisible by 5. for $5 each. If the company’s cost of
A. Statement (1) ALONE is sufficient, producing each unit of its product was
but statement (2) alone is not sufficient. $6, what
B. Statement (2) ALONE is sufficient, was the company’s profit or loss on the
but statement (1) alone is not sufficient. 1,800 units of its product?
C. BOTH statements TOGETHER are A. $1,600 loss
sufficient, but NEITHER statement B. $600 loss
ALONE is C. No profit or loss
sufficient. D. $600 profit
D. EACH statement ALONE is E. $1,600 profit
sufficient. Answer:
E. Statements (1) and (2) TOGETHER *******************
are NOT sufficient. Q29:
What is the remainder when the positive B. Statement (2) ALONE is sufficient,
integer n is divided by the positive but statement (1) alone is not sufficient.
integer k, C. BOTH statements TOGETHER are
where k > 1? sufficient, but NEITHER statement
(1) n = (k+1)3 ALONE is
(2) k = 5 sufficient.
A. Statement (1) ALONE is sufficient, D. EACH statement ALONE is
but statement (2) alone is not sufficient. sufficient.
B. Statement (2) ALONE is sufficient, E. Statements (1) and (2) TOGETHER
but statement (1) alone is not sufficient. are NOT sufficient.
C. BOTH statements TOGETHER are Answer:
sufficient, but NEITHER statement *******************
ALONE is Q32:
sufficient. What is the remainder when the sum of
D. EACH statement ALONE is the positive integers x and y is divided
sufficient. by 6?
E. Statements (1) and (2) TOGETHER (1) When x is divided by 6, the
are NOT sufficient. remainder is 3.
Answer: (2) When y is divided by 6, the
******************* remainder is 1.
Q30: A. Statement (1) ALONE is sufficient,
In a stack of cards, 9 cards are blue and but statement (2) alone is not sufficient.
the rest are red. If 2 cards are to be B. Statement (2) ALONE is sufficient,
chosen at but statement (1) alone is not sufficient.
random from the stack without C. BOTH statements TOGETHER are
replacement, the probability that the sufficient, but NEITHER statement
cards chosen will ALONE is
both be blue is 6/11. What is the number sufficient.
of cards in the stack? D. EACH statement ALONE is
A. 10 sufficient.
B. 11 E. Statements (1) and (2) TOGETHER
C. 12 are NOT sufficient.
D. 15 Answer:
E. 18 *******************
Answer: Q33:
******************* If xy <> 0, what is the value of 1/x +
Q31: 1/y?
10 (1) 1 / (x+y) = -1
In triangle PQR, the measure of angle P (2) xy = 6(x+y)
is 30° more than twice the measure of A. Statement (1) ALONE is sufficient,
angle Q. but statement (2) alone is not sufficient.
What is the measure of angle R? B. Statement (2) ALONE is sufficient,
(1) PQ = QR but statement (1) alone is not sufficient.
(2) The measure of angle P is 78°. C. BOTH statements TOGETHER are
A. Statement (1) ALONE is sufficient, sufficient, but NEITHER statement
but statement (2) alone is not sufficient. ALONE is
sufficient. If x, y, and k are positive numbers such
D. EACH statement ALONE is that (
sufficient. xy
E. Statements (1) and (2) TOGETHER x
are NOT sufficient. 
Answer: )(10) + (
******************* xy
Q34: y
If b(a+1) = b, then ab = 
A. –1 )(20) = k and if x < y,
11 which of the following could be the
B. 0 value of k?
C. 1 A. 10
E. a B. 12
F. b C. 15
Answer: D. 18
******************* E. 30
Q35: Answer:
In the rectangular solid above, the three *******************
sides shown have areas 12, 15, and 20, Q37:
respectively. What is the volume of the If k is an integer and
solid? (0.0025)(0.025)(0.00025)×10k is an
A. 60 integer, what is the least
B. 120 possible value of k?
C. 450 A. -12
D. 1,800 B. -6
E. 3,600 C. 0
Answer: 12
******************* D. 6
Q36: E. 12
Answer:
*******************
*******************
Answers:
DAEDD, CCEDB, AEADE, ECDBB, ACBDE, ABDAC, DCBBA, DE

1
Math Section
*******************
Q1: B. 9
When 20 is divided by the C. 10
positive integer k, the remainder D. 11
is k – 2. Which of the E. 12
following is a possible value of k Answer:
? *******************
A. 8 Q2:
QR Q4:
PS If xy = 165, where x and y are
In the figure above, is positive integers and x > y, what
quadrilateral PQRS a is the least possible value
parallelogram? of x – y ?
(1) The area of ΔPQS is equal to A. 2
the area of ΔQRS. B. 4
(2) QR = RS C. 8
A. Statement (1) ALONE is D. 15
sufficient, but statement (2) E. 28
alone is not sufficient. Answer:
B. Statement (2) ALONE is *******************
sufficient, but statement (1) Q5:
alone is not sufficient. According to a survey, 93
C. BOTH statements TOGETHER percent of teenagers have used
are sufficient, but NEITHER a computer to play games, 89
statement ALONE is percent have used a computer
sufficient. to write reports, and 5 percent
D. EACH statement ALONE is have not used a computer
sufficient. for either of these purposes.
E. Statements (1) and (2) What percent of the teenagers
TOGETHER are NOT sufficient. in the survey have used a
Answer: computer both to play games
******************* and to write reports?
Q3: A. 82%
If n is an integer and xn – x-n = 0, B. 87%
what is the value of x ? C. 89%
(1) x is an integer. D. 92%
(2) n ≠ 0 E. 95%
A. Statement (1) ALONE is Answer:
sufficient, but statement (2) *******************
alone is not sufficient. Q6:
B. Statement (2) ALONE is The value of –3 + 9  k is positive
sufficient, but statement (1) for which of the following values
alone is not sufficient. of k ?
C. BOTH statements TOGETHER I. -1
are sufficient, but NEITHER II. 0
statement ALONE is III. 1
sufficient. A. None
2 B. I only
D. EACH statement ALONE is C. II only
sufficient. D. III only
E. Statements (1) and (2) E. I and III
TOGETHER are NOT sufficient. Answer:
Answer: *******************
******************* Q7:
3 A. Statement (1) ALONE is
There were 2 apples and 5 sufficient, but statement (2)
bananas in a basket. After alone is not sufficient.
additional apples and bananas B. Statement (2) ALONE is
were sufficient, but statement (1)
placed in the basket, the ratio of alone is not sufficient.
the number of apples to the C. BOTH statements TOGETHER
number of bananas was are sufficient, but NEITHER
2 statement ALONE is
1 sufficient.
. D. EACH statement ALONE is
How many apples were added? sufficient.
(1) The number of apples added E. Statements (1) and (2)
was TOGETHER are NOT sufficient.
3 Answer:
2 *******************
the number of bananas added. Q9:
(2) A total of 5 apples and A total of 100 customers
bananas were added. purchased books at a certain
A. Statement (1) ALONE is bookstore last week. If these
sufficient, but statement (2) customers purchased a total of
alone is not sufficient. 200 books, how many of the
B. Statement (2) ALONE is customers purchased only 1
sufficient, but statement (1) book each?
alone is not sufficient. (1) None of the customers
C. BOTH statements TOGETHER purchased more than 3 books.
are sufficient, but NEITHER (2) 20 of the customers
statement ALONE is purchased only 2 books each.
sufficient. A. Statement (1) ALONE is
D. EACH statement ALONE is sufficient, but statement (2)
sufficient. alone is not sufficient.
E. Statements (1) and (2) B. Statement (2) ALONE is
TOGETHER are NOT sufficient. sufficient, but statement (1)
Answer: alone is not sufficient.
******************* C. BOTH statements TOGETHER
Q8: are sufficient, but NEITHER
S is a set of points in the plane. statement ALONE is
How many distinct triangles can sufficient.
be drawn that have three D. EACH statement ALONE is
of the points in S as vertices? sufficient.
(1) The number of distinct points E. Statements (1) and (2)
in S is 5. TOGETHER are NOT sufficient.
(2) No three of the points in S Answer:
are collinear. *******************
Q10:
4
C If the integer k is a multiple of 3,
B which of the following is also a
D multiple of 3 ?
AE A. k + 2
In the figure, AB = AE = 8, BC = B. 2k + 8
CD = 13, and DE = 2. What is C. 3k + 5
the area of region D. 4k + 4
ABCDE ? E. 5k + 12
A. 76 Answer:
B. 84 5
C. 92 *******************
D. 100 Q13:
E. 108 For any positive integer x, the 2-
Answer: height of x is defined to be the
******************* greatest nonnegative
Q11: integer n such that 2n is a factor
A certain business produced x of x. If k and m are positive
rakes each month form integers, is the 2-height of k
November through February and greater than the 2-height of m ?
shipped (1) k > m
2 (2)
x m
rakes at the beginning of each k
month from March through is an even integer.
October. The A. Statement (1) ALONE is
business paid no storage costs sufficient, but statement (2)
for the rakes from November alone is not sufficient.
through February, but it paid B. Statement (2) ALONE is
storage costs of $0.10 per rake sufficient, but statement (1)
each month from March through alone is not sufficient.
October for the rakes that C. BOTH statements TOGETHER
had not been shipped. In terms are sufficient, but NEITHER
of x, what was the total storage statement ALONE is
cost, in dollars, that the sufficient.
business paid for the rakes for D. EACH statement ALONE is
the 12 months form November sufficient.
through October? E. Statements (1) and (2)
A. 0.40x TOGETHER are NOT sufficient.
B. 1.20x Answer:
C. 1.40x *******************
D. 1.60x Q14:
E. 3.20x If 4x < x < x3 < x2, which of the
Answer: following is a possible value of x
******************* ?
Q12: A. –2
B. –
2 hour for the remaining 3 hours.
3 What was her average speed, in
C. –1 miles per hour, for the
D. – entire trip?
2 (1) 2x + 3y = 280
1 (2) y = x + 10
E. 0 A. Statement (1) ALONE is
Answer: sufficient, but statement (2)
******************* alone is not sufficient.
Q15: B. Statement (2) ALONE is
What was the range of the sufficient, but statement (1)
selling prices of the 30 wallets alone is not sufficient.
sold by a certain store C. BOTH statements TOGETHER
yesterday? are sufficient, but NEITHER
(1) statement ALONE is
3 sufficient.
1 D. EACH statement ALONE is
of the wallets had a selling price sufficient.
of $24 each. E. Statements (1) and (2)
(2) The lowest selling price of TOGETHER are NOT sufficient.
the wallets was Answer:
3 *******************
1 Q17:
the highest selling price of the In a box of 12 pens, a total of 3
wallets. are defective. If a customer
A. Statement (1) ALONE is buys 2 pens selected at
sufficient, but statement (2) random from the box, what is
alone is not sufficient. the probability that neither pen
B. Statement (2) ALONE is will be defective?
sufficient, but statement (1) A.
alone is not sufficient. 6
C. BOTH statements TOGETHER 1
are sufficient, but NEITHER B.
statement ALONE is 9
sufficient. 2
D. EACH statement ALONE is C.
sufficient. 11
E. Statements (1) and (2) 6
TOGETHER are NOT sufficient. D.
Answer: 16
6 9
******************* E.
Q16: 4
On a certain nonstop trip, Marta 3
averaged x miles per hour for 2 Answer:
hours and y miles per *******************
Q18: B. Statement (2) ALONE is
A scientist recorded the number sufficient, but statement (1)
of eggs in each of 10 birds’ alone is not sufficient.
nests. What was the C. BOTH statements TOGETHER
standard deviation of the are sufficient, but NEITHER
numbers of eggs in the 10 statement ALONE is
nests? sufficient.
(1) The average (arithmetic D. EACH statement ALONE is
mean) number of eggs for the sufficient.
10 nests was 4. E. Statements (1) and (2)
(2) Each of the 10 nests TOGETHER are NOT sufficient.
contained the same number of Answer:
eggs. *******************
A. Statement (1) ALONE is Q20:
sufficient, but statement (2) If a1 = 1 and an + 1 = 1 +
alone is not sufficient. na
B. Statement (2) ALONE is 1
sufficient, but statement (1) for all n ≥ 1, what is the value of
alone is not sufficient. a5 ?
C. BOTH statements TOGETHER A.
are sufficient, but NEITHER 8
statement ALONE is 3
sufficient. B.
D. EACH statement ALONE is 8
sufficient. 5
E. Statements (1) and (2) C.
TOGETHER are NOT sufficient. 5
Answer: 8
7 D.
******************* 3
Q19: 5
If p, r, s, and t are nonzero E.
integers, is 3
r 8
p Answer:
= *******************
t Q21:
s A toy store’s gross profit on a
? computer game was 10 percent
(1) s = 3p and t = 3r. of the cost of the game. If
(2) 3p = 2r and 3s = 2t. the store increased the selling
A. Statement (1) ALONE is price of the game from $44 to
sufficient, but statement (2) $46 and the cost of the game
alone is not sufficient. remained the same, then the
store’s gross profit on the game
after the price increase was
what percent of the cost of the m
game? D.
A. 10.5% ( 4)
B. 11% 48
C. 12.5% 
D. 13% 
E. 15% mm
Answer: m
******************* E.
Q22: ( 4)
8 4 12
Four extra-large sandwiches of 
exactly the same size were 
ordered for m students, where mm
m > 4. Three of the sandwiches m
were evenly divided among the Answer:
students. Since 4 *******************
students did not want any of the Q23:
fourth sandwich, it was evenly When the wind speed is 9 miles
divided among the per hour, the wind-chill factor w
remaining students. If Carol ate is given by w = -17.366
one piece from each of the four + 1.19t, where t is the
sandwiches, the amount temperature in degrees
of sandwich that she ate would Fahrenheit. If at noon yesterday
be what fraction of a whole the wind
extra-large sandwich? speed was 9 miles per hour, was
A. the wind-chill factor greater
( 4) than 0 ?
4 (1) The temperature at noon
 yesterday was greater than 10
 degrees Fahrenheit.
mm (2) The temperature at noon
m yesterday was less than 20
B. degrees Fahrenheit.
( 4) A. Statement (1) ALONE is
24 sufficient, but statement (2)
 alone is not sufficient.
 B. Statement (2) ALONE is
mm sufficient, but statement (1)
m alone is not sufficient.
C. C. BOTH statements TOGETHER
( 4) are sufficient, but NEITHER
44 statement ALONE is
 sufficient.
 D. EACH statement ALONE is
mm sufficient.
E. Statements (1) and (2) n

TOGETHER are NOT sufficient.


Answer:
*******************

Q24:
5
Which of the following is closest
1
in value to 99 – 92 ?
will be greatest for n =
A. 99
A. 3
B. 98
B. 2
C. 97
C. 0
D. 96
D. –2
E. 95
E. –3
Answer:
Answer:
*******************
*******************
Q25:
Q27:
9
For all numbers x, the function f
An office supply store received a
is defined by f(x) = 3x + 1 and
shipment of boxes of pencils.
the function g is defined
How many pencils were
by g(x) =
in the shipment altogether? (1
3
gross = 12 dozen)
x 1
(1) The number of boxes of
. If c is a positive number, what
pencils in the shipment was 6
gross. is the value of g(c)?
(2) There were 3 dozen pencils (1) f(c) = 13
in each box of pencils in the (2) f(1) = c
shipment. A. Statement (1) ALONE is
A. Statement (1) ALONE is sufficient, but statement (2)
sufficient, but statement (2) alone is not sufficient.
alone is not sufficient. B. Statement (2) ALONE is
B. Statement (2) ALONE is sufficient, but statement (1)
sufficient, but statement (1) alone is not sufficient.
alone is not sufficient. C. BOTH statements TOGETHER
C. BOTH statements TOGETHER are sufficient, but NEITHER
are sufficient, but NEITHER statement ALONE is
statement ALONE is sufficient.
sufficient. D. EACH statement ALONE is
D. EACH statement ALONE is sufficient.
sufficient. E. Statements (1) and (2)
E. Statements (1) and (2) TOGETHER are NOT sufficient.
TOGETHER are NOT sufficient. Answer:
Answer: *******************
******************* Q28:
Q26: A certain company expects
Of the following values of n, the quarterly earnings of $0.80 per
value of share of stock, half of which
will be distributed as dividends C. BOTH statements TOGETHER
to shareholders while the rest are sufficient, but NEITHER
will be used for research statement ALONE is
and development. If earnings sufficient.
are greater than expected, D. EACH statement ALONE is
shareholders will receive an sufficient.
additional $0.04 per share for E. Statements (1) and (2)
each additional $0.10 of per TOGETHER are NOT sufficient.
share earnings. If quarterly Answer:
earnings are $1.10 per share, *******************
what will be the dividend paid to Q31:
a person who owns 200 Phobus, one of the satellites of
shares of the company’s stock? Mars, revolves about Mars at a
10 distance of approximately
A. $92 5,800 miles from the center of
B. $96 Mars and approximately 3,700
C. $104 miles from the surface of
D. $120 Mars. If it is assumed that Mars
E. $240 is spherical and the orbit of
Answer: Phobus is circular, what is
******************* the approximate circumference,
Q29: in miles, of Mars at its equator?
For which real numbers x is the A. 2,100π
expression B. 4,200π
( 2)( 1) C. 7,400π
2 D. 9,500π
xx E. 190,000π
not defined? Answer:
A. x = 0 only *******************
B. x = 2 only Q32:
C. x = –1 only 11
D. x = –1 or 2 In the decimal representation of
E. x = –1, 0, or 2 x, where 0 < x < 1, is the tenths
Answer: digit of x nonzero?
******************* (1) 16x is an integer.
Q30: (2) 8x is an integer.
What is the value of (2x – 5)3 ? A. Statement (1) ALONE is
(1) (2x)3 = 216 sufficient, but statement (2)
(2) (2x + 3)2 = 81 alone is not sufficient.
A. Statement (1) ALONE is B. Statement (2) ALONE is
sufficient, but statement (2) sufficient, but statement (1)
alone is not sufficient. alone is not sufficient.
B. Statement (2) ALONE is C. BOTH statements TOGETHER
sufficient, but statement (1) are sufficient, but NEITHER
alone is not sufficient. statement ALONE is
sufficient.
D. EACH statement ALONE is C. BOTH statements TOGETHER
sufficient. are sufficient, but NEITHER
E. Statements (1) and (2) statement ALONE is
TOGETHER are NOT sufficient. sufficient.
Answer: D. EACH statement ALONE is
******************* sufficient.
Q33: E. Statements (1) and (2)
A small business invests $9,900 TOGETHER are NOT sufficient.
in equipment to produce a Answer:
product. Each unit of the *******************
product costs $0.65 to produce Q35:
and is sold for $1.20. How many If g(x) = x + 4 for all x and f(x) =
units of the product x
must be sold before the revenue 2x2  5
received equals the total for all nonzero x, then f(x) = g(x)
expense of production, for which of
including the initial investment the following values of x ?
in equipment? 12
A. 12,000 A. –5
B. 14,500 B. –3
C. 15,230 C. 1
D. 18,000 D. 3
E. 20,000 E. 5
Answer: Answer:
******************* *******************
Q34: Q36:
A merchant discounted the sale Is x > 5 ?
price of a coat and the sale price (1) x2 > 25
of a sweater. Which of (2) 2x + 7 > 0
the two articles of clothing was A. Statement (1) ALONE is
discounted by the greater dollar sufficient, but statement (2)
amount? alone is not sufficient.
(1) The percent discount on the B. Statement (2) ALONE is
coat was 2 percentage points sufficient, but statement (1)
greater than the alone is not sufficient.
percent discount on the sweater. C. BOTH statements TOGETHER
(2) Before the discounts, the are sufficient, but NEITHER
sale price of the coat was $10 statement ALONE is
less than the sale price of sufficient.
the sweater. D. EACH statement ALONE is
A. Statement (1) ALONE is sufficient.
sufficient, but statement (2) E. Statements (1) and (2)
alone is not sufficient. TOGETHER are NOT sufficient.
B. Statement (2) ALONE is Answer:
sufficient, but statement (1) *******************
alone is not sufficient. Q37:
Pumping alone at their in 6 hours. How many hours will
respective constant rates, one it take both pipes, pumping
inlet pipe fills an empty tank to simultaneously at their
2 respective constant rates, to fill
1 the empty tank to capacity?
of capacity in 3 hours and a A. 3.25
second inlet pipe fills the same B. 3.6
empty tank to C. 4.2
3 D. 4.4
2 E. 5.5
of capacity
Answer:
*******************
*******************
Answers:
DEEBB, BDCCD, CEBDE, ACBDC, EEEAC, BDCDA, BBDEE, CB

14
Verbal Section
*******************
Q1:
Being that she was secretary of labor, Frances Perkins’ considerable influence with
Franklin D. Roosevelt was used preventing him from restraining strikes by
longshoremen and automobile workers.
A. Being that she was secretary of labor, Frances Perkins’ considerable influence
with Franklin D. Roosevelt was used preventing
B. As secretary of labor, Frances Perkins’ considerable influence with Franklin D.
Roosevelt was used to prevent
C. Being secretary of labor, Frances Perkins’ considerable influence with Franklin D.
Roosevelt was used preventing
D. As secretary of labor, Frances Perkins used her considerable influence with
Franklin D. Roosevelt to prevent
E. Secretary of labor, Frances Perkins’ considerable influence was used with
Franklin D. Roosevelt preventing
Answer:
*******************
Q2 to Q4:
The fields of antebellum (pre-Civil 1840’s of a new “American political
War) political history and women’s nation,” and since women were neither
history (10) voters nor politicians, they receive
use separate sources and focus little
Line on separate issues. Political histori- discussion. Women’s historians,
(5) ans, examining sources such as meanwhile,
voting have shown little interest in the
records, newspapers, and politicians’ subject of party politics, instead drawing
writings, focus on the emergence in the on personal papers, legal records
(15) such as wills, and records of female speeches. According to Whig
associations to illuminate women’s propaganda,
domestic lives, their moral reform women who turned out at the
activities, and the emergence of the (30) party’s rallies gathered information
woman’s rights movement. that enabled them to mold party-loyal
(20) However, most historians have families, reminded men of moral values
underestimated the extent and that transcended party loyalty, and
significance conferred
of women’s political allegiance moral standing on the party.
in the antebellum period. For example, (35) Virginia Democrats, in response,
in the presidential election campaigns began to make similar appeals to
15 women as well. By the mid-1850’s
(25) of the 1840’s, the Virginia Whig the inclusion of women in the rituals of
party party politics had become common-
strove to win the allegiance of Virginia’s (40) place, and the ideology that justified
women by inviting them to rallies and such inclusion had been assimilated
by the Democrats.
--------------------------------------------------------------------------------
Q2:
The primary purpose of the passage as a whole is to
A. examine the tactics of antebellum political parties with regard to women
B. trace the effect of politics on the emergence of the woman’s rights movement
C. point out a deficiency in the study of a particular historical period
D. discuss the ideologies of opposing antebellum political parties
E. contrast the methodologies in two differing fields of historical inquiry
Answer:
--------------------------------------------------------------------------------
Q3:
The author of the passage would be most likely to agree with which of the following
statements regarding most historians of the antebellum period?
A. They have failed to adequately contrast the differing roles that women played in
the Democratic and Whig parties in the 1850’s.
B. They have failed to see that political propaganda advocating women’s political
involvement did not reflect the reality of women’s actual roles.
C. They have incorrectly assumed that women’s party loyalty played a small role in
Whig and Democratic party politics.
D. They have misinterpreted descriptions of women’s involvement in party politics
in records of female associations and women’s personal papers.
E. They have overlooked the role that women’s political activities played in the
woman’s rights movement.
Answer:
--------------------------------------------------------------------------------
Q4:
16
According to the second paragraph of the passage (lines 20-42), Whig propaganda
included the assertion that
A. women should enjoy more political rights than they did
B. women were the most important influences on political attitudes within a family
C. women’s reform activities reminded men of important moral values
D. women’s demonstrations at rallies would influence men’s voting behavior
E. women’s presence at rallies would enhance the moral standing of the party
Answer:
*******************
Q5:
A South American bird that forages for winged termites and other small insects while
swinging upside down form the foliage of tall trees, the graveteiro belongs to the
ovenbird family, a group of New World tropical birds that includes more than 230
species and that are represented in virtually every kind of habitat.
A. graveteiro belongs to the ovenbird family, a group of New World tropical birds
that includes more than 230 species and that are
B. graveteiro belongs to the ovenbird family, a group of New World tropical birds
that includes more than 230 species and is
C. graveteiro belongs to the ovenbird family, a group of New World tropical birds
that include more than 230 species and is
D. graveteiro, which belongs to the ovenbird family, a group of New World tropical
birds that includes more than 230 species and that are
E. graveteiro, which belongs to the ovenbird family, a group of New World tropical
birds that includes more than 230 species and is
Answer:
*******************
Q6:
The population of India has been steadily increasing for decades, and it will probably
have what is estimated as 1.6 billion people by 2050 and surpass China as the world’s
most populous nation.
A. it will probably have what is estimated as
B. they are likely to have
C. the country will probably have
D. there will probably be
E. there will be an estimated
Answer:
*******************
Q7:
The Quechuans believed that all things participated in both the material level and the
mystical level of reality, and many individual Quechuans claimed to have contact with it
directly with an ichana (dream) experience.
A. contact with it directly with
17
B. direct contact with it by way of
C. contact with the last directly through
D. direct contact with the latter by means of
E. contact directly with the mystical level due to
Answer:
*******************
Q8:
Personnel officer: The exorbitant cost of our health-insurance benefits reflects the high
dollar amount of medical expenses incurred by our employees. Employees who are out
of shape, as a group, have higher doctor bills and longer hospital stays than do their
colleagues who are fit. Therefore, since we must reduce our health-insurance costs, we
should offer a rigorous fitness program of jogging and weight lifting to all employees,
and require employees who are out of shape to participate.
The conclusion reached by the personnel officer depends on which of the following
assumptions?
A. A person who is fit would receive a routine physical checkup by a doctor less
regularly than would a person who is out of shape.
B. The medical expenses incurred by employees who are required to participate in
the fitness program would be less than those incurred by employees who are not
required to participate.
C. The strenuous activities required of out-of-shape employees by the program
would not by themselves generate medical expenses greater than any reduction
achieved by the program.
D. The fitness program would serve more employees who are out of shape than it
would employees who are fit.
E. The employees who participate in the fitness program would be away from work
because of illness less than would the employees who do not participate.
Answer:
*******************
Q9:
The population of desert tortoises in Targland’s Red Desert has declined, partly because
they are captured for sale as pets and partly because people riding all-terrain vehicles
have damaged their habitat. Targland plans to halt this population decline by blocking
the current access routes into the desert and announcing new regulations to allow access
only on foot. Targland’s officials predict that these measures will be adequate, since it is
difficult to collect the tortoises without a vehicle.
Which of the following would it be most important to establish in order to evaluate the
officials’ prediction?
A. Whether possessing the tortoises as pets remains legally permissible in Targland
B. Whether Targland is able to enforce the regulations with respect to all-terrain
vehicle entry at points other than the current access routes
C. Whether the Red Desert tortoises are most active during the day or at night
18
D. Whether people who travel on foot in the Red Desert often encounter the tortoises
E. Whether the Targland authorities held public hearings before restricting entry by
vehicle into the Red Desert
Answer:
*******************
Q10:
The single-family house constructed by the Yana, a Native American people who lived in
what is now northern California, was conical in shape, its framework of poles overlaid
with slabs of bark, either cedar or pine, and banked with dirt to a height of three to four
feet.
A. banked with dirt to a height of
B. banked with dirt as high as that of
C. banked them with dirt to a height of
D. was banked with dirt as high as
E. was banked with dirt as high as that of
Answer:
*******************
Q11:
Finding of a survey of Systems magazine subscribers: Thirty percent of all merchandise
orders placed by subscribers in response to advertisements in the magazine last year were
placed by subscribers under age thirty-five.
Finding of a survey of advertisers in Systems magazine: Most of the merchandise orders
placed in response to advertisements in Systems last year were placed by people under
age thirty-five.
For both of the findings to be accurate, which of the following must be true?
A. More subscribers to Systems who have never ordered merchandise in response to
advertisements in the magazine are age thirty-five or over than are under age
thirty-five.
B. Among subscribers to Systems, the proportion who are under age thirty-five was
considerably lower last year than it is now.
C. Most merchandise orders placed in response to advertisements in Systems last
year were placed by Systems subscribers over age thirty-five.
D. Last year, the average dollar amount of merchandise orders placed was less for
subscribers under age thirty-five than for those age thirty-five or over.
E. Last year many people who placed orders for merchandise in response to
advertisements in Systems were not subscribers to the magazine.
Answer:
*******************
Q12:
In Brindon County, virtually all of the fasteners—such as nuts, bolts, and screws—used
by workshops and manufacturing firms have for several years been supplied by the
Brindon Bolt Barn, a specialist wholesaler. In recent months many of Brindon County’s
19
workshops and manufacturing firms have closed down, and no new ones have opened.
Therefore, the Brindon Bolt Barn will undoubtedly show a sharp decline in sales volume
and revenue for this year as compared to last year.
The argument depends on assuming which of the following?
A. Last year the Brindon Bolt Barn’s sales volume and revenue were significantly
higher than they had been the previous year.
B. The workshops and manufacturing firms that have remained open have a smaller
volume of work to do this year than they did last year.
C. Soon the Brindon Bolt Barn will no longer be the only significant supplier of
fasteners to Brindon County’s workshops.
D. The Brindon Bolt Barn’s operating expenses have not increased this year.
E. The Brindon Bolt Barn is not a company that gets the great majority of its
business from customers outside Brindon County.
Answer:
*******************
Q13:
Today’s technology allows manufacturers to make small cars more fuel-efficient now
than at any time in their production history.
A. small cars more fuel-efficient now than at any time in their
B. small cars that are more fuel-efficient than they were at any time in their
C. small cars that are more fuel-efficient than those at any other time in
D. more fuel-efficient small cars than those at any other time in their
E. more fuel-efficient small cars now than at any time in
Answer:
*******************
Q14:
The themes that Rita Dove explores in her poetry is universal, encompassing much of the
human condition while occasionally she deals with racial issues.
A. is universal, encompassing much of the human condition while occasionally she
deals
B. is universal, encompassing much of the human condition, also occasionally it
deals
C. are universal, they encompass much of the human condition and occasionally
deals
D. are universal, encompassing much of the human condition while occasionally
dealing
E. are universal, they encompass much of the human condition, also occasionally are
dealing
Answer:
*******************
Q15:
20
The commission’s office of compliance, inspections, and investigations plans to intensify
its scrutiny of stock analysts to investigate not only whether research is an independent
function at brokerage firms, but also whether conflicts result when analysts own the
stocks they write about or when they are paid for their work by a firm’s investment
banking division.
A. to investigate not only whether research is an independent function at brokerage
firms, but also whether conflicts result when analysts own the stocks they write
about or when they are
B. to investigate not only whether research is an independent function at brokerage
firms, but also if conflicts result when analysts own the stocks they write about or
they are
C. to not only investigate whether or not research is an independent function at
brokerage firms, but also if conflicts result when analysts own the stocks they
write about or are
D. not only to investigate whether or not research is an independent function at
brokerage firms, but also whether conflicts result when analysts own the stocks
they write about or are
E. not only to investigate whether research is an independent function at brokerage
firms, but also whether conflicts result when analysts own the stocks they write
about or when
Answer:
*******************
Q16:
Frobisher, a sixteenth-century English explorer, had soil samples from Canada’s
Kodlunarn Island examined for gold content. Because high gold content was reported,
Elizabeth I funded two mining expeditions. Neither expedition found any gold there.
Modern analysis of the island’s soil indicates a very low gold content. Thus the methods
used to determine the gold content of Frobisher’s samples must have been inaccurate.
Which of the following is an assumption on which the argument depends?
A. The gold content of the soil on Kodlunarn Island is much lower today than it was
in the sixteenth century.
B. The two mining expeditions funded by Elizabeth I did not mine the same part of
Kodlunarn Island.
C. The methods used to assess gold content of the soil samples provided by
Frobisher were different from those generally used in the sixteenth century.
D. Frobisher did not have soil samples from any other Canadian island examined for
gold content.
E. Gold was not added to the soil samples collected by Frobisher before the samples
were examined.
Answer:
*******************
Q17:
21
Scholars who once thought Native American literatures were solely oral narratives
recorded by missionaries or anthropologists now understand this body of work to consist
of both oral literatures and the written works of Native American authors, who have been
publishing since 1772.
A. Scholars who once thought Native American literatures were solely oral
narratives
B. Scholars thinking of Native American literatures once solely as oral narratives,
and
C. Scholars who once had thought of Native American literatures solely as oral
narratives and
D. Native American literatures, which some scholars once thought were solely oral
narratives
E. Native American literatures, which some scholars once, thinking they were solely
oral narratives
Answer:
*******************
Q18:
Analyzing campaign expenditures, the media has had as a focus the high costs and low
ethics of campaign finance, but they have generally overlooked the cost of actually
administering elections, which includes facilities, transport, printing, staffing, and
technology.
A. Analyzing campaign expenditures, the media has had as a focus
B. Analyses of campaign expenditures by the media has been focus on
C. In analyzing campaign expenditures, the media have focused on
D. Media analyses of campaign expenditures have had as a focus
E. In their analysis of campaign expenditures, the media has been focusing on
Answer:
*******************
Q19:
In Rubaria, excellent health care is available to virtually the entire population, whereas
very few people in Terland receive adequate medical care. Yet, although the death rate
for most diseases is higher in Terland than in Rubaria, the percentage of the male
population that dies from prostate cancer is significantly higher in Rubaria than in
Terland.
Which of the following, if true, most helps to explain the disparity between the prostate
cancer death rate in Rubaria and Terland?
A. Effective treatment of prostate cancer in its early stages generally requires
medical techniques available in Rubaria but not in Terland.
B. Most men who have prostate cancer are older than the average life expectancy for
male inhabitants of Terland.
C. Being in poor general health does not increase one’s risk of developing prostate
cancer.
22
D. It is possible to decrease one’s risk of getting prostate cancer by eating certain
kinds of foods, and such foods are more readily available in Rubaria than in
Terland.
E. Among men in Rubaria, the death rate from prostate cancer is significantly higher
for those who do not take full advantage of Rubaria’s health care system than for
those who do.
Answer:
*******************
Q20 to 23:
Recently biologists have been Just before the sand is inundated by
interested in a tide-associated the rising tide, the diatoms burrow
periodic behavior displayed by (15) again. Some scientists hypothesize
Line the diatom Hantzschia virgata, a that commuter diatoms know that it
(5) microscopic golden-brown alga that is low tide because they sense an
inhabits that portion of a shoreline environmental change, such as an
washed by tides (the intertidal zone). alteration in temperature or a change
Diatoms of this species, sometimes (20) in pressure caused by tidal
called “commuter” diatoms, remain movement.
(10) burrowed in the sand during high However, when diatoms are
tide, and emerge on the sand surface observed under constant conditions
during the daytime low tide. in a laboratory, they still display
periodic behavior, continuing to bur- (45) factors—including changes in the
(25) row on schedule for several weeks. tide’s hydrostatic pressure, salinity,
This indicates that commuter diatoms, mechanical agitation, and
rather than relying on environmental temperature—can alter the period
cues to keep time, possess an internal of its biological clock according to
pacemaker or biological clock (50) changes in the tidal cycle. In short,
(30) that enables them to anticipate the relation between an organism’s
periodic biological clock and its environment
changes in the environment. is similar to that between a wristwatch
A commuter diatom has an unusually and its owner: the owner cannot
accurate biological clock, a (55) make the watch run faster or slower,
consequence of the unrelenting but can reset the hands. However,
(35) environmental pressures to which this relation is complicated in intertidal
it is subjected; any diatoms that do dwellers such as commuter diatoms
not burrow before the tide arrives by the fact that these organisms are
23 (60) exposed to the solar-day cycle as
are washed away. well as to the tidal cycle, and sometimes
This is not to suggest that the display both solar-day and
(40) period of this biological clock is tidal periods in a single behavior.
immutably fixed. Biologists have Commuter diatoms, for example,
concluded that even though a (65) emerge only during those low tides
diatom does not rely on the environment that occur during the day.
to keep time, environmental
--------------------------------------------------------------------------------
Q20:
The passage suggests which of the following about the accuracy of the commuter
diatom’s biological clock?
A. The accuracy of the commuter diatom’s biological clock varies according to
changes in the tidal cycle.
B. The unusual accuracy that characterizes the commuter diatom’s biological clock
is rare among intertidal species.
C. The commuter diatom’s biological clock is likely to be more accurate than the
biological clock of a species that is subject to less intense environmental
pressures.
D. The commuter diatom’s biological clock tends to be more accurate than the
biological clocks of most other species because of the consistency of the tidal
cycle.
E. The accuracy of the commuter diatom’s biological clock tends to fluctuate when
the diatom is observed under variable laboratory conditions.
24
Answer:
--------------------------------------------------------------------------------
Q21:
Which of the following can be inferred from the passage about the effect of the solar-day
cycle on the tide-associated periodic behavior displayed by commuter diatoms?
A. The solar-day cycle makes this behavior less advantageous to the commuter
diatoms at certain times of the year.
B. The solar-day cycle makes this behavior somewhat more erratic.
C. The solar-day cycle makes this behavior less important to the survival of the
commuter diatoms.
D. Because of the solar-day cycle, this behavior is forced to proceed at a more rapid
pace at certain times during the 24-hour day.
E. Because of the solar-day cycle, this behavior is not expressed at certain times
during the 24-hour day.
Answer:
--------------------------------------------------------------------------------
Q22:
According to the passage, the periodic behavior displayed by commuter diatoms under
constant laboratory conditions is characterized by which of the following?
A. Greater unpredictability than the corresponding behavior under natural conditions
B. A consistent periodic schedule in the short term
C. No difference over the long term from the corresponding behavior under natural
conditions
D. Initial variability caused by the constant conditions of the laboratory
E. Greater sensitivity to environmental factors than is the case under natural
conditions
Answer:
--------------------------------------------------------------------------------
QX:
According to the passage, each of the following is characteristic of the tide-associated
periodic behavior of commuter diatoms EXCEPT:
A. It is triggered when the diatoms are inundated by the tide.
B. It is correlated with the rise and fall of the tide.
C. It adjusts to changes in the tidal cycle.
D. It is influenced by the solar-day cycle.
E. It is regulated by an innate time-keeping mechanism.
Answer:
--------------------------------------------------------------------------------
Q23:
The author of the passage compares the relationship between an organism’s biological
clock and its environment to the relation between a wristwatch and its owner most
probably in order to
25
A. point out a fundamental difference between the function of biological clocks in
organisms and the use of mechanical clocks by humans
B. illustrate the way in which the period of an organism’s biological clock can be
altered by environmental factors
C. suggest that there are important similarities between the biological clock in
organisms such as the commuter diatom and the biological clock in humans
D. support an argument regarding the methods used by certain organisms to
counteract the influence of the environment on their biological clocks
E. question the accuracy of the biological clock in organisms such as the commuter
diatom
Answer:
*******************
Q24 to Q27:
Anole lizard species that at a time when the islands
occur together (sympatrically) were connected, which some
on certain Caribbean islands of these islands may once
Line occupy different habitats: (45) have been. After the islands
(5) some live only in the grass, separated, the isolated lizard
some only on tree trunks, and populations would have become
some only on twigs. These distinct species while also
species also differ morphologically: retaining their ancestors’ niche
grass dwellers are (50) adaptations. Both of these
(10) slender with long tails, tree scenarios imply that specialization
dwellers are stocky with long to each niche occurred
legs, twig dwellers are slender only once. Alternatively, each
but stubby-legged. What is specialist could have arisen
striking about these lizards (55) independently on each of the
(15) is not that coexisting species islands.
differ in morphology and habitat If each type of specialist
use (such differences are evolved just once, then similar
common among closely related specialists on different islands
sympatric species), but that (60) would be closely related.
(20) the same three types of habitat Conversely, if the specialists
specialists occur on each evolved independently on each
of four islands: Puerto Rico, island, then a specialist on one
Cuba, Hispaniola, and Jamaica. island would be more closely
Moreover, the Puerto Rican (65) related to other types of anoles
(25) twig species closely resembles on the same island—regardless
the twig species of Cuba, of their ecological niches—
Hispaniola, and Jamaica in than it would be to a similar
morphology, habitat use, and specialist on a different island.
behavior. Likewise, the spe- (70) Biologists can infer how
(30) cialists for other habitats are species are related evolutionarily
similar across the islands. by comparing DNA
The presence of similar sequences for the same genes
species on different islands in different species. Species
26 (75) with similar DNA sequences
could be variously explained. for these genes are generally
(35) An ancestral species might more closely related to each
have adapted to exploit a other than to species with lesssimilar
particular ecological niche on DNA sequences. DNA
one island and then traveled 27
over water to colonize other (80) evidence concerning the anoles
(40) islands. Or this ancestral led researchers to conclude
species might have evolved that habitat specialists on one
island are not closely related evolved independently
to the same habitat specialists on each island.
(85) elsewhere, indicating that
specialists
--------------------------------------------------------------------------------
Q24:
The primary purpose of the passage is to
A. describe some unusual features of anole lizard species
B. account for a particular type of behavior found among anole lizard species
C. contrast two types of evidence that have been used to support a particular
hypothesis concerning anole lizard species
D. explain how researchers resolved a particular scientific question concerning anole
lizard species
E. examine different explanations for a particular trait common to certain anole
lizard species
Answer:
--------------------------------------------------------------------------------
Q25:
Which of the following best describes the purpose of the sentence in lines 13-23 (“What
is … Jamaica”)?
A. It raises a question about why coexisting anole lizard species occupy the different
types of habitats mentioned in the first sentence.
B. It introduces a fact about anole lizard species that the passage will go on to
explore.
C. It identifies a particular aspect of anole lizard behavior that distinguishes anoles
from other lizard species.
D. It explains why one aspect of anole lizard species’ habitat use has been difficult to
account for.
E. It points out a surprising relationship between morphology and habitat use that is
explained in the concluding paragraph.
Answer:
--------------------------------------------------------------------------------
Q26:
It can be inferred form the passage that which of the following is true of the Cuban
treedwelling
anole lizard and the Jamaican tree-dwelling anole lizard?
A. They share a morphology characterized by stocky bodies and long legs.
B. They have bodies that are relatively slender compared to their stubby legs.
C. They differ significantly form one another in size.
D. They differ significantly from one another in behavior and habitat use.
28
E. They are genetically closely related to one another.
Answer:
--------------------------------------------------------------------------------
Q27:
The passage suggests that if a grass-dwelling anole lizard species evolved on one island
and then traveled over water to colonize a second island, the grass-dwelling anoles on the
two islands would eventually
A. develop very different DNA sequences
B. develop into different species that are more distantly related to each other than to
tree- and twig-dwelling anoles on their own islands
C. come to differ significantly from one another in habitat use
D. develop into different, but closely related, species
E. evolve significant morphological differences
Answer:
*******************
Q28:
In the sixteenth century, the push for greater precision in measuring time was not, like
more recently, motivated by complicated philosophical questions about the nature of
matter and the universe, but the practical matters of navigation: sailors simply needed
more highly accurate timepieces in order to compute their longitude form the positions of
the stars.
A. not, like more recently, motivated by complicated philosophical questions about
the nature of matter and the universe, but the practical matters of navigation
B. being motivated by the practical matters of navigation, instead of complicated
philosophical questions about the nature of matter and the universe, as it has been
recently
C. motivated not by complicated philosophical questions about the nature of matter
and the universe, like they were more recently, but by the practical matters of
navigation
D. motivated by the practical matters of navigation, not complicated philosophical
questions about the nature of matter and the universe, which was the case more
recently
E. motivated not by complicated philosophical questions about the nature of matter
and the universe, as has been the case more recently, but by the practical matters
of navigation
Answer:
*******************
Q29:
As Charles Darwin observed, natural selection operates whenever individuals of one
genetic composition are better at reproducing than that of others.
A. that of others
B. the other
C. another
29
D. those of another
E. that of the next
Answer:
*******************
Q30:
Editorial:
In Ledland, unemployed adults receive government assistance. To reduce
unemployment, the government proposes to supplement the income of those who accept
jobs that pay less than government assistance, thus enabling employers to hire workers
cheaply. However, the supplement will not raise any worker’s income above what
government assistance would provide if he or she were not gainfully employed.
Therefore, unemployed people will have no financial incentive to accept jobs that would
entitle them to the supplement.
Which of the following, if true about Ledland, most seriously weakens the argument of
the editorial?
A. The government collects no taxes on assistance it provides to unemployed
individuals and their families.
B. Neighboring countries with laws that mandate the minimum wage an employer
must pay an employee have higher unemployment rates than Ledland currently
has.
C. People who are employed and look for a new job tend to get higher-paying jobs
than job seekers who are unemployed.
D. The yearly amount unemployed people receive from government assistance is less
than the yearly income that the government defines as the poverty level.
E. People sometimes accept jobs that pay relatively little simply because they enjoy
the work.
Answer:
*******************
Q31:
Under high pressure and intense heat, graphite, the most stable form of pure carbon,
changes into the substance commonly referred to as diamond and remaining this way
whether or not the heat and pressure are removed.
A. remaining this way whether or not
B. remaining like that even as
C. remaining as such whether or not
D. remains in this way although
E. remains thus even when
Answer:
*******************
Q32:
Editorial:
30
An arrest made by a Midville police officer is provisional until the officer has taken the
suspect to the police station and the watch commander has officially approved the arrest.
Such approval is denied if the commander judges that the evidence on which the
provisional arrest is based is insufficient. A government efficiency expert has observed
that almost all provisional arrests meet the standards for adequacy of evidence that
the watch commanders enforce. The expert has therefore recommended that because
the officers’ time spent obtaining approval is largely wasted, the watch commander’s
approval no longer be required. This recommendation should be rejected as dangerous,
however, since there is no assurance that the watch commanders’ standards will continue
to be observed once approval is longer required.
In the editorial, the two portions in boldface play which of the following roles?
A. The first is a claim, the accuracy of which is disputed by the editorial; the second
is a conclusion drawn in order to support the main conclusion of the editorial.
B. The first is an observation that the editorial disputes; the second is a conclusion
that was drawn from that observation.
C. The first is a finding that was used in support of a proposal that the editorial
opposes; the second is a judgment that was based on that finding and in turn was
used to support the proposal.
D. The first is a finding introduced to support the main conclusion of the editorial;
the second is that main conclusion.
E. The first is a conclusion, the evidence for which the editorial evaluates; the
second is part of the evidence cited in favor of that conclusion.
Answer:
*******************
Q33:
For the first time in the modern era, non-Hispanic Whites are officially a minority in
California, which amounts to a little less than half the population of the state, down from
nearly three-quarters only a decade ago.
A. which amounts to a little less than half the population of the state, down from
nearly three-quarters only a decade ago
B. which amounts to a little less than half the population of the state, down from a
decade ago, when it was nearly three-quarters
C. and that amounts to a little less than half the population of the state, down from a
decade ago, when they were nearly three-quarters
D. amounting to a little less than half the population of the state, down from nearly
three-quarters a decade ago
E. amounting to a little less than half the population of the state, down from what it
was a decade ago by nearly three-quarters
Answer:
*******************
Q34:
31
Imported into Massachusetts from Europe in 1869, the gypsy moth was used by a French
scientist in an attempt at developing a strong strain of silk-producing insects, crossing
gypsy moths with adult silkworms.
A. Imported into Massachusetts from Europe in 1869, the gypsy moth was used by a
French scientist in an attempt at developing a strong strain of silk-producing
insects, crossing gypsy moths with adult silkworms.
B. Imported into Massachusetts from Europe in 1869, a French scientist was
attempting to develop a strong strain of silk-producing insects by crossing gypsy
moths with adult silkworms.
C. To cross gypsy moths with adult silkworms, in attempting the development of a
strong strain of silk-producing insects, a French scientist in 1869 imported the
gypsy moth into Massachusetts from Europe.
D. The gypsy moth was imported into Massachusetts from Europe in 1869 by a
French scientist attempting to develop a strong strain of silk-producing insects by
crossing gypsy moths with adult silkworms.
E. In an attempt at the development of a strong strain of silk-producing insects, a
French scientist, importing the gypsy moth from Europe into Massachusetts in
1869 in order to cross gypsy moths and adult silkworms.
Answer:
*******************
Q35 to Q37:
Citing the fact that the real gross worked for wages in 1997 whereas
domestic product (GDP) per capita was only forty percent worked for wages
higher in 1997 than ever before, some between 1873 and 1973. If the growth
Line journalists have argued that the rate of labor productivity (output per
United 32
(5) States economy performed ideally in (25) hour of goods and services) in 1997
1997. However, the real GDP is almost had equaled its average growth rate
always higher than ever before; it falls between 1873 and 1973 of more than
only during recessions. One point two percent, then, given the
these journalists overlooked is that in proportionately
(10) 1997, as in the twenty-four years larger workforce that existed in
immediately (30) 1997, real GDP per capita in 1997
preceding it, the real GDP per would
capita grew nearly one-half percent a have been higher than it actually was,
year more slowly than it had on average since output is a major factor in GDP.
between 1873 and 1973. Were the However, because labor productivity
(15) 1997 economy as robust as claimed, grew by only one percent in 1997, real
the growth rate of real GDP per capita (35) GDP per capita grew more slowly
in 1997 would have surpassed the in
average growth rate of real GDP per 1997 than it had on average between
capita between 1873 and 1973 because 1873 and 1973.
(20) over fifty percent of the population
--------------------------------------------------------------------------------
Q35:
The passage is primarily concerned with
A. comparing various measures used to assess the performance of the United States
economy in 1997
B. providing evidence that the performance of the United States economy in 1997
was similar to its performance between 1873 and 1973
C. evaluating an argument concerning the performance of the United States economy
in1997
D. examining the consequences of a popular misconception about the performance of
the United States economy in 1997
E. supporting an assertion made by journalists about the performance of the United
States economy in 1997
Answer:
--------------------------------------------------------------------------------
Q36:
According to the passage, which of the following is true of the average rate at which real
GDP per capita grew in the twenty-four years immediately before 1997?
A. It was less than it had been between 1873 and 1973 because only forty percent of
the population worked for wages between 1873 and 1973.
B. It was less than it had been between 1873 and 1973 because labor productivity
grew less between 1973 and 1997 than it had between 1873 and 1973.
C. It was less than it had been between 1873 and 1973 as a result of an increase in
the percentage of the population earning wages during these years.
D. It was less than the average rate at which real GDP per capita grew between 1873
and 1973.
E. It was less than the rate at which real GDP per capita grew in 1997.
Answer:
--------------------------------------------------------------------------------
Q37:
33
It can be inferred from the passage that which of the following is the reason that the
author faults the journalists referred to in line 4?
A. They believe that the real GDP per capita in 1997 was higher than the real GDP
per capita had ever been before.
B. They argue that the rate at which real GDP per capita grew in 1997 was faster
than the average rate at which it had grown between 1873 and 1973.
C. They overestimate the effect of labor productivity on the real GDP per capita in
1997.
D. They overestimate the amount by which real GDP per capita in 1997 surpassed
real GDP per capita in earlier years.
E. They fail to consider the real GDP per capita in 1997 within an appropriate
historical context.
Answer:
*******************
Q38:
Retail sales rose 8/10 of 1 percent in August, intensifying expectations that personal
spending in the July-September quarter more than doubled that of the 1.4 percent growth
rate in personal spending for the previous quarter.
A. that personal spending in the July-September quarter more than doubled that of
B. that personal spending in the July-September quarter would more than double
C. of personal spending in the July-September quarter, that it more than doubled
D. of personal spending in the July-September quarter more than doubling that of
E. of personal spending in the July-September quarter, that it would more than
double that of
Answer:
*******************
Q39:
Because mining and refining nickel is costly, researchers have developed an alternative
method for extracting nickel using Streptanthus polygaloides, a plant that absorbs and
stores nickel form the soil as it grows. The researchers incinerated a crop of Streptanthus
they grew in nickel-rich soil. By chemically extracting nickel from the ash, they
produced 100 pounds of nickel per acre of land at a total cost per pound slightly above
that of current mining.
Which of the following, if true, most strongly supports the conclusion that the use of
Streptanthus to extract nickel will be commercially adopted?
A. The season in which the researchers grew Streptanthus was an unusually
favorable one, with the right amount of precipitation to maximize the growth rate
of Streptanthus.
B. Because lowering the concentration of nickel in the soil can make land much
better for agriculture in general, a plot on which Streptanthus has been grown and
harvested can be sold for substantially more than it cost.
34
C. More air pollution is generated for each pound of nickel produced by extracting it
from Streptanthus than is generated using conventional mining and refining.
D. The land on which the researchers planted Streptanthus was unusually free of the
various weeds that can compete with Streptanthus for water, nutrients, and
sunlight.
E. It is extremely rare for soil to contain higher concentrations of nickel than the
concentrations present in the researchers’ experimental plot.
Answer:
*******************
Q40:
Parland’s alligator population has been declining in recent years, primarily because of
hunting. Alligators prey heavily on a species of freshwater fish that is highly valued as
food by Parlanders, who had hoped that the decline in the alligator population would lead
to an increase in the numbers of these fish available for human consumption. Yet the
population of this fish species has also declined, even though the annual number caught
for human consumption has not increased.
Which of the following, if true, most helps to explain the decline in the population of the
fish species?
A. The decline in the alligator population has meant that fishers can work in some
parts of lakes and rivers that were formerly too dangerous.
B. Over the last few years, Parland’s commercial fishing enterprises have increased
the number of fishing boats they use.
C. Many Parlanders who hunt alligators do so because of the high market price of
alligator skins, not because of the threat alligators pose to the fish population.
D. During Parland’s dry season, holes dug by alligators remain filled with water long
enough to provide a safe place for the eggs of this fish species to hatch.
E. In several neighboring countries through which Parland’s rivers also flow,
alligators are at risk of extinction as a result of extensive hunting.
Answer:
*******************
Q41:
The normative model of strategic decision-making suggests that executives examine a
firm’s external environment and internal conditions, and in using the set of objective
criteria they derive from these analyses, can decide on a strategy.
A. conditions, and in using the set of objective criteria they derive from these
analyses, can decide
B. conditions, and they use the set of objective criteria derived from these analyses in
deciding
C. conditions and, in using the set of objective criteria derived from these analyses,
deciding
D. conditions and, using the set of objective criteria derived from these analyses,
decide
35
E. conditions and, in their use of the set of objective criteria they derive from these
analyses, they decide
Answer:
*******************
*******************
Answers:
DCCEB, CDCBA, EECDA, EACBC, EB(A)BDB, ADEDC, DCDDC, DEBBD, D
AWA:
AA
Read the statement and the instructions that follow it,and then make any notes that will
help you plan your response.Begin typing your response in the box at the bottom of the
screen.
The following appeared in the editorial section ~f a corporate newsleuer,
"The common notion that workers are generally apathetic about management/ssucs is
false, or at least outdated: a recently published survey indicates that 79 pal~eent of the
neatly 1,200 workers who responded to aurvey questionnaires expreased a high level of
interest in the topics of corporate restructuring and redesign of benefits programs."
Discuss how well re,x~oned.., etc.
AI
Read the statement and the instructions that follow it, and then make any notes that will
help you plan your response. Begin typing your response in the box at the bottom of the
screen.
"When judging the qualifications of potential employees, business employers should rely
solely on objective information, such as a candidate's resume and education. Personal
interviews are much too subjective and are therefore not a valid basis on which to judge a
person's qualifications for a job."
Discuss the extent to which you agree or disagree with the opinion stated above. Support
your views with reasons and/or examples from your own experience, observations, or
reading.

MATH E. 96
*******************---------- Answer:
Q1: *******************----------
If Henry were to add 5 gallons of water to a Q2:
tank that is already 3/4 full of water, the tank The function f is defined for each positive
would three-digit integer n by f(n) = 2x3y5z, where x,
be 7/8 full. How many gallons of water would y and z
the tank hold if it were full? are the hundreds, tens, and units digits of n,
A. 25 respectively. If m and v are three-digit positive
B. 40 integers such that f(m) = 9f(v), then m-v = ?
C. 64 A. 8
D. 80 B. 9
C. 18 A. Statement (1) ALONE is sufficient, but
D. 20 statement (2) alone is not sufficient.
E. 80 B. Statement (2) ALONE is sufficient, but
Answer: statement (1) alone is not sufficient.
*******************---------- C. BOTH statements TOGETHER are
Q3: sufficient, but NEITHER statement ALONE
At a certain food stand, the price of each apple is
is ¢ 40 and the price of each orange is ¢ 60. sufficient.
Mary D. EACH statement ALONE is sufficient.
selects a total of 10 apples and oranges from E. Statements (1) and (2) TOGETHER are
the food stand, and the average (arithmetic NOT sufficient.
mean) Answer:
price of the 10 pieces of fruit is ¢ 56. How *******************----------
many oranges must Mary put back so that the Q6:
average If d - 9 = 2d, then d =
price of the pieces of fruit that she keeps is ¢ A. -9
52? B. -3
A. 1 C. 1
B. 2 D. 3
C. 3 E. 9
D. 4 Answer:
E. 5 *******************----------
Answer: Q7:
*******************---------- There are 11 women and 9 men in a certain
Q4: club. If the club is to select a committee of 2
Professor Vasquez gave a quiz to two classes. women
Was the range of scores for the first class equal and 2 men, how many different such
to committees are possible?
the range of scores for the second class? A. 120
(1) In each class, the number of students B. 720
taking the quiz was 26, and the lowest score in C. 1,060
each class was 70. D. 1,520
(2) In each class, the average (arithmetic E. 1,980
mean) score on the quiz was 85. Answer:
A. Statement (1) ALONE is sufficient, but *******************----------
statement (2) alone is not sufficient. Q8:
B. Statement (2) ALONE is sufficient, but In the figure shown, two identical squares are
statement (1) alone is not sufficient. inscribed in the rectangle. If the perimeter of
C. BOTH statements TOGETHER are the
sufficient, but NEITHER statement ALONE rectangle is 18 2, then what is the perimeter
is of each square?
sufficient. A. 8 2
D. EACH statement ALONE is sufficient. B. 12
E. Statements (1) and (2) TOGETHER are C. 12 2
NOT sufficient. D. 16
Answer: E. 18
*******************---------- Answer:
Q5: *******************----------
If x and y are positive integers, what is the Q9:
value of x? Is the measure of one of the interior angles of
(1) 3x5y = 1,125 quadrilateral ABCD equal to 60 degrees?
(2) y = 3 (1) Two of the interior angles of ABCD are
right angles.
(2) The degree measure of angle ABC is twice E. Statements (1) and (2) TOGETHER are
the degree measure of angle BCD. NOT sufficient.
A. Statement (1) ALONE is sufficient, but Answer:
statement (2) alone is not sufficient. *******************----------
B. Statement (2) ALONE is sufficient, but Q12:
statement (1) alone is not sufficient. Working alone at its constant rate, machine K
C. BOTH statements TOGETHER are took 3 hours to produce
sufficient, but NEITHER statement ALONE 4
is 1
sufficient. of the units produced
D. EACH statement ALONE is sufficient. last Friday. Then machine M started working
E. Statements (1) and (2) TOGETHER are and the two machines, working simultaneously
NOT sufficient. at
Answer: their respective constant rates, took 6 hours to
*******************---------- produce the rest of the units produced last
Q10: Friday.
$10,000 is deposited in a certain account that How many hours would it have taken machine
pays r percent annual interest compounded M, working alone at its constant rate, to
annually, produce all
the amount D(t), in dollars, that the deposit of the units produced last Friday?
will grow to in t years is given by D(t) = A. 8
10,000 B. 12
{1+(r/100)}t. What amount will the deposit C. 16
grow to in 3 years? D. 24
(1) D(t) = 11,000 E. 30
(2) r =10 Answer:
A. Statement (1) ALONE is sufficient, but *******************
statement (2) alone is not sufficient. Q13:
B. Statement (2) ALONE is sufficient, but If x + y is an integer, is y an integer?
statement (1) alone is not sufficient. (1) x – y is an integer.
C. BOTH statements TOGETHER are (2) x + 2y is an integer.
sufficient, but NEITHER statement ALONE A. Statement (1) ALONE is sufficient, but
is statement (2) alone is not sufficient.
sufficient. B. Statement (2) ALONE is sufficient, but
D. EACH statement ALONE is sufficient. statement (1) alone is not sufficient.
E. Statements (1) and (2) TOGETHER are C. BOTH statements TOGETHER are
NOT sufficient. sufficient, but NEITHER statement ALONE
Answer: is
*******************---------- sufficient.
Q11: D. EACH statement ALONE is sufficient.
If a and b are integers, is b even? E. Statements (1) and (2) TOGETHER are
(1) 3a + 4b is even. NOT sufficient.
(2) 3a + 5b is even. Answer:
A. Statement (1) ALONE is sufficient, but *******************
statement (2) alone is not sufficient. Q14:
B. Statement (2) ALONE is sufficient, but A certain computer program generates a
statement (1) alone is not sufficient.
sequence of numbers a1, a2, … , an such that
C. BOTH statements TOGETHER are
sufficient, but NEITHER statement ALONE a1 = a2 = 1
is and ak = ak-1 + 2ak-2 for all integers k such that
sufficient.
3 k n. If n 6, then a7 = ?
D. EACH statement ALONE is sufficient.
A. 32
B. 43 A. 3
C. 64 B. 6
D. 100 C. 12
E. 128 D. 18
Answer: E. 24
*******************---------- Answer:
Q15: *******************----------
At a certain bookstore, the regular price of Q18:
each book is 20 percent less than its list price. If k is a positive integer, is k the square of an
If integer?
during a sale the price of each book at the (1) k is divisible by 4.
store was 15 percent less than its regular price, (2) k is divisible by exactly 4 different prime
then the numbers.
price of a book during the sale was what A. Statement (1) ALONE is sufficient, but
percent less than its list price? statement (2) alone is not sufficient.
A. 30% B. Statement (2) ALONE is sufficient, but
B. 32% statement (1) alone is not sufficient.
C. 35% C. BOTH statements TOGETHER are
D. 38% sufficient, but NEITHER statement ALONE
E. 40% is
Answer: sufficient.
*******************---------- D. EACH statement ALONE is sufficient.
Q16: E. Statements (1) and (2) TOGETHER are
What is the value of the ratio of x to y2? NOT sufficient.
(1) The ratio of x2 to y is 5/3. Answer:
(2) The ratio of x to 1 is 5/3. *******************----------
A. Statement (1) ALONE is sufficient, but Q19:
statement (2) alone is not sufficient. If n is positive, which of the following is equal
B. Statement (2) ALONE is sufficient, but to 1 / { (n+1) - n}?
statement (1) alone is not sufficient. A. 1
C. BOTH statements TOGETHER are B. (2n+1)
sufficient, but NEITHER statement ALONE C. (n+1) / n
is D. (n+1) - n
sufficient. E. (n+1) + n
D. EACH statement ALONE is sufficient. Answer:
E. Statements (1) and (2) TOGETHER are *******************----------
NOT sufficient. Q20:
Answer: Bob, Mia, and Lou each own a number of
*******************---------- shares of stock in a company. How many
Q17: shares does
Box W and Box V each contain several blue Lou own?
sticks and red sticks, and all of the red sticks (1) Bob and Mia together own a total of 800
have the shares.
same length. The length of each red stick is 18 (2) Bob and Lou together own a total of 500
inches less than the average length of the shares.
sticks A. Statement (1) ALONE is sufficient, but
in Box W and 6 inches greater than the statement (2) alone is not sufficient.
average length of the sticks in Box V. What is B. Statement (2) ALONE is sufficient, but
the statement (1) alone is not sufficient.
average (arithmetic mean) length, in inches, of C. BOTH statements TOGETHER are
the sticks in Box W minus the average length, sufficient, but NEITHER statement ALONE
in is
inches, of the sticks in Box V? sufficient.
D. EACH statement ALONE is sufficient. By what percent was the price of a certain
E. Statements (1) and (2) TOGETHER are television set discounted for a sale?
NOT sufficient. (1) The price of the television set before it was
Answer: discounted for the sale was 25 percent
*******************---------- greater than the discounted price.
Q21: (2) The price of the television set was
In the xy-plane, what is the x-intercept of a discounted by $60 for the sale.
certain line? A. Statement (1) ALONE is sufficient, but
(1) The line passes through the point (0,2). statement (2) alone is not sufficient.
(2) The y-intercept of the line is 2. B. Statement (2) ALONE is sufficient, but
A. Statement (1) ALONE is sufficient, but statement (1) alone is not sufficient.
statement (2) alone is not sufficient. C. BOTH statements TOGETHER are
B. Statement (2) ALONE is sufficient, but sufficient, but NEITHER statement ALONE
statement (1) alone is not sufficient. is
C. BOTH statements TOGETHER are sufficient.
sufficient, but NEITHER statement ALONE D. EACH statement ALONE is sufficient.
is E. Statements (1) and (2) TOGETHER are
sufficient. NOT sufficient.
D. EACH statement ALONE is sufficient. Answer:
E. Statements (1) and (2) TOGETHER are *******************----------
NOT sufficient. Q25:
Answer: For a certain race, 3 teams were allowed to
*******************---------- enter 3 members each. A team earned 6 – n
Q22: points
B whenever one of its members finished in nth
A place, where 1 n 5. There were no ties,
What is the volume of the cube above? disqualifications, or withdrawals. If no team
(1) The surface area of the cube is 600 square earned more than 6 points, what is the least
inches. possible
(2) The length of diagonal AB is 10 3 inches. score a team could have earned?
A. Statement (1) ALONE is sufficient, but A. 0
statement (2) alone is not sufficient. B. 1
B. Statement (2) ALONE is sufficient, but C. 2
statement (1) alone is not sufficient. D. 3
C. BOTH statements TOGETHER are E. 4
sufficient, but NEITHER statement ALONE Answer:
is *******************----------
sufficient. Q26:
D. EACH statement ALONE is sufficient. Is x y ?
E. Statements (1) and (2) TOGETHER are (1) The ratio of x to y is 7/9.
NOT sufficient. (2) xy
Answer: A. Statement (1) ALONE is sufficient, but
*******************---------- statement (2) alone is not sufficient.
Q23: B. Statement (2) ALONE is sufficient, but
(0.8)-5 / (0.4)-4 = statement (1) alone is not sufficient.
A. 3/32 C. BOTH statements TOGETHER are
B. 5/64 sufficient, but NEITHER statement ALONE
C. 1/2 is
D. 1 sufficient.
E. 2 D. EACH statement ALONE is sufficient.
Answer: E. Statements (1) and (2) TOGETHER are
*******************---------- NOT sufficient.
Q24: Answer:
*******************---------- Q30:
Q27: A train traveled from station X to station Y at a
For each landscaping job that takes more than constant speed of 88 feet per second. Is the
4 hours, a certain contractor charges a total of distance that the train traveled from station X
r to Station Y greater than 40 miles? (1 mile =
dollars for the first 4 hours plus 0.2r dollars 5,280 feet)
for each additional hour or fraction of an hour, (1) It took less than 45 minutes for the train to
where travel from station X to Station Y.
r > 100. Did a particular landscaping job take (2) It took more than 42 minutes for the train
more than 10 hours? to travel from station X to Station Y.
(1) The contractor charged a total of $288 for A. Statement (1) ALONE is sufficient, but
the job. statement (2) alone is not sufficient.
(2) The contractor charged a total of 2.4r B. Statement (2) ALONE is sufficient, but
dollars for the job. statement (1) alone is not sufficient.
A. Statement (1) ALONE is sufficient, but C. BOTH statements TOGETHER are
statement (2) alone is not sufficient. sufficient, but NEITHER statement ALONE
B. Statement (2) ALONE is sufficient, but is
statement (1) alone is not sufficient. sufficient.
C. BOTH statements TOGETHER are D. EACH statement ALONE is sufficient.
sufficient, but NEITHER statement ALONE E. Statements (1) and (2) TOGETHER are
is NOT sufficient.
sufficient. Answer:
D. EACH statement ALONE is sufficient. *******************----------
E. Statements (1) and (2) TOGETHER are Q31:
NOT sufficient. There are 5 cars to be displayed in 5 parking
Answer: spaces with all the cars facing the same
*******************---------- direction.
Q28: Of the 5 cars, 3 are red, 1 is blue, and 1 is
In the rectangular coordinate system, points (4, yellow. If the cars are identical except for
0) and (– 4, 0) both lie on circle C. What is the color, how
maximum possible value of the radius of C ? many different display arrangements of the 5
A. 2 cars are possible?
B. 4 A. 20
C. 8 B. 25
D. 16 C. 40
E. There is no finite maximum value. D. 60
Answer: E. 125
*******************---------- Answer:
Q29: *******************----------
y Q32:
l If x is an integer, is 3x + 7 even?
2 (1) (x – 5)(x + 1) = 0
-3 0 x (2) x is a factor of 105.
The graph of which of the following equations A. Statement (1) ALONE is sufficient, but
is a straight line that is parallel to line l in the statement (2) alone is not sufficient.
figure above? B. Statement (2) ALONE is sufficient, but
A. 3y – 2x = 0 statement (1) alone is not sufficient.
B. 3y + 2x = 0 C. BOTH statements TOGETHER are
C. 3y + 2x = 6 sufficient, but NEITHER statement ALONE
D. 2y – 3x = 6 is
E. 2y + 3x = -6 sufficient.
Answer: D. EACH statement ALONE is sufficient.
*******************----------
E. Statements (1) and (2) TOGETHER are project.
NOT sufficient. A. Statement (1) ALONE is sufficient, but
Answer: statement (2) alone is not sufficient.
******************* B. Statement (2) ALONE is sufficient, but
Q33: statement (1) alone is not sufficient.
(0.8)-5 / (0.4)-4 = C. BOTH statements TOGETHER are
A. 3/32 sufficient, but NEITHER statement ALONE
B. 5/64 is
C. 1/2 sufficient.
D. 1 D. EACH statement ALONE is sufficient.
E. 2 E. Statements (1) and (2) TOGETHER are
Answer: NOT sufficient.
*******************---------- Answer:
Q34: *******************----------
A certain company that sells only cars and Q36:
trucks reported that revenues from car sales in If x2 + y2 = 1, is x + y = 1?
1997 (1) xy = 0
were down 11 percent from 1996 and revenues (2) y = 0
from truck sales in 1997 were up 7 percent A. Statement (1) ALONE is sufficient, but
from statement (2) alone is not sufficient.
1996. If total revenues from car sales and truck B. Statement (2) ALONE is sufficient, but
sales in 1997 were up 1 percent from 1996, statement (1) alone is not sufficient.
what C. BOTH statements TOGETHER are
is the ratio of revenue from car sales in 1996 sufficient, but NEITHER statement ALONE
to revenue from truck sales in 1996? is
A. 1 : 2 sufficient.
B. 4 : 5 D. EACH statement ALONE is sufficient.
C. 1 : 1 E. Statements (1) and (2) TOGETHER are
D. 3 : 2 NOT sufficient.
E. 5 : 3 Answer:
Answer: *******************----------
*******************---------- Q37:
Q35: Of the following, which is greatest?
What is the median number of employees A. (1/5)-2
assigned per project for the projects at B. (1/3)-2
Company Z? C. 3-2
(1) 25 percent of the projects at Company Z D. 5-2
have 4 or more employees assigned to each E. 23
project. Answer:
(2) 35 percent of the projects at Company Z
have 2 or fewer employees assigned to each
*******************----------
*******************----------
Answers:
BDEEA, DEBCD, CDBBB, CEEEE, EDBAD, EBEAB, ADBAC, EA

1
Verbal Section
*******************
Q1:
The artist Renoir’s last word was “flowers,” spoken as they arranged a bouquet of roses
just picked from his garden in a vase on his bedroom windowsill.
A. as they arranged a bouquet of roses just picked from his garden
B. as a bouquet of roses had been just picked from his garden and was being
arranged
C. as a bouquet of roses just picked from his garden was being arranged
D. during the arrangement of a bouquet of roses just picked from his garden
E. while they arranged a bouquet of roses that had just been picked from his garden
Answer:
*******************
Q2:
Caribou are wary animals with excellent hearing, so stalking them over the treeless
landscape, getting close enough to kill it with nothing but a handheld lance, as Dorset
people did, required exceptional hunting skill.
A. so stalking them over the treeless landscape, getting close enough to kill it
B. so to stalk them over the treeless landscape and get close enough to kill one
C. so in order to stalk them over the treeless landscape and get close enough to kill
one
D. and so in order to stalk it over the treeless landscape, getting close enough to kill
it
E. and so stalking them over the treeless landscape and getting close enough in order
to kill it
Answer:
*******************
Q3:
Like ants, termites have an elaborate social structure in which a few individuals
reproduce and the rest are serving the colony by tending juveniles, gathering food,
building the nest, or they battle intruders.
A. are serving the colony by tending juveniles, gathering food, building the nest, or
they battle
B. are serving the colony in that they tend juveniles, gather food, build the nest, or
battle
C. serve the colony, tending juveniles, gathering food, building the nest, or by
battling
D. serve the colony by tending juveniles, gathering food, by building the nest, or by
battling
E. serve the colony by tending juveniles, gathering food, building the nest, or
battling
Answer:
*******************
2
Q4:
The three women, liberal activists who strongly support legislation in favor of civil rights
and environmental protection, have consistently received labor’s unqualifying support.
A. have consistently received labor’s unqualifying support
B. are consistently receiving the unqualifying support of labor
C. have consistently received the unqualified support of labor
D. receive consistent and unqualified support by labor
E. are receiving consistent and unqualified support by labor
Answer:
*******************
Q5:
Citing the recent increase in earnings by several computer companies, economists feel
that a cycle has begun in which personal computer users, especially corporate consumers,
are replacing their PC’s with more powerful models.
A. a cycle has begun in which personal computer users
B. a cycle for personal computer users has begun in which they
C. there is a cycle beginning for personal computer users
D. it is the beginning of a cycle for personal computer users
E. personal computer users are in the beginning of a cycle when they
Answer:
*******************
Q6 to Q9: the historical reality is not reducible
Some historians contend that conditions to one or the other.
in the United States during the (20) Unions faced a choice between
Second World War gave rise to a either maintaining the prewar status
Line dynamic wartime alliance between quo or promoting a more inclusive
(5) trade unions and the African 3
American approach that sought for all members
community, an alliance that advanced the right to participate in the internal
the cause of civil rights. They conclude (25) affairs of unions, access to skilled
that the postwar demise of this and high-paying positions within the
vital alliance constituted a lost oppor- occupational hierarchy, and protection
(10) tunity for the civil rights movement against management’s arbitrary
that authority in the workplace. While
followed the war. Other scholars, (30) union representatives often voiced
however, have portrayed organized this inclusive ideal, in practice unions
labor as defending all along the far more often favored entrenched
relatively interests. The accelerating development
privileged position of White of the civil rights movement
(15) workers relative to African (35) following the Second World War
American exacerbated the unions’ dilemma,
workers. Clearly, these two perspectives forcing trade unionists to confront
are not easily reconcilable, but contradictions in their own practices.
--------------------------------------------------------------------------------
Q6:
The passage is primarily concerned with
A. providing a context within which to evaluate opposing viewpoints about a
historical phenomenon
B. identifying a flawed assumption underlying one interpretation of a historical
phenomenon
C. assessing the merits and weaknesses of a controversial theory about a historical
phenomenon
D. discussing the historical importance of the development of a wartime alliance
E. evaluating evidence used to support a particular interpretation of a historical
phenomenon
Answer:
--------------------------------------------------------------------------------
Q7:
According to the passage, the historians mentioned in line 1 and the scholars mentioned
in line 11 disagree about the
A. contribution made by organized labor to the war effort during the Second World
War
B. issues that union members considered most important during the Second World
War
C. relationship between unions and African Americans during the Second World
War
D. effect of the Second World War on the influence of unions in the workplace
E. extent to which African Americans benefited from social and political changes
following the Second World War
Answer:
--------------------------------------------------------------------------------
4
Q8:
Which of the following best describes the purpose of the first sentence (lines 20-29) in
the second paragraph in the passage?
A. To summarize a situation confronted by unions during the Second World War
B. To summarize the role of unions in the workplace during the Second World War
C. To explain the philosophy supported by most unions during the Second World
War
D. To assess the effect of the growth of the civil rights movement on unions during
the Second World War
E. To present a criticism of the unions’ approach to representing workers during the
Second World War
Answer:
--------------------------------------------------------------------------------
Q9:
Which of the following best summarizes the opinion of the author of the passage
regarding the two points of view presented in the first paragraph (lines 1-19) ?
A. Neither point of view reflects the views of certain African American historians on
trade unions during the Second World War.
B. Neither point of view reflects the full complexity of the historical reality.
C. One point of view is based on more reliable research than is the other.
D. Both points of view have misinterpreted recent research on trade unions during
the Second World War.
E. The two points of view can be readily harmonized into a coherent interpretation.
Answer:
*******************
Q10:
A manufacturer of workstations for computer-aided design seeks to increase sales to its
most important corporate customers. Its strategy is to publish very low list prices for
workstations in order to generate interest among the buyers for those corporations.
Which of the following, if characteristic of the marketplace, would tend to cause the
manufacturer’s strategy to fail?
A. The proposed list prices would seem low to a typical buyer for the manufacturer’s
most important corporate customers.
B. The capabilities of workstations suitable for given jobs are not significantly
different among various manufacturers.
C. The manufacturer’s most important corporate customers employ as buyers
persons who are very knowledgeable about prices for workstations for customeraided
design.
D. Customers differ significantly in the percentage of resources they can devote to
computer workstations.
E. Buyers for corporations that purchase workstations for computer-aided design
receive bonuses for negotiating large discounts from the list price.
5
Answer:
*******************
Q11 to Q13:
First identified in 1969, komatiites needed to produce magmas
are Earth’s oldest known volcanic rich in magnesium could have existed
rocks and contain three times as much on Earth. Hess suggested that the
Line magnesium as do most volcanic presence of water, probably released
rocks. (25) from minerals decomposing in the
(5) This chemical composition suggests Earth’s mantle, might have meant that
that komatiites formed from the hottest a high-magnesium magma could have
lava known ever to have erupted: a existed at a lower temperature. But
high concentration of magnesium Bowen showed experimentally that the
changes the physical properties of (30) high temperatures were indeed
(10) lava so that unusually high necessary.
temperatures By 1960, it was generally
would be required for the lava accepted that volcanic rocks with such
to exist as a liquid. high levels of magnesium could not
Komatiites’ discovery was surprising exist, and thus the discovery of koma-
in light of then-current geological (35) tiites changed geologists’
(15) theories about magmas, molten rock assumptions
that forms in the Earth’s mantle (the about the characteristics of the Earth’s
layer beneath the crust) and composes mantle around the time of the formation
volcanic lava eruptions. Prior to 1960, of komatiites, between 2.5 and 4 billion
geologists Bowen and Hess disagreed years ago.
(20) over whether or not the very high
temperatures
--------------------------------------------------------------------------------
Q11:
Which of the following most accurately describes the organization of the passage?
6
A. Two divergent views of a scientific phenomenon are reconciled.
B. A phenomenon is described and its scientific significance is discussed.
C. The discovery of a scientific phenomenon is traced and its implications for further
research are suggested.
D. A long-standing scientific theory is examined and recently discovered evidence is
shown to support it.
E. The ways in which a particular geological phenomenon is exceptional are detailed
and classified.
Answer:
--------------------------------------------------------------------------------
Q12:
Information in the passage suggests which of the following concerning the Earth’s mantle
2.5 to 4 billion years ago?
A. It contained magmas that were more significantly affected by the decomposition
of minerals than are current-day magmas.
B. It contained a lower proportion of water that it contains today.
C. Its characteristics were accurately described by both Bowen and Hess.
D. Its temperature was sufficiently high to produce magmas with high magnesium
content.
E. Its total magnesium content then was roughly equivalent to its magnesium content
today.
Answer:
--------------------------------------------------------------------------------
Q13:
Which of the following most accurately states the main point of the passage?
A. Komatiites provide information about rates of volcanic eruption between 2.5 and
4 billion years ago.
B. Komatiites provide information about how the physical properties of lava in the
Earth’s past compare with those of current-day lava.
C. Komatiites provide evidence that undermines Bowen’s experimental conclusions
regarding the temperatures at which lava exists as a liquid.
D. Komatiites provide evidence that has changed geologists’ ideas about the
characteristics of the Earth’s mantle between 2.5 and 4 billion years ago.
E. Komatiites provide evidence that water in the Earth’s mantle may have reduced
the temperature required for lava to exist as a liquid.
Answer:
*******************
Q14:
Background information: This year, each film submitted to the Barbizon Film Festival
was submitted in one of ten categories. For each category, there was a panel that decided
which submitted films to accept.
Fact 1: Within each category, the rate of acceptance for domestic films was the same as
that for foreign films.
7
Fact 2: The overall rate of acceptance of domestic films was significantly higher than
that of foreign films.
In light of the background information, which of the following, if true, can account for
fact 1 and fact 2 both being true of the submissions to this year’s Barbizon Film Festival?
A. In each category, the selection panel was composed of filmmakers, and some
selection panels included no foreign filmmakers.
B. Significantly more domestic films than foreign films were submitted to the
festival.
C. In each of the past three years, the overall acceptance rate was higher for foreign
than for domestic films, an outcome that had upset some domestic filmmakers.
D. The number of films to be selected in each category was predetermined, but in no
category was it required that the acceptance rate of foreign films should equal that
of domestic films.
E. Most foreign films, unlike most domestic films, were submitted in categories with
high prestige, but with correspondingly low rates of acceptance.
Answer:
*******************
Q15:
Kernland imposes a high tariff on the export of unprocessed cashew nuts in order to
ensure that the nuts are sold to domestic processing plants. If the tariff were lifted and
unprocessed cashews were sold at world market prices, more farmers could profit by
growing cashews. However, since all the processing plants are in urban areas, removing
the tariff would seriously hamper the government’s effort to reduce urban unemployment
over the next five years.
Which of the following, if true, most seriously weakens the argument?
A. Some of the by-products of processing cashews are used for manufacturing paints
and plastics.
B. Other countries in which cashews are processed subsidize their processing plants.
C. More people in Kernland are engaged in farming cashews than in processing
them.
D. Buying unprocessed cashews at lower than world market prices enables cashew
processors in Kernland to sell processed nuts at competitive prices.
E. A lack of profitable crops is driving an increasing number of small farmers in
Kernland off their land and into the cities.
Answer:
*******************
Q16:
Researchers are using computer images to help surgeons plan difficult operations and to
develop programs that will work for doctors and nurses in the same way that flight
simulators do for pilots, letting medical personnel practice their techniques and test their
reflexes before they ever see a patient.
8
A. plan difficult operations and to develop programs that will work for doctors and
nurses in the same way that flight simulators do
B. plan difficult operations and develop programs to work for doctors and nurses the
same way as with flight simulators
C. to plan difficult operations and to develop programs that will work for doctors and
nurses like flight simulators
D. plan difficult operations and in developing programs to work for doctors and
nurses the same way as flight simulators
E. to plan difficult operations and developing programs that will work for doctors
and nurses like flight simulators do
Answer:
*******************
Q17:
By the same techniques used for genetically enhancing plants, making them disease- or
pest-resistant, researchers have been able to increase the amount of protein in potatoes,
sweet potatoes, and tobacco.
A. By the same techniques used for genetically enhancing plants, making them
B. With the same techniques to genetically enhance plants, so that they are
C. Employing the same techniques used to genetically enhance plants so that they are
D. Employing the same techniques to genetically enhance plants, which makes them
E. Employing the same techniques for genetically enhancing plants that make them
Answer:
*******************
Q18:
Several financial officers of the company spoke on condition that they not be named in
the press reports.
A. that they not be named
B. that their names will not be used
C. that their names are not used
D. of not having their names
E. of not naming them
Answer:
*******************
Q19:
Industrial accidents are more common when some of the people in safety-sensitive jobs
have drinking problems than when none do. Since, even after treatment, people who
have had drinking problems are somewhat more likely than other people to have drinking
problems in the future, any employer trying to reduce the risk of accidents should bar
anyone who has ever been treated for a drinking problem from holding a safety-sensitive
job.
Which of the following, if true, most seriously undermines the argument above?
9
A. Some companies place employees who are being treated for drinking problems in
residential programs and allow them several weeks of paid sick leave.
B. Many accidents in the workplace are the result of errors by employees who do not
hold safety-sensitive jobs.
C. Workers who would permanently lose their jobs if they sought treatment for a
drinking problem try instead to conceal their problem and continue working for as
long as possible.
D. People who hold safety-sensitive jobs are subject to stresses that can exacerbate
any personal problems they may have, including drinking problems.
E. Some industrial accidents are caused by equipment failure rather than by
employee error.
Answer:
*******************
Q20:
Researchers have announced that the magnetic fields emitted by one manufacturer’s
security surveillance system, of which type there are 200,000 worldwide, can interfere
with pacemakers and that this interaction can bring on missed or irregular heartbeats,
nausea, breathlessness, dizziness, and even fainting.
A. of which type there are 200,000 worldwide, can interfere with pacemakers and
that
B. of which, worldwide, there are 200,000 of that type, and which can interfere with
pacemakers, and
C. of which type there are 200,000 worldwide, they can interfere with pacemakers
and
D. 200,000 of which type exist worldwide, interfering with pacemakers, and
E. 200,000 of that type existing worldwide and can interfere with pacemakers, and
Answer:
*******************
Q21:
Most of the purported health benefits of tea comes from antioxidants—compounds also
found in beta carotene, vitamin E, and vitamin C that inhibit the formation of plaque
along the body’s blood vessels.
A. comes from antioxidants—compounds also found in beta carotene, vitamin E, and
vitamin C that
B. comes from antioxidants—compounds that are also found in beta carotene,
vitamin E, and vitamin C, and they
C. come from antioxidants—compounds also found in beta carotene, vitamin E, and
vitamin C, and
D. come from antioxidants—compounds that are also found in beta carotene, vitamin
E, and vitamin C and that
E. come from antioxidants—compounds also found in beta carotene, vitamin E, and
vitamin C, and they
Answer:
*******************
10
Q22 to Q25:
While acknowledging that (10) the effects of migration are
there are greater employment more complex than these
opportunities for Latin Ameri- theorists presume. For
Line can women in cities than in the example, effects can vary
(5) countryside, social science depending on women’s
theorists have continued to (15) financial condition and social
argue that urban migration class. Brazilian women in the
has unequivocally hurt lowest socioeconomic class
women’s status. However, have relatively greater job
opportunities and job security However, the structure
(20) in cities than do men of the of many poor women’s
same class, although there is work—often a labor force
no compelling evidence that (55) of one in an employer’s
for these women the move to home—makes it difficult for
the city is a move out of pov- them to organize to improve
(25) erty. Thus, these women their economic conditions in
may improve their status in general.
relation to men but at the (60) Not surprisingly, then,
same time may experience Latin American women in the
no improvement in their lowest socioeconomic class
(30) economic standing. differ in their opinions about
In addition, working outside the effects of urban migration
the home, which is more (65) on their lives. Some find
common in urban than in rural urban living, with access to
areas, helps women in the electricity and running water,
(35) lowest socioeconomic class an improvement and would
make contacts to extend never return to the country-
exchange networks—the flow (70) side. Others, disliking the
of gifts, loans, or child care overcrowding and crime,
from those who currently would return to the countryside
(40) have access to resources if there were work
to those who do not. Moreover, opportunities for them there.
poor women working in (75) Thus, urban life has had both
urban areas actively seek to negative and positive impacts
cultivate long-term employer- on women’s lives. In general,
(45) employee relations. When urban migration has not
11 provided economic pros-
an emergency arises that (80) perity or upward mobility for
requires greater resources women in the lowest socioeconomic
than an exchange network class, despite their
can provide, these women intelligent and energetic utilization
(50) often appeal for and receive of the resources
aid from their wealthy employers. available to them.
--------------------------------------------------------------------------------
Q22:
In the first paragraph, the author refers to the experiences of Brazilian women most
probably in order to
12
A. support an earlier assertion made by social science theorists about the effects of
urban migration
B. provide an example of one area in which urban migration has failed to improve
Latin American women’s lives
C. substantiate the claim that the effects of urban migration cannot be easily
characterized
D. illustrate the effect that urban migration has had on the economic status of Latin
American women
E. compare the effect that urban migration has had on the economic status of Latin
American women with its effect on the economic status of Latin American men
Answer:
--------------------------------------------------------------------------------
Q23:
NOTE: You must scroll to read the answer choices for this question.
Which of the following best summarizes the main point of the passage?
A. Although Latin American women disagree about the effects urban migration has
had on their lives, they agree that migration has provided them with greater
opportunities for stable employment outside the home.
B. Although urban migration has improved the quality of life for Latin American
women, it has weakened the social support systems that these women enjoyed in
rural communities.
C. The effects that urban migration has had on Latin American women’s lives are
complex and are best evaluated in light of a range of issues concerning Latin
American women’s overall quality of life.
D. The effects of urban migration in Latin America are different for men than they
are for women because of the relatively greater job opportunities and job security
enjoyed by women in urban areas.
E. Urban migration has led to an increasing disparity between the economic
prosperity of Latin American women in the lowest socioeconomic classes and that
of women in the higher socioeconomic classes.
Answer:
--------------------------------------------------------------------------------
Q24:
The author mentions which of the following as a disadvantage of urban employment for
Latin American women in the lowest socioeconomic group?
A. It is difficult for these women to obtain reliable, long-term employment.
B. It is difficult for these women to organize effectively in order to obtain better
wages.
C. It is difficult for these women to find employers who are supportive when
emergencies arise.
D. The structure of their jobs makes it difficult for these women to participate in
exchange networks.
13
E. Working in urban areas makes these women more vulnerable to health problems
than they would be in rural areas.
Answer:
--------------------------------------------------------------------------------
Q25:
The author of the passage would most likely agree that the opinions of the Latin
American women discussed in the third paragraph (lines 60-85) are influenced by the
A. fact that urban life has provided them with greater opportunities for upward
mobility than did rural life
B. relative importance they place on the benefits of urban exchange networks in
comparison to those of rural networks.
C. relative importance they place on the conveniences and drawbacks of urban life in
comparison to those of rural life
D. difference in the effects of urban migration on women of higher and lower
socioeconomic classes
E. difference in the effects of urban migration on men and women of the same social
and economic class
Answer:
*******************
Q26:
In countries where automobile insurance includes compensation for whiplash
injuries sustained in automobile accidents, reports of having suffered such injuries
are twice as frequent as they are in countries where whiplash is not covered. Some
commentators have argued, correctly, that since there is presently no objective test for
whiplash, spurious reports of whiplash injuries cannot be readily identified. These
commentators are, however, wrong to draw the further conclusion that in the countries
with the higher rates of reported whiplash injuries, half of the reported cases are spurious:
clearly, in countries where automobile insurance does not include compensation for
whiplash, people often have little incentive to report whiplash injuries that they actually
have suffered.
In the argument given, the two boldfaced portions play which of the following roles?
A. The first is a finding whose accuracy is evaluated in the argument; the second is
an intermediate conclusion drawn to support the judgment reached by the
argument on the accuracy of that finding.
B. The first is a finding whose accuracy is evaluated in the argument; the second is
evidence that has been used to challenge the accuracy of that finding.
C. The first is a finding whose implications are at issue in the argument; the second
is an intermediate conclusion that has been used to support a conclusion that the
argument criticizes.
D. The first is a claim that the argument disputes; the second is a narrower claim that
the argument accepts.
E. The first is a claim that has been used to support a conclusion that the argument
accepts; the second is that conclusion.
14
Answer:
*******************
Q27:
Personnel officer: The exorbitant cost of our health-insurance benefits reflects the high
dollar amount of medical expenses incurred by our employees. Employees who are out
of shape, as a group, have higher doctor bills and longer hospital stays than do their
colleagues who are fit. Therefore, since we must reduce our health-insurance costs, we
should offer a rigorous fitness program of jogging and weight lifting to all employees,
and require employees who are out of shape to participate.
The conclusion reached by the personnel officer depends on which of the following
assumptions?
A. A person who is fit would receive a routine physical checkup by a doctor less
regularly than would a person who is out of shape.
B. The medical expenses incurred by employees who are required to participate in
the fitness program would be less than those incurred by employees who are not
required to participate.
C. The strenuous activities required of out-of-shape employees by the program
would not by themselves generate medical expenses greater than any reduction
achieved by the program.
D. The fitness program would serve more employees who are out of shape than it
would employees who are fit.
E. The employees who participate in the fitness program would be away from work
because of illness less than would the employees who do not participate.
Answer:
-----------------------------------------------------------------------------------------------------------
Q28:
In countries where automobile insurance includes compensation for whiplash
injuries sustained in automobile accidents, reports of having suffered such injuries
are twice as frequent as they are in countries where whiplash is not covered.
Presently, no objective test for whiplash exists, so it is true that spurious reports of
whiplash injuries cannot be readily identified. Nevertheless, these facts do not warrant
a conclusion that has been drawn by some commentators: that in the countries with
the higher rates of reported whiplash injuries, half of the reported cases are spurious.
Clearly, in countries where automobile insurance does not include compensation for
whiplash, people often have little incentive to report whiplash injuries that they actually
have suffered.
In the argument given, the two boldfaced portions play which of the following roles?
A. The first is a claim that the argument disputes; the second is a conclusion that has
been based on that claim.
B. The first is claim that has been used to support a position that the argument
accepts; the second is a position that the argument rejects.
15
C. The first is a finding whose accuracy is evaluated in the argument; the second is
the judgment reached by the argument concerning the accuracy of the finding.
D. The first is a finding whose implications are at issue in the argument; the second
is the judgment reached by the argument concerning one alleged implication.
E. The first is a finding, the explanation of which is at issue in the argument; the
second is an objection that has been raised against the explanation that the
argument defends.
Answer:
*******************
Q29:
Researchers agreed that the study of new treatments for heart attack patients was
extremely important but more research was needed to determine that balloon angioplasty
preceded with ultrasound was or was not any better for heart attack patients than the
balloon procedure by itself.
A. more research was needed to determine that balloon angioplasty preceded with
ultrasound was or was not any better for heart attack patients than
B. more research was needed for determining whether or not balloon angioplasty
preceded by ultrasound is any better for heart attack patients than is
C. that more research was needed to determine whether balloon angioplasty preceded
by ultrasound is any better for heart attack patients than
D. that more research was needed to determine that balloon angioplasty preceded
with ultrasound was any better for heart attack patients than
E. that more research was needed for determining that balloon angioplasty preceded
by ultrasound is or is not any better for heart attack patients than is
Answer:
*******************
Q30:
An exceptionally literate people, more Icelanders publish books per capita than do the
people of any other nation.
A. more Icelanders publish books per capita than do the people of
B. more Icelandic books are published by Icelanders per capita than by the people of
C. Icelanders publish more books per capita than do the people of
D. Iceland’s per capita publication of books is higher than that by
E. the per capita publication of Icelandic books is higher than that in
Answer:
*******************
Q31:
When a ton of wheat was being moved from Buffalo to New York City before the
opening of the Erie Canal in 1825, it took three weeks and cost $100, but when the canal
was used, it took less than eight days and cost less than $6.
A. When a ton of wheat was being moved from Buffalo to New York City before the
opening of the Erie Canal in 1825, it took three weeks and cost $100, but when
the canal was used, it
16
B. Moving a ton of wheat from Buffalo to New York City before the opening of the
Erie Canal in 1825, it took three weeks and cost $100; using the canal, it
C. Before the opening of the Erie Canal in 1825, moving a ton of wheat from Buffalo
to New York City took three weeks and cost $100; on the canal the move
D. In 1825, before opening the Erie Canal, to move a ton of wheat took three weeks
from Buffalo to New York City, which cost $100; moving it on the canal
E. To move a ton of wheat in 1825 from Buffalo to New York City took three weeks
before the opening of the Erie Canal, costing $100, but in using the canal, the
move
Answer:
*******************
Q32:
A major chemical spill occurred five years ago at Baker’s Beach, the world’s sole nesting
ground for Merrick sea turtles, and prevented nearly all the eggs laid that year from
hatching. Yet the number of adult female Merricks returning to lay their eggs at Baker’s
Beach has actually increased somewhat since five years ago. Clearly, environmentalists’
prediction that the world’s Merrick population would decline as a result of the spill has
proven unfounded.
Which of the following, if true, most seriously undermines the argument offered in
refutation of the environmentalists’ prediction?
A. The chemical spill five years ago occurred at a time when there were neither
Merrick sea turtles nor Merrick sea turtle eggs on Baker’s Beach.
B. Female Merrick sea turtles begin returning to Baker’s Beach to lay their eggs
when they are ten years old.
C. Under normal conditions, only a small proportion of hatchling female Merrick sea
turtles survive in the ocean until adulthood and return to lay their eggs at Baker’s
Beach.
D. Environmental pressures unrelated to the chemical spill have caused a significant
decline in the population of one of the several species of sea birds that prey on
Merrick sea turtle eggs.
E. After the chemical spill, an environmental group rejected a proposal to increase
the Merrick sea turtle population by transferring eggs from Baker’s Beach to
nearby beaches that had not been affected by the spill.
Answer:
*******************
Q33:
In order to reduce the number of items damaged while in transit to customers, packaging
consultants recommended that the TrueSave mail-order company increase the amount of
packing material so as to fill any empty spaces in its cartons. Accordingly, TrueSave
officials instructed the company’s packers to use more packing material than before, and
the packers zealously acted on these instructions and used as much as they could.
Nevertheless, customer reports of damaged items rose somewhat.
17
Which of the following, if true, most helps to explain why acting on the consultants’
recommendation failed to achieve its goal?
A. The change in packing policy led to an increase in expenditure on packing
material and labor.
B. When packing material is compressed too densely, it loses some of its capacity to
absorb shock.
C. The amount of packing material used in a carton does not significantly influence
the ease with which a customer can unpack the package.
D. Most of the goods that TrueSave ships are electronic products that are highly
vulnerable to being damaged in transit.
E. TrueSave has lost some of its regular customers as a result of the high number of
damaged items they received.
Answer:
*******************
Q34:
Books in European libraries last longer than books in libraries in the United States
because, although the climate in Europe is fairly humid, libraries there are not subjected
to the extremes of temperature and humidity that damage collections in the United States.
A. because, although the climate in Europe is fairly humid, libraries there are not
subjected to the extremes of temperature and humidity that
B. because, although the climate in Europe is fairly humid, libraries there are not
subject of the extremes of temperature and humidity as those that
C. because, although the climate in Europe is fairly humid, libraries there are not
subjected to the extremities of temperature and humidity as
D. because the climate in Europe is fairly humid, but still libraries there are not
subject of the extremes of temperature and humidity as
E. because the climate in Europe is fairly humid, but libraries there are not subjected
to the extremities of temperature and humidity such as those that
Answer:
-----------------------------------------------------------------------------------------------------------
Q35 to Q37:
(The following was excerpted from exceptions of Malaysia
material written in 1988.) and Thailand—have even
For over a decade the approached their success. In
most common policy advice “No More NIC’s,” Robin Broad
given to developing countries (30) and John Cavanagh search for
Line by international development the reasons behind these failures,
(5) institutions has been to copy identifying far-reaching
the export-oriented path of the changes in the global economy—
newly industrializing countries, from synthetic substitutes
the celebrated NIC’s. These (35) for commodity exports to
economies—Brazil, Hong unsustainable levels of foreign
(10) Kong, Mexico, Singapore, debt—as responsible for a glut
18 economy offering little room for
South Korea, and Taiwan— new entrants. Despite these
burst into the world manufacturing (40) changes, the authors maintain,
market in the late 1960’s the World Bank and the International
and the 1970’s; by 1978 these Monetary Fund—the
(15) six economies, along with India, foremost international development
enjoyed unequaled growth institutions—have
rates for gross national product (45) continued to promote the NIC
and for exports, with exports path as the way for heavily
accounting for 70 percent of indebted developing countries
(20) the developing world’s to proceed. And yet the futility
manufactured of this approach should,
exports. It was, (50) according to the authors, be
therefore, not surprising that all too apparent so many years
dozens of other countries into a period of reduced growth
attempted to follow their model, in world markets.
(25) yet no countries—with the possible
--------------------------------------------------------------------------------
Q35:
19
Given the information in the passage, which of the following is a true statement about the
NIC’s?
A. Their economic success among developing countries has been exceeded only by
the successes of Malaysia and Thailand.
B. By 1978 they produced 70 percent of the world’s manufactured exports.
C. In the late 1970’s, their growth rates for gross national product were among the
highest in the world.
D. In recent years their development has been heavily subsidized by major
international development institutions.
E. They received conflicting policy advice from international development
institutions in the late 1960’s and the 1970’s.
Answer:
--------------------------------------------------------------------------------
Q36:
The author of the passage most clearly implies that Broad and Cavanagh disagree with
the World Bank and the International Monetary Fund about which of the following?
A. The ways in which the global economy has changed in recent years
B. The causes of the unsustainable levels of foreign debt that the developing
countries have incurred in recent years
C. The level of foreign debt that should be maintained by developing countries
D. The degree to which international development institutions should monitor the
growth of developing countries
E. The degree to which heavily indebted developing countries should emphasize
exports in their overall economic strategy
Answer:
--------------------------------------------------------------------------------
Q37:
The author mentions Malaysia and Thailand in order to
A. acknowledge the appearance of implausibility in a broad claim
B. concede the possible existence of counter-examples to a generalization
C. offer additional evidence in support of a disputed conclusion
D. illustrate the broad applicability of a hypothesis
E. admit the limited scope of a standard analysis
Answer:
*******************
Q38:
The ancient Nubians inhabited an area in which typhus occurred, yet surprisingly few of
their skeletons show the usual evidence of this disease. The skeletons do show deposits
of tetracycline, an antibiotic produced by a bacterium common in Nubian soil. This
bacterium can flourish on the dried grain used for making two staples of the Nubian diet,
beer and bread. Thus, tetracycline in their food probably explains the low incidence of
typhus among ancient Nubians.
20
Which of the following is an assumption on which the argument relies?
A. The tetracycline deposits did not form after the bodies were buried.
B. The diseases other than typhus to which the ancient Nubians were exposed would
not be affected by tetracycline.
C. Typhus is generally fatal.
D. Nubian grain became contaminated with tetracycline-producing bacteria prior to
being harvested.
E. Bread and beer were the only foods eaten by the ancient Nubians which could
have contained tetracycline.
Answer:
Compare with Q30 in Sep. 18th, 2002.
Q30:
The ancient Nubians inhabited an area in which typhus occurs, yet surprisingly few of
their skeletons show the usual evidence of this disease. The skeletons do show deposits of
tetracycline, an antibiotic produced by a bacterium common in Nubian soil. This
bacterium can flourish on the dried grain used for making two staples of the Nubian diet,
beer and bread. Thus, tetracycline in their food probably explains the low incidence of
typhus among ancient Nubians.
Which of the following is an assumption on which the argument relies?
A. Infectious diseases other than typhus to which the ancient Nubians were exposed
are unaffected by tetracycline.
B. Tetracycline is not rendered ineffective as an antibiotic by exposure to the
processes involved in making bread and beer.
C. Typhus cannot be transmitted by ingesting bread or beer contaminated with the
infectious agents of this disease.
D. Bread and beer were the only items in the diet of the ancient Nubians which could
have contained tetracycline.
E. Typhus is generally fatal.
Answer:
*******************
Q39:
In addition to being China’s first administrators, in the sense that they developed a
coherent bureaucracy for their empire, the first literate culture in East Asia were the
Shang, and they were well known for crafting ornate bronze ritual vessels.
A. the first literate culture in East Asia were the Shang, and they were well known
for crafting
B. the first literate culture in East Asia were the Shang, well known as the crafters of
C. the Shang, as the first literate culture in East Asia, is well known for its crafting
D. the Shang were the first literate culture in East Asia and are well known as the
crafters of
E. the Shang were the first literate culture in East Asia and well known for its crafted
Answer:
21
*******************
Q40:
Investment banks often have conflicting roles. They sometimes act for a client company
by raising capital from other investment institutions as advantageously as possible, but
their analysts also sometimes send unfavorable reports on the financial health of
companies for whom they are raising capital to other clients who wish to make
investments. Analyses of companies’ financial health need to be unbiased if an
investment bank is to achieve long-term success.
If the statements above are true, which of the following practices, if adopted by an
investment bank, would hinder its long-term success?
A. Evaluating and rewarding the bank’s analysts on the basis of recommendations
made by managers who are solely engaged in raising capital for clients
B. Using reports by the investment bank’s analysts to determine how best to raise
capital for a client
C. Sharing the task of raising capital for a client with other investment banks
D. Ensuring that conflicts between analysts and those who raise capital for clients are
carefully mediated and resolved by impartial arbitrators
E. Monitoring the success or failure of analysts’ current predictions about how
companies will perform financially, in order to determine the value of future
predictions
Answer:
*******************
Q41:
Elk now live almost solely in the Rocky Mountains, which would make it seem that elk
are mountain dwellers, while they once ranged over virtually all of the continental United
States except for a small strip in the extreme Southwest.
A. Elk now live almost solely in the Rocky Mountains, which would make it seem
that elk are mountain dwellers, while
B. The fact that elk now live almost solely in the Rocky Mountains would make it
seem that they are mountain dwellers, but
C. It would seem that elk would be mountain dwellers because of their living now
solely almost in the Rocky Mountains, but still
D. Now living almost solely in the Rocky Mountains, it would seem that elk were
mountain dwellers, although
E. It seems that elk would be mountain dwellers from the fact that they now live
solely almost in the Rocky Mountains, since
Answer:
*******************
Answer:
CBECA,ABABE,BDDEE,ACACD,DCCBC,CCDBC,CBBAC,EBA(B)DA,A

1
Verbal Section
*******************
Q1:
The artist Renoir’s last word was “flowers,” spoken as they arranged a bouquet of roses
just picked from his garden in a vase on his bedroom windowsill.
A. as they arranged a bouquet of roses just picked from his garden
B. as a bouquet of roses had been just picked from his garden and was being
arranged
C. as a bouquet of roses just picked from his garden was being arranged
D. during the arrangement of a bouquet of roses just picked from his garden
E. while they arranged a bouquet of roses that had just been picked from his garden
Answer:
*******************
Q2:
Caribou are wary animals with excellent hearing, so stalking them over the treeless
landscape, getting close enough to kill it with nothing but a handheld lance, as Dorset
people did, required exceptional hunting skill.
A. so stalking them over the treeless landscape, getting close enough to kill it
B. so to stalk them over the treeless landscape and get close enough to kill one
C. so in order to stalk them over the treeless landscape and get close enough to kill
one
D. and so in order to stalk it over the treeless landscape, getting close enough to kill
it
E. and so stalking them over the treeless landscape and getting close enough in order
to kill it
Answer:
*******************
Q3:
Like ants, termites have an elaborate social structure in which a few individuals
reproduce and the rest are serving the colony by tending juveniles, gathering food,
building the nest, or they battle intruders.
A. are serving the colony by tending juveniles, gathering food, building the nest, or
they battle
B. are serving the colony in that they tend juveniles, gather food, build the nest, or
battle
C. serve the colony, tending juveniles, gathering food, building the nest, or by
battling
D. serve the colony by tending juveniles, gathering food, by building the nest, or by
battling
E. serve the colony by tending juveniles, gathering food, building the nest, or
battling
Answer:
*******************
2
Q4:
The three women, liberal activists who strongly support legislation in favor of civil rights
and environmental protection, have consistently received labor’s unqualifying support.
A. have consistently received labor’s unqualifying support
B. are consistently receiving the unqualifying support of labor
C. have consistently received the unqualified support of labor
D. receive consistent and unqualified support by labor
E. are receiving consistent and unqualified support by labor
Answer:
*******************
Q5:
Citing the recent increase in earnings by several computer companies, economists feel
that a cycle has begun in which personal computer users, especially corporate consumers,
are replacing their PC’s with more powerful models.
A. a cycle has begun in which personal computer users
B. a cycle for personal computer users has begun in which they
C. there is a cycle beginning for personal computer users
D. it is the beginning of a cycle for personal computer users
E. personal computer users are in the beginning of a cycle when they
Answer:
*******************
Q6 to Q9:
Some historians contend that conditions to one or the other.
in the United States during the (20) Unions faced a choice between
Second World War gave rise to a either maintaining the prewar status
Line dynamic wartime alliance between quo or promoting a more inclusive
(5) trade unions and the African 3
American approach that sought for all members
community, an alliance that advanced the right to participate in the internal
the cause of civil rights. They conclude (25) affairs of unions, access to skilled
that the postwar demise of this and high-paying positions within the
vital alliance constituted a lost oppor- occupational hierarchy, and protection
(10) tunity for the civil rights movement against management’s arbitrary
that authority in the workplace. While
followed the war. Other scholars, (30) union representatives often voiced
however, have portrayed organized this inclusive ideal, in practice unions
labor as defending all along the far more often favored entrenched
relatively interests. The accelerating development
privileged position of White of the civil rights movement
(15) workers relative to African (35) following the Second World War
American exacerbated the unions’ dilemma,
workers. Clearly, these two perspectives forcing trade unionists to confront
are not easily reconcilable, but contradictions in their own practices.
the historical reality is not reducible
--------------------------------------------------------------------------------
Q6:
The passage is primarily concerned with
A. providing a context within which to evaluate opposing viewpoints about a
historical phenomenon
B. identifying a flawed assumption underlying one interpretation of a historical
phenomenon
C. assessing the merits and weaknesses of a controversial theory about a historical
phenomenon
D. discussing the historical importance of the development of a wartime alliance
E. evaluating evidence used to support a particular interpretation of a historical
phenomenon
Answer:
--------------------------------------------------------------------------------
Q7:
According to the passage, the historians mentioned in line 1 and the scholars mentioned
in line 11 disagree about the
A. contribution made by organized labor to the war effort during the Second World
War
B. issues that union members considered most important during the Second World
War
C. relationship between unions and African Americans during the Second World
War
D. effect of the Second World War on the influence of unions in the workplace
E. extent to which African Americans benefited from social and political changes
following the Second World War
Answer:
--------------------------------------------------------------------------------
4
Q8:
Which of the following best describes the purpose of the first sentence (lines 20-29) in
the second paragraph in the passage?
A. To summarize a situation confronted by unions during the Second World War
B. To summarize the role of unions in the workplace during the Second World War
C. To explain the philosophy supported by most unions during the Second World
War
D. To assess the effect of the growth of the civil rights movement on unions during
the Second World War
E. To present a criticism of the unions’ approach to representing workers during the
Second World War
Answer:
--------------------------------------------------------------------------------
Q9:
Which of the following best summarizes the opinion of the author of the passage
regarding the two points of view presented in the first paragraph (lines 1-19) ?
A. Neither point of view reflects the views of certain African American historians on
trade unions during the Second World War.
B. Neither point of view reflects the full complexity of the historical reality.
C. One point of view is based on more reliable research than is the other.
D. Both points of view have misinterpreted recent research on trade unions during
the Second World War.
E. The two points of view can be readily harmonized into a coherent interpretation.
Answer:
*******************
Q10:
A manufacturer of workstations for computer-aided design seeks to increase sales to its
most important corporate customers. Its strategy is to publish very low list prices for
workstations in order to generate interest among the buyers for those corporations.
Which of the following, if characteristic of the marketplace, would tend to cause the
manufacturer’s strategy to fail?
A. The proposed list prices would seem low to a typical buyer for the manufacturer’s
most important corporate customers.
B. The capabilities of workstations suitable for given jobs are not significantly
different among various manufacturers.
C. The manufacturer’s most important corporate customers employ as buyers
persons who are very knowledgeable about prices for workstations for customeraided
design.
D. Customers differ significantly in the percentage of resources they can devote to
computer workstations.
E. Buyers for corporations that purchase workstations for computer-aided design
receive bonuses for negotiating large discounts from the list price.
5
Answer:
*******************
Q11 to Q13:
First identified in 1969, komatiites needed to produce magmas
are Earth’s oldest known volcanic rich in magnesium could have existed
rocks and contain three times as much on Earth. Hess suggested that the
Line magnesium as do most volcanic presence of water, probably released
rocks. (25) from minerals decomposing in the
(5) This chemical composition suggests Earth’s mantle, might have meant that
that komatiites formed from the hottest a high-magnesium magma could have
lava known ever to have erupted: a existed at a lower temperature. But
high concentration of magnesium Bowen showed experimentally that the
changes the physical properties of (30) high temperatures were indeed
(10) lava so that unusually high necessary.
temperatures By 1960, it was generally
would be required for the lava accepted that volcanic rocks with such
to exist as a liquid. high levels of magnesium could not
Komatiites’ discovery was surprising exist, and thus the discovery of koma-
in light of then-current geological (35) tiites changed geologists’
(15) theories about magmas, molten rock assumptions
that forms in the Earth’s mantle (the about the characteristics of the Earth’s
layer beneath the crust) and composes mantle around the time of the formation
volcanic lava eruptions. Prior to 1960, of komatiites, between 2.5 and 4 billion
geologists Bowen and Hess disagreed years ago.
(20) over whether or not the very high
temperatures
--------------------------------------------------------------------------------
Q11:
Which of the following most accurately describes the organization of the passage?
6
A. Two divergent views of a scientific phenomenon are reconciled.
B. A phenomenon is described and its scientific significance is discussed.
C. The discovery of a scientific phenomenon is traced and its implications for further
research are suggested.
D. A long-standing scientific theory is examined and recently discovered evidence is
shown to support it.
E. The ways in which a particular geological phenomenon is exceptional are detailed
and classified.
Answer:
--------------------------------------------------------------------------------
Q12:
Information in the passage suggests which of the following concerning the Earth’s mantle
2.5 to 4 billion years ago?
A. It contained magmas that were more significantly affected by the decomposition
of minerals than are current-day magmas.
B. It contained a lower proportion of water that it contains today.
C. Its characteristics were accurately described by both Bowen and Hess.
D. Its temperature was sufficiently high to produce magmas with high magnesium
content.
E. Its total magnesium content then was roughly equivalent to its magnesium content
today.
Answer:
--------------------------------------------------------------------------------
Q13:
Which of the following most accurately states the main point of the passage?
A. Komatiites provide information about rates of volcanic eruption between 2.5 and
4 billion years ago.
B. Komatiites provide information about how the physical properties of lava in the
Earth’s past compare with those of current-day lava.
C. Komatiites provide evidence that undermines Bowen’s experimental conclusions
regarding the temperatures at which lava exists as a liquid.
D. Komatiites provide evidence that has changed geologists’ ideas about the
characteristics of the Earth’s mantle between 2.5 and 4 billion years ago.
E. Komatiites provide evidence that water in the Earth’s mantle may have reduced
the temperature required for lava to exist as a liquid.
Answer:
*******************
Q14:
Background information: This year, each film submitted to the Barbizon Film Festival
was submitted in one of ten categories. For each category, there was a panel that decided
which submitted films to accept.
Fact 1: Within each category, the rate of acceptance for domestic films was the same as
that for foreign films.
7
Fact 2: The overall rate of acceptance of domestic films was significantly higher than
that of foreign films.
In light of the background information, which of the following, if true, can account for
fact 1 and fact 2 both being true of the submissions to this year’s Barbizon Film Festival?
A. In each category, the selection panel was composed of filmmakers, and some
selection panels included no foreign filmmakers.
B. Significantly more domestic films than foreign films were submitted to the
festival.
C. In each of the past three years, the overall acceptance rate was higher for foreign
than for domestic films, an outcome that had upset some domestic filmmakers.
D. The number of films to be selected in each category was predetermined, but in no
category was it required that the acceptance rate of foreign films should equal that
of domestic films.
E. Most foreign films, unlike most domestic films, were submitted in categories with
high prestige, but with correspondingly low rates of acceptance.
Answer:
*******************
Q15:
Kernland imposes a high tariff on the export of unprocessed cashew nuts in order to
ensure that the nuts are sold to domestic processing plants. If the tariff were lifted and
unprocessed cashews were sold at world market prices, more farmers could profit by
growing cashews. However, since all the processing plants are in urban areas, removing
the tariff would seriously hamper the government’s effort to reduce urban unemployment
over the next five years.
Which of the following, if true, most seriously weakens the argument?
A. Some of the by-products of processing cashews are used for manufacturing paints
and plastics.
B. Other countries in which cashews are processed subsidize their processing plants.
C. More people in Kernland are engaged in farming cashews than in processing
them.
D. Buying unprocessed cashews at lower than world market prices enables cashew
processors in Kernland to sell processed nuts at competitive prices.
E. A lack of profitable crops is driving an increasing number of small farmers in
Kernland off their land and into the cities.
Answer:
*******************
Q16:
Researchers are using computer images to help surgeons plan difficult operations and to
develop programs that will work for doctors and nurses in the same way that flight
simulators do for pilots, letting medical personnel practice their techniques and test their
reflexes before they ever see a patient.
8
A. plan difficult operations and to develop programs that will work for doctors and
nurses in the same way that flight simulators do
B. plan difficult operations and develop programs to work for doctors and nurses the
same way as with flight simulators
C. to plan difficult operations and to develop programs that will work for doctors and
nurses like flight simulators
D. plan difficult operations and in developing programs to work for doctors and
nurses the same way as flight simulators
E. to plan difficult operations and developing programs that will work for doctors
and nurses like flight simulators do
Answer:
*******************
Q17:
By the same techniques used for genetically enhancing plants, making them disease- or
pest-resistant, researchers have been able to increase the amount of protein in potatoes,
sweet potatoes, and tobacco.
A. By the same techniques used for genetically enhancing plants, making them
B. With the same techniques to genetically enhance plants, so that they are
C. Employing the same techniques used to genetically enhance plants so that they are
D. Employing the same techniques to genetically enhance plants, which makes them
E. Employing the same techniques for genetically enhancing plants that make them
Answer:
*******************
Q18:
Several financial officers of the company spoke on condition that they not be named in
the press reports.
A. that they not be named
B. that their names will not be used
C. that their names are not used
D. of not having their names
E. of not naming them
Answer:
*******************
Q19:
Industrial accidents are more common when some of the people in safety-sensitive jobs
have drinking problems than when none do. Since, even after treatment, people who
have had drinking problems are somewhat more likely than other people to have drinking
problems in the future, any employer trying to reduce the risk of accidents should bar
anyone who has ever been treated for a drinking problem from holding a safety-sensitive
job.
Which of the following, if true, most seriously undermines the argument above?
9
A. Some companies place employees who are being treated for drinking problems in
residential programs and allow them several weeks of paid sick leave.
B. Many accidents in the workplace are the result of errors by employees who do not
hold safety-sensitive jobs.
C. Workers who would permanently lose their jobs if they sought treatment for a
drinking problem try instead to conceal their problem and continue working for as
long as possible.
D. People who hold safety-sensitive jobs are subject to stresses that can exacerbate
any personal problems they may have, including drinking problems.
E. Some industrial accidents are caused by equipment failure rather than by
employee error.
Answer:
*******************
Q20:
Researchers have announced that the magnetic fields emitted by one manufacturer’s
security surveillance system, of which type there are 200,000 worldwide, can interfere
with pacemakers and that this interaction can bring on missed or irregular heartbeats,
nausea, breathlessness, dizziness, and even fainting.
A. of which type there are 200,000 worldwide, can interfere with pacemakers and
that
B. of which, worldwide, there are 200,000 of that type, and which can interfere with
pacemakers, and
C. of which type there are 200,000 worldwide, they can interfere with pacemakers
and
D. 200,000 of which type exist worldwide, interfering with pacemakers, and
E. 200,000 of that type existing worldwide and can interfere with pacemakers, and
Answer:
*******************
Q21:
Most of the purported health benefits of tea comes from antioxidants—compounds also
found in beta carotene, vitamin E, and vitamin C that inhibit the formation of plaque
along the body’s blood vessels.
A. comes from antioxidants—compounds also found in beta carotene, vitamin E, and
vitamin C that
B. comes from antioxidants—compounds that are also found in beta carotene,
vitamin E, and vitamin C, and they
C. come from antioxidants—compounds also found in beta carotene, vitamin E, and
vitamin C, and
D. come from antioxidants—compounds that are also found in beta carotene, vitamin
E, and vitamin C and that
E. come from antioxidants—compounds also found in beta carotene, vitamin E, and
vitamin C, and they
Answer:
*******************
10
Q22 to Q25:
While acknowledging that
there are greater employment
opportunities for Latin Ameri-
Line can women in cities than in the
(5) countryside, social science
theorists have continued to
argue that urban migration
has unequivocally hurt
women’s status. However,
(10) the effects of migration are
more complex than these
theorists presume. For
example, effects can vary
depending on women’s
(15) financial condition and social
class. Brazilian women in the
lowest socioeconomic class
have relatively greater job
opportunities and job security
(20) in cities than do men of the
same class, although there is
no compelling evidence that
for these women the move to
the city is a move out of pov-
(25) erty. Thus, these women
may improve their status in
relation to men but at the
same time may experience
no improvement in their
(30) economic standing.
In addition, working outside
the home, which is more
common in urban than in rural
areas, helps women in the
(35) lowest socioeconomic class
make contacts to extend
exchange networks—the flow
of gifts, loans, or child care
from those who currently
(40) have access to resources
to those who do not. Moreover,
poor women working in
urban areas actively seek to
cultivate long-term employer-
(45) employee relations. When
11
an emergency arises that
requires greater resources
than an exchange network
can provide, these women
(50) often appeal for and receive
aid from their wealthy employers.
However, the structure
of many poor women’s
work—often a labor force
(55) of one in an employer’s
home—makes it difficult for
them to organize to improve
their economic conditions in
general.
(60) Not surprisingly, then,
Latin American women in the
lowest socioeconomic class
differ in their opinions about
the effects of urban migration
(65) on their lives. Some find
urban living, with access to
electricity and running water,
an improvement and would
never return to the country-
(70) side. Others, disliking the
overcrowding and crime,
would return to the countryside
if there were work
opportunities for them there.
(75) Thus, urban life has had both
negative and positive impacts
on women’s lives. In general,
urban migration has not
provided economic pros-
(80) perity or upward mobility for
women in the lowest socioeconomic
class, despite their
intelligent and energetic utilization
of the resources
available to them.
--------------------------------------------------------------------------------
Q22:
In the first paragraph, the author refers to the experiences of Brazilian women most
probably in order to
12
A. support an earlier assertion made by social science theorists about the effects of
urban migration
B. provide an example of one area in which urban migration has failed to improve
Latin American women’s lives
C. substantiate the claim that the effects of urban migration cannot be easily
characterized
D. illustrate the effect that urban migration has had on the economic status of Latin
American women
E. compare the effect that urban migration has had on the economic status of Latin
American women with its effect on the economic status of Latin American men
Answer:
--------------------------------------------------------------------------------
Q23:
NOTE: You must scroll to read the answer choices for this question.
Which of the following best summarizes the main point of the passage?
A. Although Latin American women disagree about the effects urban migration has
had on their lives, they agree that migration has provided them with greater
opportunities for stable employment outside the home.
B. Although urban migration has improved the quality of life for Latin American
women, it has weakened the social support systems that these women enjoyed in
rural communities.
C. The effects that urban migration has had on Latin American women’s lives are
complex and are best evaluated in light of a range of issues concerning Latin
American women’s overall quality of life.
D. The effects of urban migration in Latin America are different for men than they
are for women because of the relatively greater job opportunities and job security
enjoyed by women in urban areas.
E. Urban migration has led to an increasing disparity between the economic
prosperity of Latin American women in the lowest socioeconomic classes and that
of women in the higher socioeconomic classes.
Answer:
--------------------------------------------------------------------------------
Q24:
The author mentions which of the following as a disadvantage of urban employment for
Latin American women in the lowest socioeconomic group?
A. It is difficult for these women to obtain reliable, long-term employment.
B. It is difficult for these women to organize effectively in order to obtain better
wages.
C. It is difficult for these women to find employers who are supportive when
emergencies arise.
D. The structure of their jobs makes it difficult for these women to participate in
exchange networks.
13
E. Working in urban areas makes these women more vulnerable to health problems
than they would be in rural areas.
Answer:
--------------------------------------------------------------------------------
Q25:
The author of the passage would most likely agree that the opinions of the Latin
American women discussed in the third paragraph (lines 60-85) are influenced by the
A. fact that urban life has provided them with greater opportunities for upward
mobility than did rural life
B. relative importance they place on the benefits of urban exchange networks in
comparison to those of rural networks.
C. relative importance they place on the conveniences and drawbacks of urban life in
comparison to those of rural life
D. difference in the effects of urban migration on women of higher and lower
socioeconomic classes
E. difference in the effects of urban migration on men and women of the same social
and economic class
Answer:
*******************
Q26:
In countries where automobile insurance includes compensation for whiplash
injuries sustained in automobile accidents, reports of having suffered such injuries
are twice as frequent as they are in countries where whiplash is not covered. Some
commentators have argued, correctly, that since there is presently no objective test for
whiplash, spurious reports of whiplash injuries cannot be readily identified. These
commentators are, however, wrong to draw the further conclusion that in the countries
with the higher rates of reported whiplash injuries, half of the reported cases are spurious:
clearly, in countries where automobile insurance does not include compensation for
whiplash, people often have little incentive to report whiplash injuries that they actually
have suffered.
In the argument given, the two boldfaced portions play which of the following roles?
A. The first is a finding whose accuracy is evaluated in the argument; the second is
an intermediate conclusion drawn to support the judgment reached by the
argument on the accuracy of that finding.
B. The first is a finding whose accuracy is evaluated in the argument; the second is
evidence that has been used to challenge the accuracy of that finding.
C. The first is a finding whose implications are at issue in the argument; the second
is an intermediate conclusion that has been used to support a conclusion that the
argument criticizes.
D. The first is a claim that the argument disputes; the second is a narrower claim that
the argument accepts.
E. The first is a claim that has been used to support a conclusion that the argument
accepts; the second is that conclusion.
14
Answer:
*******************
Q27:
Personnel officer: The exorbitant cost of our health-insurance benefits reflects the high
dollar amount of medical expenses incurred by our employees. Employees who are out
of shape, as a group, have higher doctor bills and longer hospital stays than do their
colleagues who are fit. Therefore, since we must reduce our health-insurance costs, we
should offer a rigorous fitness program of jogging and weight lifting to all employees,
and require employees who are out of shape to participate.
The conclusion reached by the personnel officer depends on which of the following
assumptions?
A. A person who is fit would receive a routine physical checkup by a doctor less
regularly than would a person who is out of shape.
B. The medical expenses incurred by employees who are required to participate in
the fitness program would be less than those incurred by employees who are not
required to participate.
C. The strenuous activities required of out-of-shape employees by the program
would not by themselves generate medical expenses greater than any reduction
achieved by the program.
D. The fitness program would serve more employees who are out of shape than it
would employees who are fit.
E. The employees who participate in the fitness program would be away from work
because of illness less than would the employees who do not participate.
Answer:
-----------------------------------------------------------------------------------------------------------
Q28:
In countries where automobile insurance includes compensation for whiplash
injuries sustained in automobile accidents, reports of having suffered such injuries
are twice as frequent as they are in countries where whiplash is not covered.
Presently, no objective test for whiplash exists, so it is true that spurious reports of
whiplash injuries cannot be readily identified. Nevertheless, these facts do not warrant
a conclusion that has been drawn by some commentators: that in the countries with
the higher rates of reported whiplash injuries, half of the reported cases are spurious.
Clearly, in countries where automobile insurance does not include compensation for
whiplash, people often have little incentive to report whiplash injuries that they actually
have suffered.
In the argument given, the two boldfaced portions play which of the following roles?
A. The first is a claim that the argument disputes; the second is a conclusion that has
been based on that claim.
B. The first is claim that has been used to support a position that the argument
accepts; the second is a position that the argument rejects.
15
C. The first is a finding whose accuracy is evaluated in the argument; the second is
the judgment reached by the argument concerning the accuracy of the finding.
D. The first is a finding whose implications are at issue in the argument; the second
is the judgment reached by the argument concerning one alleged implication.
E. The first is a finding, the explanation of which is at issue in the argument; the
second is an objection that has been raised against the explanation that the
argument defends.
Answer:
*******************
Q29:
Researchers agreed that the study of new treatments for heart attack patients was
extremely important but more research was needed to determine that balloon angioplasty
preceded with ultrasound was or was not any better for heart attack patients than the
balloon procedure by itself.
A. more research was needed to determine that balloon angioplasty preceded with
ultrasound was or was not any better for heart attack patients than
B. more research was needed for determining whether or not balloon angioplasty
preceded by ultrasound is any better for heart attack patients than is
C. that more research was needed to determine whether balloon angioplasty preceded
by ultrasound is any better for heart attack patients than
D. that more research was needed to determine that balloon angioplasty preceded
with ultrasound was any better for heart attack patients than
E. that more research was needed for determining that balloon angioplasty preceded
by ultrasound is or is not any better for heart attack patients than is
Answer:
*******************
Q30:
An exceptionally literate people, more Icelanders publish books per capita than do the
people of any other nation.
A. more Icelanders publish books per capita than do the people of
B. more Icelandic books are published by Icelanders per capita than by the people of
C. Icelanders publish more books per capita than do the people of
D. Iceland’s per capita publication of books is higher than that by
E. the per capita publication of Icelandic books is higher than that in
Answer:
*******************
Q31:
When a ton of wheat was being moved from Buffalo to New York City before the
opening of the Erie Canal in 1825, it took three weeks and cost $100, but when the canal
was used, it took less than eight days and cost less than $6.
A. When a ton of wheat was being moved from Buffalo to New York City before the
opening of the Erie Canal in 1825, it took three weeks and cost $100, but when
the canal was used, it
16
B. Moving a ton of wheat from Buffalo to New York City before the opening of the
Erie Canal in 1825, it took three weeks and cost $100; using the canal, it
C. Before the opening of the Erie Canal in 1825, moving a ton of wheat from Buffalo
to New York City took three weeks and cost $100; on the canal the move
D. In 1825, before opening the Erie Canal, to move a ton of wheat took three weeks
from Buffalo to New York City, which cost $100; moving it on the canal
E. To move a ton of wheat in 1825 from Buffalo to New York City took three weeks
before the opening of the Erie Canal, costing $100, but in using the canal, the
move
Answer:
*******************
Q32:
A major chemical spill occurred five years ago at Baker’s Beach, the world’s sole nesting
ground for Merrick sea turtles, and prevented nearly all the eggs laid that year from
hatching. Yet the number of adult female Merricks returning to lay their eggs at Baker’s
Beach has actually increased somewhat since five years ago. Clearly, environmentalists’
prediction that the world’s Merrick population would decline as a result of the spill has
proven unfounded.
Which of the following, if true, most seriously undermines the argument offered in
refutation of the environmentalists’ prediction?
A. The chemical spill five years ago occurred at a time when there were neither
Merrick sea turtles nor Merrick sea turtle eggs on Baker’s Beach.
B. Female Merrick sea turtles begin returning to Baker’s Beach to lay their eggs
when they are ten years old.
C. Under normal conditions, only a small proportion of hatchling female Merrick sea
turtles survive in the ocean until adulthood and return to lay their eggs at Baker’s
Beach.
D. Environmental pressures unrelated to the chemical spill have caused a significant
decline in the population of one of the several species of sea birds that prey on
Merrick sea turtle eggs.
E. After the chemical spill, an environmental group rejected a proposal to increase
the Merrick sea turtle population by transferring eggs from Baker’s Beach to
nearby beaches that had not been affected by the spill.
Answer:
*******************
Q33:
In order to reduce the number of items damaged while in transit to customers, packaging
consultants recommended that the TrueSave mail-order company increase the amount of
packing material so as to fill any empty spaces in its cartons. Accordingly, TrueSave
officials instructed the company’s packers to use more packing material than before, and
the packers zealously acted on these instructions and used as much as they could.
Nevertheless, customer reports of damaged items rose somewhat.
17
Which of the following, if true, most helps to explain why acting on the consultants’
recommendation failed to achieve its goal?
A. The change in packing policy led to an increase in expenditure on packing
material and labor.
B. When packing material is compressed too densely, it loses some of its capacity to
absorb shock.
C. The amount of packing material used in a carton does not significantly influence
the ease with which a customer can unpack the package.
D. Most of the goods that TrueSave ships are electronic products that are highly
vulnerable to being damaged in transit.
E. TrueSave has lost some of its regular customers as a result of the high number of
damaged items they received.
Answer:
*******************
Q34:
Books in European libraries last longer than books in libraries in the United States
because, although the climate in Europe is fairly humid, libraries there are not subjected
to the extremes of temperature and humidity that damage collections in the United States.
A. because, although the climate in Europe is fairly humid, libraries there are not
subjected to the extremes of temperature and humidity that
B. because, although the climate in Europe is fairly humid, libraries there are not
subject of the extremes of temperature and humidity as those that
C. because, although the climate in Europe is fairly humid, libraries there are not
subjected to the extremities of temperature and humidity as
D. because the climate in Europe is fairly humid, but still libraries there are not
subject of the extremes of temperature and humidity as
E. because the climate in Europe is fairly humid, but libraries there are not subjected
to the extremities of temperature and humidity such as those that
Answer:
-----------------------------------------------------------------------------------------------------------
Q35 to Q37:
(The following was excerpted from
material written in 1988.)
For over a decade the
most common policy advice
given to developing countries
Line by international development
(5) institutions has been to copy
the export-oriented path of the
newly industrializing countries,
the celebrated NIC’s. These
economies—Brazil, Hong
(10) Kong, Mexico, Singapore,
18
South Korea, and Taiwan—
burst into the world manufacturing
market in the late 1960’s
and the 1970’s; by 1978 these
(15) six economies, along with India,
enjoyed unequaled growth
rates for gross national product
and for exports, with exports
accounting for 70 percent of
(20) the developing world’s manufactured
exports. It was,
therefore, not surprising that
dozens of other countries
attempted to follow their model,
(25) yet no countries—with the possible
exceptions of Malaysia
and Thailand—have even
approached their success. In
“No More NIC’s,” Robin Broad
(30) and John Cavanagh search for
the reasons behind these failures,
identifying far-reaching
changes in the global economy—
from synthetic substitutes
(35) for commodity exports to
unsustainable levels of foreign
debt—as responsible for a glut
economy offering little room for
new entrants. Despite these
(40) changes, the authors maintain,
the World Bank and the International
Monetary Fund—the
foremost international development
institutions—have
(45) continued to promote the NIC
path as the way for heavily
indebted developing countries
to proceed. And yet the futility
of this approach should,
(50) according to the authors, be
all too apparent so many years
into a period of reduced growth
in world markets.
--------------------------------------------------------------------------------
Q35:
19
Given the information in the passage, which of the following is a true statement about the
NIC’s?
A. Their economic success among developing countries has been exceeded only by
the successes of Malaysia and Thailand.
B. By 1978 they produced 70 percent of the world’s manufactured exports.
C. In the late 1970’s, their growth rates for gross national product were among the
highest in the world.
D. In recent years their development has been heavily subsidized by major
international development institutions.
E. They received conflicting policy advice from international development
institutions in the late 1960’s and the 1970’s.
Answer:
--------------------------------------------------------------------------------
Q36:
The author of the passage most clearly implies that Broad and Cavanagh disagree with
the World Bank and the International Monetary Fund about which of the following?
A. The ways in which the global economy has changed in recent years
B. The causes of the unsustainable levels of foreign debt that the developing
countries have incurred in recent years
C. The level of foreign debt that should be maintained by developing countries
D. The degree to which international development institutions should monitor the
growth of developing countries
E. The degree to which heavily indebted developing countries should emphasize
exports in their overall economic strategy
Answer:
--------------------------------------------------------------------------------
Q37:
The author mentions Malaysia and Thailand in order to
A. acknowledge the appearance of implausibility in a broad claim
B. concede the possible existence of counter-examples to a generalization
C. offer additional evidence in support of a disputed conclusion
D. illustrate the broad applicability of a hypothesis
E. admit the limited scope of a standard analysis
Answer:
*******************
Q38:
The ancient Nubians inhabited an area in which typhus occurred, yet surprisingly few of
their skeletons show the usual evidence of this disease. The skeletons do show deposits
of tetracycline, an antibiotic produced by a bacterium common in Nubian soil. This
bacterium can flourish on the dried grain used for making two staples of the Nubian diet,
beer and bread. Thus, tetracycline in their food probably explains the low incidence of
typhus among ancient Nubians.
20
Which of the following is an assumption on which the argument relies?
A. The tetracycline deposits did not form after the bodies were buried.
B. The diseases other than typhus to which the ancient Nubians were exposed would
not be affected by tetracycline.
C. Typhus is generally fatal.
D. Nubian grain became contaminated with tetracycline-producing bacteria prior to
being harvested.
E. Bread and beer were the only foods eaten by the ancient Nubians which could
have contained tetracycline.
Answer:
Compare with Q30 in Sep. 18th, 2002.
Q30:
The ancient Nubians inhabited an area in which typhus occurs, yet surprisingly few of
their skeletons show the usual evidence of this disease. The skeletons do show deposits of
tetracycline, an antibiotic produced by a bacterium common in Nubian soil. This
bacterium can flourish on the dried grain used for making two staples of the Nubian diet,
beer and bread. Thus, tetracycline in their food probably explains the low incidence of
typhus among ancient Nubians.
Which of the following is an assumption on which the argument relies?
A. Infectious diseases other than typhus to which the ancient Nubians were exposed
are unaffected by tetracycline.
B. Tetracycline is not rendered ineffective as an antibiotic by exposure to the
processes involved in making bread and beer.
C. Typhus cannot be transmitted by ingesting bread or beer contaminated with the
infectious agents of this disease.
D. Bread and beer were the only items in the diet of the ancient Nubians which could
have contained tetracycline.
E. Typhus is generally fatal.
Answer:
*******************
Q39:
In addition to being China’s first administrators, in the sense that they developed a
coherent bureaucracy for their empire, the first literate culture in East Asia were the
Shang, and they were well known for crafting ornate bronze ritual vessels.
A. the first literate culture in East Asia were the Shang, and they were well known
for crafting
B. the first literate culture in East Asia were the Shang, well known as the crafters of
C. the Shang, as the first literate culture in East Asia, is well known for its crafting
D. the Shang were the first literate culture in East Asia and are well known as the
crafters of
E. the Shang were the first literate culture in East Asia and well known for its crafted
Answer:
21
*******************
Q40:
Investment banks often have conflicting roles. They sometimes act for a client company
by raising capital from other investment institutions as advantageously as possible, but
their analysts also sometimes send unfavorable reports on the financial health of
companies for whom they are raising capital to other clients who wish to make
investments. Analyses of companies’ financial health need to be unbiased if an
investment bank is to achieve long-term success.
If the statements above are true, which of the following practices, if adopted by an
investment bank, would hinder its long-term success?
A. Evaluating and rewarding the bank’s analysts on the basis of recommendations
made by managers who are solely engaged in raising capital for clients
B. Using reports by the investment bank’s analysts to determine how best to raise
capital for a client
C. Sharing the task of raising capital for a client with other investment banks
D. Ensuring that conflicts between analysts and those who raise capital for clients are
carefully mediated and resolved by impartial arbitrators
E. Monitoring the success or failure of analysts’ current predictions about how
companies will perform financially, in order to determine the value of future
predictions
Answer:
*******************
Q41:
Elk now live almost solely in the Rocky Mountains, which would make it seem that elk
are mountain dwellers, while they once ranged over virtually all of the continental United
States except for a small strip in the extreme Southwest.
A. Elk now live almost solely in the Rocky Mountains, which would make it seem
that elk are mountain dwellers, while
B. The fact that elk now live almost solely in the Rocky Mountains would make it
seem that they are mountain dwellers, but
C. It would seem that elk would be mountain dwellers because of their living now
solely almost in the Rocky Mountains, but still
D. Now living almost solely in the Rocky Mountains, it would seem that elk were
mountain dwellers, although
E. It seems that elk would be mountain dwellers from the fact that they now live
solely almost in the Rocky Mountains, since
Answer:
*******************
Answer:
CBECA,ABABE,BDDEE,ACACD,DCCBC,CCDBC,CBBAC,EBA(B)DA,A

Vous aimerez peut-être aussi